Sie sind auf Seite 1von 112

BLACKWELL'S

V I M B H U S W , MJl
Ll~liversi~y of California, San Francisco, !:lxw o f 7(1(11
Series Editor, Diagnostic hdiologiss

VISHAL PALL, MBBS


Gorrrnmcnt Mcdiral Cr,llrge, Cha~ldig-~rh. Intlia, Clasr o f l9i)fi Series Editcrr, LJ, of Texas. Galveston, Rrsidrnt in Internal Medicine & Prrvcntir,e Medicine

TAO LE,MD
Uuivrrsity of Califoi-tlia, San Francisco. Class or 10116

ALFXANDER GRTMM, MD
St. L o u ~ s Urliversity Schuol of Meclicirle.

CIms nf 1999

Blackwell
Science

CONTRIBUTORS
Robert Nason I'nivrrsit~ of'Texas Medical Rmnct~, Class of " ( 1 0 9

Tishiha Wang University of Texas M~diral Branch. Class of 21702

Kristen Lem Mygdal, iMD Ur~ivetsitvo ~ b n s School : ~ ~ of ILZerlicinr. R r s i r l e ~ in ~t Rarliolog


Fadi Abu Shahin. MTI Uni~crsin, o f Uaniascl~s. Svria. Class of IY!Cl

Jose M. Kerro, MD La SSae University3Mexico Citv


Vipal Soni, MD LIC.L4 Srhonl nf hledi~irlr. Class nF 191)tI
Hoang Nguyen, MD, MEA For tl~weqternUniversity. Clay\ of 2001

Editorial Officcs: Chmmerce Placc. 330 Main Strrct. M:klrle~~. Mass;~clii~setts ( P ! 14H. I!S#l nsnev Mead, Dxfnrrl OX? OEL.. Enalaiid 25Jul111S~r,ccl, Lo~lrluirWC1 N 'll3S. End;ultl 23 Ainslic Plscc. Lclinbul.gIl El43 641. Scntland 54 Clniv~*rsir.y Strcct. C:a~.lron. Vic1nl.i~3053. ?\listralia

.%cq~~isitio~~s: Laura Uell'ortn~ Ilevelopmcnt: Amr: Nritth1-clck Pmdurtion: Imma Hind ;lnd Sliai+'tiC~i~skr-rger Mar~ii~dcuirin#: Lis:~ Fl;ula~an fiarlirlins Manap-r: K a l h l ~ e Mulcally i~ Crwer desig11by L d i c 1laimcr Inwr,ior dccig~ihv Shawn Girsberger Tvpcxet I)\:TcchDookf Printed a n d hn~lnd hv Capital City Prevv

Other Editorid O f f j c ~ ~ : 1llack~~-tl W~c~cnccl~aItq-V Chn ~ ~ hl rl~ 1,g l<ui.K~rstendamln 57. 107117 H r r l i ~Germaiiv ~, Rlarkwrll Sriencr K l i hlC: l i o d t . n n ~ ~ r hRuilding. o 7-10 K O ~ C I I I I I ; INil~u~~j!xl>l~i, ~~II> ( Jli~to-ktt3 7irL.m 104,1ap:3n Irlwa S k ~ t Llnivct>~lv c PI.CLS. . I U l dh ~~ f ' ~ S 11 C~C~ICC C:urnlxwv. 2121 5 . Statr Avcnue. Xmcs, low 500 I +H'404la Ll\ i
Dish-ibutors: T f l. ~ ' I ?ll<-ni<fi Rt,irk\+rll PrlhlisI~in~ c / o nrnc: P,O. Rnx 20 50 Wuizcr Spnr t Lane Willis~rm. \ T0 5 4 9 ~ 4 0 2 0 ('l*clepll<>nu nr-ders SO(Y21li-25!!2: AX o r d c ~ H ~O: ? 4 f i I-7fi26)
rlzirfrrrlin

Blackwell's Underqround Clinical Vifpettes: Pathophysiology TI, 3e ISDM O-(j39-04.55?-1

'l'he BlacliweIl Scicnre lngc i q uadr mark of RlarkweII Sc irncr Lirl., i,c.giutc.rcrlAT thc L111irerl Rrngcloni Trdcle Ll.la~-LqRcgi5irv

T~I,IL~IWII Scivl~cc I*!\,, l,ic!. 54 Ilnivcrqiry Strvet IL~rliun. l'ictr>ria :%f15:% (Trlrpl~oncnrrlrn: OL%934'i-0300: tLm~ ~ r r l03-~~3~1~4-301 e~~. ti \ OTI/$II!P '/'I).+ A I ~ I ~ ~ I I(trt11 YI,A I ~I+/TII/~~< Rl;tcknmc~ll S( ieilcc. L.tcl. C / O Mllmlnr~ Bonk k n i c r c . I d . P.O. Box 2li9

Library nf Cong-ess Catalo~ng-in-Publication Data Rhushan, Vikau. BlacLwcEl'c iulrlcrgrout~d c l ~ n ~ cvim-nptre%. al I'a~l~oplivsiuio~ J aulhor, t'ikac R l ~ ~ ~ r h an 3rd . ed. p. : cln. - ~ L ' I I ~ I c ~ , ~clinical cIII~c \ iI~ 3 e r r f i j KPV.ed. nf. Pa~hr>phvsiol~~gv / \Skh< D1111~ll;ln. 2t1rl crl. ~19'39. ISRV Ofi324)45:>I-.5 (phk.) I. Pl~!?iinlr>gy, Pat11rhr)~ral - U s e str~dics ' 1 . Ph!ulcran~ - L.iccnses - Lln~ter! Stares Lx;~n~inatlons - Studv p i d r s [DNI ,121I . Clinical Medicine - Cntse Rcport. 2. C1iiiic;~l hledicint - t*tnl)len~s and F:Xer(.i~e~. WR 1 H . 2 R37.ihh 2002J I Tide: L ~ n ~ l ~ r ~ clinical ~~~tricI \-i?pieuc. Pa~E>oplly~ir,lugy 11. I*;ill~oplivsinlo~ II1. Iitle. 1Y. bcrlcs. RR 1 13 R45D 2002 lilG.07'07tLdc21

hbing<lon Ounn O X I 4 41'K


ErigLlnrl (Trlephnne nrc1lt.m: 4-W 123T4fi500: I A K ori1c.n: 4-P 11 2354fi5r755) All l i ~ h t res~ivctl. s Kr, ]la!-Iof this honk m:w h rt'pnx1u~~cl in itlit. f r ~ m vr Ilv ;~riy rlcctn,i~ico r nierlianical means, incl~lrling infomiation storage ;unrl rc1ricv;)l svslctns, u'~tllr>ut pc~,n~iscimn in writing frnln thr pt~hlislirr.rxrrpl hy ;I rrvie~vrr who may q l ~ n t I,littf r p&svaRes in ;t revit-w.

No tire The a ~ ~ t h nof' i s this volume have takrn care that rhr infnrmation ct~ntainrdhr.1-eir~ is nccunte and cornpatiM e with rhe swncl:~rds genrr.11~;ic-r-epred ; ~thy t tinic. of pul,licatiort. Nwcrrl~rles~, i t is ditficult to enrurc that :ill rhr infnrniaeioll p+n is rnrirt.1~ acrlwate F r ~ all r ~irci~nlsr;~nct-s. ~ I C .puh1ishc.r ;mrl a ~ i t l i n m dn n o t g~lk1miI~C [he ' ct,nLcliW ~ L ~ l l l li > s c ~ ):ht~cl k clisclairn any linhiliry, Ins?. 01. d a m a ~ r inc~~rrrcl a . ; a consequence, direcrty u r irtdirert I?. :.>I' IIIPus? anrl appliration r)i' anv n i the ctmtents uf ?Ibis\.ol~irnc.

CONTENTS
Acknowledgments Preface to the 3rd Edition How t o Use This Book AbbreGations General Surgery
x

xiii
mFi

xvii
Abdominal Aoxtic Aneurysm Acute Cholecystitis Appendicitis Cecal Carcinoma Esophageal Carcinoma Gastric Carcinoma Gastric Leiomyoma Intestinal Obstruction-Acute Tntussusception Pancreatic Carcinoma Down's Syndrome Marfan's Syndrome Acute Lymphocytic Leukemia (ALL) Acute Myelogenous Leukemia (AML)

B Y I= r
,
AA

'7
+
-""

:
I

Genetics
Hematology/Oncology

, <

-*

p,"'

" I

Anemia-Aplastic ;-' Anemia-Autoimmune Hemolytic 1 Antiphospholipid Antibody Syndrome !' ' Burkitt's Lymphoma 1 Chronic Lymphocytic Leukemia (CLL) I ^' Chronic Myelogenous Leukemia (CML) ': , - 7 Deep Venous Thrombosis Disseminated Intmvasmlar Coagulation (DIC) ' ' '
"I

Dysbetalipoproteinemia Hairy Cell Leukemia Henoch-Schonlein Purpura Histiocytosis X-Eosinophilic Granuloma Histiocytosis X-Hand-Schu12eer-Christian

P
*

-a

""$

P ,,

Disease Imfi Histiocytosis X-Letterer-Siwe Disease 1 Aadglcin's Lymphoma " 1 Idiopathic Thrombocytopenic Purpura (PP) ' 1

"
K C

Multiple Myeloma Myelofibrosis with Myeloid Metaplasia Non-Hodgkin's Lymphoma Polycythemia Veza (PCV)
Sarcoma Botryoides

' 'i
A -

I
i

I:

- I

Sickle Cell Anemia

Immunology Infectious Disease Neonatology

Thalassemia-Beta Thrombotic Thrombocytopenic Puxpura r Transfusion Reaction-Acute Hemolytic T m s h i o n Readon-Febrile Nonhernolytic F Von Willebrand's Disease ,I Waldenstriim's Macroglobulinemia " - , Wiskott-Aldrich Syndrome Serum Sickness 1 * Syphilis-Tertiary (Aortitis) Erythroblastosis Fetabs Sudden Infant Death Syndrome (STDS) ' ' Acute Tubular Necrosis (Am) ' Adult Polycystic Kidney Disease (APKD) Alport's Disease Amyloidosis-Primasy Benign Prostatic Hypertrophy (BPH) 1 Bladder Cancer Bladder Outlet Obstruction, Nephrapathy Diabetic Nephropathy Goodpasture's Syndrome Hypertensive Renal Disease IgA Nephropathy {Bergefs Disease) I -Lupus Nephritis Membranopro~iferativeGlomerulonephritis
"
>
8-

rn

'-

8 " .

"

(MPGN)
Membranous Glomerulonephritis Minimal Change Disease Prostate Carcinoma Renal Cell Carcinoma Renal Infarction

mI .
r7-

1
1

Renovascular Hypertension Seminoma Testicular Choriocarcinoma


Testicular Dysgenesis Testicular Teratoma (Mixed) Testicular Torsion Urate Nephropathy Wilrns' Tumor Adult Respiratay Distress Syndrome (ARDS)

,
I

"

>

i
I

"

Pulmonary

Em
FT
I

Asbestosis
Asthma Atelectasis-Postapexative

1
1~

Bronchiectasis

Churg-Strauss Syndxome COPD-Chronic Bronchitis COPD-Emphysema

Fat Embolism Hypessensitivity Pneumonitis Idiopathic Pulmonary Fibrosis (IPF) Lung Carcinoma Malignant Mesothelioma Pleural Effusion
Pneumothorax-Spontaneous Pneumothorax-Tension Primary Pulmonary Hypertension Pulmonay Embolism Saxcoidosis Silicosis

Thro~~gho the l ~ tproduction ofthis book, we have had o he supt1n1-1 or man): friends and colleag-ues. Special thanks to o u r support team including Anu Gupta. PLlldrua Fellows, Anastasia hnclcrson, Srishti Gupta, Mona Pall.-Toriathan Kirsch and Chirag , b i n . For prior contributions we d ~ d n k Gianni Le Nguyrn, Talm Mathur, Alex Cirimm, Sonia Santos and Elizabeth Sanders.
\.ire have enjoved working with a wnrlrl-class ii~ternational pub lishing group at Blackwell Srirnce, including Laura DeYoung, Amv Nu tthrock, Lisa Flanagar~, Slla~vn Girsbergel; Lorna Hind and Gordnn Tihhitts. For help with securing images for the ell tire series we also thank Lee Martin, Kristopherjones, Tina Pan iz7i a i d Petcr Anderson at t h e University o t Alailama, thc Armed Forcrs Institiare of' Pathology. and many of our fellow

Blackwell Science a ~ l t l ~ o r s .
For submitting comments, corrections, editing, proofreading, and assistance across all oS t h e vignette titles in all editions, we c o l l ~ tii~ely c t1i;mk:
Tam Adamovich. Carolyn Alexander, Kris Alden, H e n ~ y E. Aryan, Lynlna~z Racolor, Natalic Barleneva, Ileam Bartholomew, Dchas11isl-iRehem, Sumit Bhatia. Sanjay Rindra, Dave Brinton. <J~~lian Brown, ne Alexi~ndelBrnwnie, Tamara Callahan, David C~nel;, Rryatl Casev, Aaron Gaughey, Hehert Chen. Jonathan C:heng-, Arnold Cheung, Arnold C h i n , Simion Cl~iosea, Yoon Cho. Samuel Chumg, Gretchen C:onant, IrIadirnir Col-ic, CIlristoplirr Cosgr-ove,Ronald Cowan, Kar.ekjn 12. Cunningham, '4. Sean Dalley. h m a D i ~ ~ i c l a r n ~ Sl~nir ~ d i , Das, Ryan Arrrlando Davr,John Dabid, Emmanliel rle la G ~ I zRobert , DeMrllo, Navncut Dhillon, Sharrnila Disqanaiku, David Donson, hdolf Etchegaray, hlea Elwehio, Priscilla A. Frase, David Frenz, Kristirl Caumt-1: Yohanncs C~hreegsiallher, h i 1 Gehi, Tony George, L.14. Govancn, Pat111 GoyaP, AIcx Grimm, Rajeev Cuptit, Ahmad Halim. Sue Hall, David Hasselbacher. Tamra Heimert, Michelle Hig-ley. Dan I-Ioi t, Eric Jack~on. Tirn Jackson, Sundar Jayarman, Pci-Ni Jone, Aarchan Jnshi, Rajni K. Juitla, Faiya7 bpadi, Seth Karp, h r n n S. ICessclheim, Sana Khan, Andrew Pin-wei KO, Francis Kong, Pat11Konity.kv, Warrcn S. Krackov, Benjamin I-I.S. Lau, An11 LaCasc~, Connie Lee, Scott Lee. G u i l l e ~ ~ n Lehmann, o Kevin Leung, PaliI Levett, IYarren Levinsoti, Eric Ley. Ken Lin,

Pavel Lobanov, J. Mark Maddox. Aram Mardian. Sarnir Mektit, Gil MeImed,-JoeMcssirla, Ruberr Mows. Michael Murphy, Vivek Nzlndkami, Siva Narapan, Cawell h'gnyen, Linh N p v e n , Dcanna Nohlt-za. Craig Nodnrft, George Noumi. D a r i t ~ T. Okrida, ,%darnI,. Palance, P a ~ d Pamphn~s,JinfiaPark, Sonny Palel, Kcarclo Pietrohon, Rim L. RahI, Aashira b n d e r i a , Rachan Reddy, Reatriu Reig. Maiilou Reyes, Jererny Richmon, Tai Roe, Rick RuIler, Rajiv Roy, Diego Ruiz, Anthong Russell, Sanjap Sahgal, Ulmimala Sal-kar.John SchiIling, Isahell Schmitt, Daren SchuhmacI~er, Soi~aI Shah, Fadi Abu Shallin. Mae SheikhAli, Edic Shen, Justin Smith,Jol111SnlIak, Lillian Su,Julie Sundararn, Rim Suti, Seth Sweetscr, A n t o ~ ~Talavern, io Merita 'San. Mark T a n a h , Eric Taylor. Jess Thompson, Indi Trehan, Kay~noncI Turner, Okafo Uchenna. Eric U y p a n c o , Richa \.'amla, John Wages. Alan Wang, Eunice U1ang, h d y Mreiss, ,%my Williams, Brian Yang, Hatry Zaky, Ashraf Zaman a n d David Zipf.
h r genernuslv conrribu ting images to thc entire Uulrlw~ly~~ind Clinical I f i g x p l t p Step 1 series, rve collectively thank the staff at Blackwell Science in Qxford, Bostnn. and Bcrlin as well as:

I\xfnrrl. J. ~ M e d i r i ?Osney ~ ~ . Mead: Rlackwell Science Ltd, 1996. Figures 2.14, 2.15, 2.16, 2.27, 2.18, 2.31, 2.35, 2.36, 2.38, 2.43, 2.tj5a. 2.65b, 2.65c, 2.103h, 2.105b. 3.20b, 3.21, 8.27,8.2%, S.733, 8.7fc,10.#lh, lO.gtia, 1P.28a, 14.6, I.Lf.16,13.50.

* Bannisrer R, Reg): N,Gillespie S. Inf~ctiuusI l i s ~ c t . c ~ 2"" , Edition,


Osuev Mead: BIackwell Science Ltrl, 2000. Figures 2.8, 5.4, 5.28. 18.10, MT5.3:!,MT5.6.

Berg D. A d-r~nnrpd Clin &.a/Skillr mnd Phg~icaf L)inposi<c. Rlnrkw~ll Science Ltd., 1999. Figl~res 7.10, 7.12. 7.13, 7.2. '7.3, 7.7, 7.5, 7.9, 8.1, 5.2, 8.4, 8.5. 9.2, 10.2, 11.3, 11.5, 12.6. C~~srhiel-i A, fIe11nessy TPJ, Greenhalgh RRM, Rowley DA, Grace FA. C l i n i r n l . 5 ' 1 1 ? ~Iq)sney . Mead: BIackwell Science Ltd, ilc391j. F i g ~ i r e 13.19, ~ 18.22. 18.33.

* Gitlespie SH , Barnford K M~ifimf Mirml~ioktgand I! fPction a fa


Glrirrrr. Osney Mearl: BIachvell Science Lid, 2000. Eipi-cs 20,23.

Ginsberg 1 . . I.rciure lVotes on Nmruluml, 7." I':clirimn. Osnry Mead: Black~vell Science 1,tcl. 1999. Figurc~ 12.3, 18.3, 18-31).

* Elliott T, Hastingq M, J>esselberger U. Lecture hlutrs m r M~diral


12fiml)iulu~ 3 ' ' , I.~~li/ion. Osiley Mead: Blackwell Sciet~ce IAd,

1 9 9 7 . F i ~ i r e s 2 , 57,8,9, . 11, 12, 14, 15, 16, 17, 19,20,25,26, 27. 29,3U, 34, 35,52,

Mehta MA, Hornrand Ak! Hn~rnnloln~v nl rt Glrmr~, 00~ney Mcad: DIackwc11 Science Ltd. 2000. F i p r e s 22.1, 22.2, 22.3.

a n d we will br

l'leaw let tis know if' your name has been rnissecl or misspelled happy to make the update in the next edition.

PREFACE '60 THE 3RQ EDITION


Wc were v e r y pleased with t h e ove~~.vhelrningly positive stliden t reeclhack Tor the 2nd edition of ollr I h d w u n d C?inirrc! ~ ~ i b r l l ~ / ~ f c series. \Tell nver 1 00,000 copies of the LTCLT book4 are in print and have been used bv shldcnts all over the world.
Over the last hvo years mrc have accu~nulated and incorporated over a thousand "updates" and improvements suggested by you, our r.eaclers. includulg:

I I
I

* many additions of specific boards a i d wards testable contetlt


d~lerinns of reclundant and overlapping cases

reordering and reorganization of all cases in both a new maqter index by case name in each Atlas
correczion of a Cew Cactrial errors

series

diagnosis and kcarmen t updates

* addition of5-20
A nalnm?

new cases in every hook

* and thr addiriorl of clinical e x a m photographs within LTCb-

And most important of all, the third rdition sets now include two brand new COLOR ATLAS; supplcments, one for each Clinical Vignette serics.

* The CTCIJ--13ccsirS r i w r p Cdur Atlar

I ) includes over 250 color plale5. divided into gross pathology, mici~oscopicpathology (I~istology) , hematolop, and mictol->iology ((smears).
(St*

Thc I 'CV-CJdni(*rrlS r i m r ~ C h r Atlas (St+ 2) has over 125 color plates, including patirnt images, dermaloIogy and hmdus-

copy.
Each atlas image is descriptirelv cagtionrd arid linked to its corresponding Step 1 case, Step 2 case, and /or Step 2 MiniCasc.

How Atlas Links Work:


Step 1 Book Codes are:

Step 2 Book Codes are: h = Anatomv RS = Beha\+c>r:tl Sricncu BC: = Rinrhemist.7 5 4 1 = J~!irrc>hinlt>~. Vd. 1 Mr?= hiirra,l~iology Vnt. IT P1 = P a t l ~ n ~ ~ t ~ y s iI'nl. n l o I~ , P2 = P a t l i o p h v s i o l o ~Vnl. ~ 11

P.7 = Pa~huphy>ioIuby. Vfh. 111 PH P l ~ ~ ~ r ~ ~ i ~ ~ c o l o ~ ~ ~

,
Indicates Type u f

\ n
Case Number
Indicates
. I

ER = En~ergenry Medicine I M l = l n r w nal Merliril~c. Vnl. I I M ! ' = Internal hlpriiriue, \bI. 11 hEL1 = Nt.ui~,logv O R = OB/C;\N PED = Pe~liatr~cr SITR = S I I I X ~ ~ ! PSY = P~yrli~atry
M(: = ViniCase

ER-035A, ER-0350

/M-p3-032A Image: I [ = f lumatnlnp


PC;
(:rrlss

UCVl or UCV2 Series

Pathnli~v

PM Microcctrpir lbarIiol[>p
; .

if'rhe Case numhcr (032, 03.5, etr.) is not f o l l o ~ ~ e by d a letter, then thcre is only cine image. Otherwise A, B, C, D indicale up to 4 ima~es,

Bold Faced Zinks: Irl order to give you access to the largest numbcr of-images possiblc, we have cliosen tn cross link he Slep 1 and 2 series.

* IT the link i s l3old-faced this indicates thal the Iink is clirect


(i.e., Step I Case with the Basic Science Step 1 Atlas link).

If~helitlkisnotbold-faced rhi~ indicates that thelink is inclirect (Step 1 c x e wit11 Clinical Sciencc Step 2 Atlas Iink u r vice
versa).

i & havc also irnpl~tn~nred a fm structutal changes z~pon your rcqucst:

Each currcnt and fi~riire edition of mu- popular First Aid fw thr i:SMI,IS Strj) I (Appletoil Pc T,ange/McGraru-Hill) atlrl Irir.~f ilidjnr thr! ITTiVILl St+ 2 (12ppleton kk Lange/McCraw-IIill) honk will be linked to the corresponding UCV rase.
\& eli~ninateclUCZ' + Fimt Aicl links i u they frequently hccu~r~e rjltl of date, as thc firsf A it! t>ooksare re\riscd yearly,

* The Color Atlas is also specially designed for ql~izringcaptions are descvi ptivp and do not ~ i away ~ e the case name directly.

We hope the updated GCV series mill remain a ilrliql~e ancl wellintegrated st~idv tool that provides compact clinical col-relations to basic science information, They are designecl to he easy and ft~n (cornparat;vely) to read, and hrlpful For both licensing exams and the warrls.
We invite your corrections and suggestions Tor thc fourth edition of these books. Fnr the f i n t subillission of tach factual correction or new vignette that is selec~ecffur i~~clusion in the fourth cdition. you will receive a personal acknowledgernenr in thr rerised book. If y o u snbmir over 20 high-qllality corrections, additions o r ncw l-ignet~es we will also consider inviting you to become a "Contihutnr" on the book o f your choice. If you are interrsterl in I~ecoming a potential "Contributor" or "Aulhor" on a future LTCV book, or-working with our tear11 in cIe~,eloping additional books. please also e-mail us your CW/resurne.
U'e prefer that you submit correclions o r sugg-cstionsvia eIectronic maiI to UCVteam@yahno.com.FIcase includr "Underground Vignetres" as the subject of yol~r message. Kyou do not have accesq to e-mail, usc the follo~vingrllailing address: Rlack~vell Publishing. Attn: UCT Editors, 350 Main Str eel. Midden, MA 021 48, USA.

Viltos Rhws/,chon Vishnl Pall


Ton I,P Octob~r 2001

Tlris qeries was originally dcveloperl IO adcll-eqs the inc~exqinji nt~mher c~f clirlical vigmecte ql~estionq otl n~eclical examinations, including the LTSMLE Step 1 and Step 2. It is alsn designcrl to st~pplement ant1 rr~rnplemen I the popular ATTI Aid /;7r I ~ P CL%iZJ.: ,$/PSI I (hppleron 8: Langc/McGmw Hill) and fin! Aid ,fm. tJw tCrSiIfESi+ 2 {Appleton II: Lange/McGraw Hill).
LTCV 1 book uses a serqies of approxirnatelv 100 %upraprototypical" cases a q a way to condense testable f a c and ~ associations. Tllc clini.cal vignetres in this serics arc designed to iricorparatt as manv tcsmhlc facts as possible into a cohesive ant1 metnorable clinical picutre. The ~ i g n e t t e s represenr com~msite\ drawn from general and qpwialry testhook?, reference honks. tFlousands of USMLE swle q ~ t e ~ t i o n and s the personal experience of the authors and r~viewers.
'ach

Althc~ugh each case tends lo present a11 t h signs, ~ symptoms, ancl cliagnostic findings for a particulzr illncss, patients genera 1 T y will not present with such a "complete"picture either clinically or on a medical examination. Cases are not meant to simulate a potential real patient or an exam iignctte. All the boldfaced hbbuzzwords" are for learning purposes and are not r~ece.;.iar-ily expected to he fol~nd in any one patient with the
rliseasc. Definitions o f scIrc tud important trrms arc. placed within the vignrties in (SL~AI.I, ~ A P S )in parent tieqeq. Other pal-en thetical rcrnark~ oftcn ~-efcr to thr pathophysiulop or mechanism of diwaqe. The forinat shot~ld also help stl~rlerlt? learn ta present cares ~ i ~ c c i n c tduring ly oral "huller" presentations on clinical rotations. Thc cases are mcant to serve ;15 a crmdcrlsrd review, not as a prirnarv reference. T h e information pro\-icled in this hook has 13een prcpared wit11 a great cleat of thought and careful rr:search. This br~ok shoulcl not, ho~vever, bc considered as I a u r solr wurce of information. Comctions. sr~ggrs~ions and sithn~issions of new cases are encoi~ragecl and will I>eacknowlcdgcrl and incorporatrd when apprnpriate in fit ruse edE [ions.

5-ASA -4BGs
hRVn

,ICE ArnH XDF-I AFr' A T AIDS ALL ALT AMI,


1l.K

m s
ASD 'IS0 AST

iZV BE BP 13UN
CAD

CALLrZ

cnc
CI-IF CK CI.1, CML
CrnT CNS

COPD CPK CSF CT


CV,4

rnR nIC DIP


T)KA

DM DTRs DVT

5-aminosalicylir acid arterial hlood gases ad tiarn;rcin/b~eomycin/vi~icri~tine/dacarb-nxine angiotunsin-convertin g en q r n e adrenocol-ticotropic hormone aritidt 111-eticlinrmone alpha fetal protein aortic insuff~cierlcy acquired iinm~~nndeliciencv s~ldrr)me acute lymphocytic leukemia nl,~nine t ratlsaminasc acute rnyrlogenous leitkrmia antiiluclear antibody adult respiratory distress syndrome atrial rrpml defect ail ti-ctreprolysin 0 aspartate transaminase arteriovennus h-niiiam enema hlnod preswre blood rtrea nnlrogett coronal? artery discase comnlon acute lyrnphobla5tic leukemia antigen completc blood count congeu~ive heart failurc CI-eatine kill as^ cElronic 1)qnphocytic leukemia chronic myelogennlis Ieukemia C)'~OJI egalo\7irus ~ central nen70ussyiterrl chrnnic nb~tr-ucrive pulmonary disease ci~atine phosphokinase cerebrospinal fluid computed romogl-aphy cerebrovascular accidenr chcst x-ray clisscminaied itlrl-a\mcularcoapibtion dislal inte~=phalan~eal diabetic kctoacidosis diahetec mellitus deep tendon reflexes clcep venous thl-onlhoqis

ERlJ ECG Echo EF EGD EMG ERCP

Epstcin-Barr vir-us rlcctrocardiograpFi~ rchuc;~~c~iu~aphy cjrction fwctior~ esol>haqogaqr mds~nrlenoscopv electromyograpl~~ encloscopic rctro<qaracle cliola1l~opalzrreat~)grap1~~ erythrocyte sedimentation rdtf ESR fi~rcrd cxpiriitnry vnl~tine FE\ ' fine needle aspiration FNiZ arllokody absorption FTIZ-A RS fluoscszcnt truponrmi~l f{~rcecl ti tal capaoi tv FVC glomrnllal- Siltralion 1-ate GFR !growth hnl-monc GH gastrointestinal GI GM-CSF granulocyte macrnphagr colon? ~tin~nlatinfi f ar - .tor ge~iiiot~~ii~a?y GU hepatitis A \ims Hrn' hr~maiz chorionic go~~adotrc~ph in hcC; I-IEENT head, eyes, ears, now, and thl-nat human i r r i mruiorleficiencr, vin-us HnT 11u1nanleukocyte antigc11 HI.:\ history of pl-esen ill ~zess 1-1r l hear1 t a t e HR Imrman l-ahies imrnunc globulin HRrG herrdi tam .;pher<)c~locis I IS idenrification and chief complairi~ ID/(:C: insulin-deperidm~ dia11e1e.i n~ellitm IDDM imrnunoglohiilin Ig insulin-like g r o ~ q h Ijctor IGF itn tmtnuscular IM jugular venous preswu.e kidnevs/ttretrr/hradclcr KLrR lac^ ate dehydrogenase LDH lower esophageal sphincter LES lilrer.fi~nction tests k.F-s% lumbar punutu7-e LY lcft vuntric~~lar LV left ven i r i c ~ar. ~ l1 1 ~ ~ ~ 1 - t r o p h y LvH electrolytes T.yt PS mt-an corp~~rcular Il~moglohin concen t r ~ ~ t i o n MCHC mean carp~~scnlar volurnc \4c\nlrrltiple e~~rlocrinc neoplasia MEh-

. P T

MGUS

MR KHL KIDDM NPO NSXl D


PA

mvocardial infarctio~i mcchlorc.chairiii~eJ\.inc~ist ine (Onco\wrin)/ procarhazir~e/pi~prl~~ison~ rnagn'ctic rrsoilance (imaging) IIO~I-Hodgkin's lymphoma non-i1isr1lin-rle1~~11clent dialrrctcs n~ellii 11s nil per os (nt~thing Fly rrlou th)

PIP PRS
PE

proximal in terghalangcal pt~ysical csam pulmorl;uv Funrtinn

P ~ s PM I I'M N PT PTTA PTH PI-I' PL'D RBC RPR


RK

lerts

RS
RSI

RVH SUFT
STAIIH S LE

~~~wdivvm lzorillonc irl partial ~hromhoplastiri ~ir~re prptic utccr disrase red blood crH rapid plasma l-eagin respiratoq mtr Rcrd-Stn-r~herg(cell) r.i~l>\ t ,e~itrir~~lar rigl~t v~ntricular hvpertrt~plir mall 1 x 1 ~ el . F I ~~tv-th I rc111g11 wndrorne of inappropriate \ecrriic~noTAD1-l scxuall!r tranlirni~ terl disease ~hyroirlr ~ ~ n c t i o tests n lli~roid+timt 11aiing Ilorrnonc total iron-Frinding rnpaciry liansiug~~lar intrahepatic portosY4stenl ir s111~1a t

STD
T I T 5

1P'l

TSI-I TIBC
TIPS

TPO TSH Trl' LT.A LiGI

ul-innlysis upper GI

rs

VIIRI,

Vcncrcal 13iqease Research Lal>oratory


vital s i g n s ventricular tac11vc;~rdia wllite blond culI

1's
IT M'BC
M-PMr

SR

Il.blff-Parkit~son-'It'tiite(s~di*ome) x-ray

IDJCC

An a~y~iptomaLic 60-year-old white male undergoing a routine

phyical exam i s dismvered to have a prilsating abdominal mass.


HPE

T h e patient has a history of occasional abdominal pain and hypercholesteroEemia that has been poorly con tl-ollerl hy diet ancl mr-dication.

PE

Pulsating, painless upper abdominal mass approximately 3 cm in diameter. KUl3, laletxi: calcification of aneurysm wall. CTJUS, abdomen: dilated aorta w i l t 1 il-regular calcifierl wall; large, eccentric m ~ ~ r a l
thmmhus wen.

2
m

s
C

V,

Imaging

; D G)

; D

<

Gross Pathology

Mosl aneur>'srnsaye Incated Iwtweet~renal arteries and iliac hifi~r.catiorb: thron~bus may alsn he 111-esent: in trnrnural clissectinn nlay also he wen. hrlenrysm wall contains all three la!.et-q (inritna, media, advent i tia) (''IHI~F'' b \ ~ ~ ~ ~ ~ % h ~ ) .
Surgical replacement with graft {if > 5 cm or symptorrla~ic) ;

Micro Pathology

Treatment

consider endovascular sten t/gr.af't.


Discussion

The iisk nf rupturt*with potcntiallv fatal bleeding increases w i t h sizc. Al~dominal aortic aneurysm i q ur~rallv cauwrl by atherosclerotic rlispase and is often associated with coronar?. artery disease. I t is also rn11scd by trarima, i~~feclioxi (r.g., syphilis), cystic medial degeneration, and art eri tiq. Sequelae incluclc rupture, embolisalion, infrrtioi~, V~SCIIIJI- O C C I L ~ S ~ O ~ ~ secondxr-y ro rhr-mnhus fortnation, a n d compression of acljacen t structures ( e . g . ,urctcrs, vertebrae).
P ~ T ~PG-P2-001 T-I

Atlas Link

A B D O M I N A L AORTIC ANEURYSM

I DJCC

h 42-year-old wI~itc female. llic mother of five, dt.veIol>sacute


intermittent pain in the right IIpper quadrant ant1 r i ~ h scap111a t after eating a fatty meal.

HPI

She is nCNative American ancestry anrl i.; 30 pn~uncis overweight. Shc also complaii~s of' nausea anrl has vomited thrce times. Skrr has I~ikcIsr\.eral pi.ini. epi~orles ol \ i ~ n i l a r pain rolluwing mrals.
VS: I'cvcr; tachycnrdia. PE: obese; tcnclcrnrss in l i ~t upper l ~ abtlomiiial rl~iarlc~!i L w i i 11 inspiratory arrest on palpation (Xlu K I ~ II ~ SH S A ) ; hypoilctiv~ lmwrl sounds.

PE

Labs

Hvpel-rliolc~terolenlia.CRC: Ie~~kocytosis with mild neutrophilia. Elc\r~ted clii-ec.1bilir~rl~in ; elevated a l k a l i phospl~atase.


IlS: rIistcnrled gallbladder with d thickening r o r ~ t i ~ i r l i i ~ g mliltiple echogenic shadows (stnncs). NIK,HIDA: hilure to \<stla!i7t, # j ~ l l l ~ l ; ~itidic-;~~e< ~ l ~ l ~ r -C ~ Jir S rit~rr ~ nhstl-uction by stone.

Imaging

Cross Pathology

Gailhlaclrlcr inflammatio~iranging tioin wall ecleini~ to acute gang?-rncI\+ th ~iccrr~sis, pur fo~-m;~t irlr~, ancl per.fnr;ltion with p ~ r i i o ~ i i l iMOSI s. S I ~ I I P IXI-P cotnpnwrl of cholesterol: less commc~n are pigmented stones made principally ol- 1rncnuj11~ a i e r I>ilir\~hin l and r a l c i ~ ~ salts. ~n

Micro Pathology Treatment Conscrvarivc trc:ltmcnr includcs n o oral i~ltakc, rli~sogastric aspiralicyn, TCT fluids. analgesic$. ancl anlibiotics: cholecystectomy ( I r.il~;{l ly li~paroucol~ic) is rlclinitivc 1i.ratmeitt. Althou~h cal~iili arr involsccl in moqt c a w s oi'actile rhc~lecsq~itis, acaIculous cases aricr a f1e1ni tnhiliat y inainr surgeries, srvcrc traunia ~ 3 1b u r i ~ smid , ~epsis a~ld i l l I ~ pmlpal-turn P qtate. DiiTei.rntial rliagnosis i n c l ~ ~ r lappcndicilis. c.~ panci-catitis, ~>~i-fi)~ t-d - i l peptic ~ ~ ~ l c r~iv~!c)n~pFiri t; tiq. m\~orardial infarction, and riqht lower lohc pneumonia. Clinical risk factor-s inclttde tl~c four F s :fat, female, forty, and fertile.

Discussion

Atlas Link

ACUTE CHOLECYSTITIS

ID/CC

A 17-vea~old tnale snident presents with anorexia and poorly Ioci~liretl ge~umhilical pain followed by nausea and two
cpisodcs of vomiting.

HPI

Four hours after 131-eser~htion, the pain shifted ta the right lower quadrant and lnc devcIopcd a low-grade fever.
CI rn

PE

1 ' s : mild tarhvrarrlia; lotv-grade Cever. PE: right lower quadrant


tenderness with guarding and rebound; pain in right lower q~mclrartt when pressure applied to left lower qr~adran t (Rol-srnn'q SIGN) ; pain localized to junction of outer and middle third of the line from anierinr snperior iliac spine to l~mbilicus (M(.R~~IZN~,'I"S POIV 1.1 ; rig11t IOIVP~ qltadranl pain elicited 1 3 ~ passive hip flexion (PSC~AS S I ;N) ~ atid bv pas~ive intcr~lal arid external rotatinil of hip ( ~ ~ ~ T L * R A T C sicw). >K

m
Ln

5
m G)
30

<

Labs

CRC: elevated WBC counrf predominance of neutrophils Normal serllrn amvlase. I!A: t~ol-mal.

Imaging

KLJB: right psoas siradow hl~11-red: ~ent-ralized ileus with air-fluid Ie\.els; increased sofl tis~lle density in sight lower quadrant; smalI rarIinp,~ql~r Fecalith i n rig111 lowrr qr~adrant. L T S : noncompressible tubular strl~cturein right lower quadrant.
&rlv Iesinn : hyperr~nic appendix with Iihl-innus exudate; late lecion: purnlent ex11 date wit11 ilccrosis and pcsforatioit; fecalizh occasionally pi-esunt. Appendectomy with preoperative antihiotic coverage. The p ~ a k incidence or appendicitis i s in tlnc second ancl bird decndes. Caures include obstruction by fecaliths (33%) ancl lymphoid hyperplasha (GO<% "u) it is occasioni~H): cazisecl kv ttlrnmrq (carrirlnitl turnor i~ l l ~ e inns1 conimc)n Lllrnor of the appenclix). para~i tes, forcign bodies, and Crohn 's discasc. Complications includc perforation, periappendiceal abscess. per-i~oni tis, and gunm~lizecl or wo~rncl srpsis. Differential diagnosis should inclt~de rnrel-oculitiq. rnesent~ric lym[rharlrtliris,acute salpingi~s, ectopic pregnancy, pain on o-vc~lation ( ~ ~ r r r , ~ s r : ~ arrd -~~~uzj, Meckel'r, tli\le~,lict~litis.

Gross %thology

Treatment

Discussion

AtZas Links

ITElTII PI-a2-003, an-a2-003

APPENDICITIS

HPI

She 11x5 atso notired sipificant weight loss and intermittent diarrhea.
Mar-kerl pallor; palprrhle I r f t ~111~1-aclatic11lar lymph node (VIRC'I IOU' 5 NC)I>F) : palpal>le niaqs in right iliac fosqa: hepatomcgaly.

PE

Labs

C:RC/PRS: microcvtir, h ~ p u c h r ~ ~anemia. ~ t i c Positive stool guaiac test: rlrvatcrd serum rarcinoen1l9r~~orlic ; ~ t - t tigen (CEA)
1ez.t-1%.

Imaging

RE: Iai-~e,i r r q ~ ~ l ah~ngaritlg i. mass in cerum. US: metastatic hepatic nodules. Colonoscopv: largr fungating growth in cecum.
Cauliflower-like, Fungating, nonobstnacting growth in cecum: Inav hc polvpc3ir1, ~ e s ~ i ln e~ . c. ~ n s t r i c t i n ~ .
WellLrliiTerentiated artenoc:~rcinoma.

Gross Pathology

Micro Pathology
Treatment

Righr hpmicolec~nmy w<th remporary colostomv; adjuvant c h ~ ~ nh~t-apy: c ~ t ToSlow up for recurruncc by mori itori ng C:FA levcls.
Earlv detecuon of cccal c;~rrinoma i.i I ~ screening v l'nr occult blood in the stool. It is the suco~ld most conlmon rallse or umnccr d r a t h ; i t s ir~cidellceirlcwaseq markedly alter age 50.

Discussion

Atlas Link

mP G - P ~ - O O ~

CECAL C A R C I N O M A

ID/CC

A 6Ryear-olcl hlack male pruse~lt~j ivi lh anorexia, progressive dysphagia. oclynophagia, arlrl weight loss.

HPI

Tl-lc patirnt has been drinking w r y hot tea since hc was E I years olcl and smokes or~e pack ofrigarcttcs per clay, His history also rcvuals heaby alcohol intake; ncc;lsional cot ~ g hvomiting, , and regurgitation: and scvcrc dysphagia with solids, progressing to liquids.
Emxciadon: fixecl, nonpainful supradavicdar node; pale

C l rn

z m
7u

PE

! e
;e <

Labs

CBC JPBS:hypoct~romic, microcytic anemia. Hemoccult-pusitire srtml; hyoalhuminrl nia.

Imaging

UGl:irre@arfungaTatingesophagealmassinmiddletl~ll-rlof nhstt-~~ction. CT, chest: ii-regular eqophagus ui th esophageal m a s with invasion of mcdiastitilrm and enlarged para-aortic lymph nocle.
Large fungatirig mass prorruding towdrrl esophageal lumen. Sqnamot~s cell carcinoma on biopsy.
Laser ablatiori of t unmr with palliative slenL placement; palliative radiotherapy: surgical rrsecliol~rollowed by cl~emotherapy plus radiotherapy for curable tumors; evenl~ral gastl-ostomy tuhr pl~cernen t.

Gross Pathology

Micro Pathology
Treatment

Discussion

T h e most cornmu11 variant of esophageal carcinoma is squarnous ceU carcinoma, which is assuciated with aEcollol rind tobacco use and is more common in blacks. A less cnmmoiz mriant is adenocminoma. which r~suallyir~trcllves the distal third of thc esophagu.; and is more coinnlon in whiles with Barrett's (glandlilar-metaplasia of thu squanluus epi theli~irl~ of thc disl;tl esophagL~s is causcd hy chronic. untreated gastrocsophageal reflt~x rliscasr).
PG-P2-OOSA, PG-P2-005B

At\as Links

ESOPHAGEAL CARCINOMA

ID/CC

tlrl %?year-old whitc male complains of anorexia, frequent vomiting, a n d a gnawing midepigastric pain of several mnnths' d~u-ation.

HPI

Thc pain is not relieved by antacids or milk. The patient has lost significant weight over t l ~ e past fcw m o n ~ h s due to diarrhea after e v r r y meal.

PE

Pale. emaciated male i n modcrate dist~.ess: left supraclavicular lynph node (\r~'r~t:~nw's NCIDF.) palpablc.

Labs

CRC: hypochromic, rnicroeic anemia. Stool positive for occdt hlood; L R s normal.
UGI: 1;ir.ge fungacing lesioil on grratci- cllrvature of stomach with kislulous tract running to uarlsverse colon. EGD: same. Pol;vpaid, raised, f m g a tirlg tnass p t-qjecting into lrunen : sitnated at distill erld of'qrnmach.

Imaging

Gross Pathology

Micro Pathology

Biopsy rcveals a well-differentiated adenocarcinorna with signet-ring cells.


S ~ ~ r g e rradiotherapy; v; c l~ernotherapy.

Treatment

Discussion

Most cnmmonly found on the lesser curvature in the antrum and pyloric areas. aderlocarcinomas nlav hc one of two types: intestinal ancl difftwe, C11ronic atrophic gaytri [is, pernicious anpmia, infection with H. #'Ion', post~usgical gastric rcmnai~tr;, and type A blood are all predisposing risk factors for the developmcn t 05 adenocarciraoma. I t most cammonlv spreads hematogcnousI!~ tn the liver- and may spread trar~speritoneallr; to thc
rrvaries

(KRUKENRERC TL-MOR).

> <>

*
v

GASTRIC CARCINOMA

ID/CC

A -14-year-old nrale is aclrniltetl to the hospital follo~~ing episodes uf vomiting blood (HI+.M.IAI I ~ U I : S I >ancl ) passi~~ black, g tarry, foul-smelling stools ( LTEI.V,NA) .
THe has experienced recurrent painless hematemesis and melena Ii)r sew t-a1 vears, I x ~repcatcd t c.va1~1xtions have heen negative.
t )

HPE

Labs

CRC: rnicrocytic, hypochromic anemia. N;lsc)gastric aspirate 11x5 coffee-ground appearance.

?
E L
; D

rn

rn

Imaging

UGI/EGLI: fixin m a w in fundus c ~ stomach f with 2-cm ulcer o n

<

Cross Pathology

I'ostoprrativc specinlerl I-evealsa firm, circ~rrr~srribecl nodrllaimass within the gastric wall cowrccl ky Initcosa.

Micro Pathology

Ill~orliug intrrlacetl bzmdles of spincllc-shaped ccrlls: nn ctvidei~ce nr anaplasia. S ~ ~ t g i cresection. al


Gas~ic lcio1nyom;-1i s
1 1 1 most ~ common

Treatment
Discussion

benign tumor of the

stomach.

GASTRIC LEIOMYOMA

IDJCC

A ~lO-vc;~~--old male p~esellts wi ~h crampkg abdominal pain anrl vomiting of 3 hourq* rlul-ation.

HPI

H r also romplains of a n inability to pass stool or flahls ( O I K T I I J A T I For ~ ) the- pitst IS days. Twn years ago, Ilc i~r~clrrwerli an emergency appenrlectr~my Li ,r a r-tlpt~~rerl appcndis.
Ileh!dl-ation ; abdominal distention: gerirr,tli/ed mild tenderness ovrr al)clr>ine~i wi thouc rebouncl o r guarding: tlowel S ~ U I ~ ~ S hearrln$ high-pitched tinkles during pain paroxysms.
CDC:/PRS: Ie~iknqt nri s with I~emoconccn tra tion. Ser.lxrn amylnse levelu ilormal.

PE

Labs

Imaging

XR, ;~htloint=n: " ~ t r p l n c l r l ~pattern r" or ~nultiplc dilated loops of s m d bowel and multiple air-fluid levels: colon and rectlim gaqless (air in rolnn or rectam wor~ld i~idic;~te an in~csrit~al ilenq): iln rrcp ail- unclez dii-lph1~1g111.
J 3 'fluid aild cluctrc~l!re repl;lrrmenr; na50pstric suctir~n/ c l r c r ~ i n ~ ) ~ ~ e br-oad-spectm~m saio~~; antibiotics: sr1rgcr.y.

Treatment

arc Di~cussion T h e most comrnori cause5 OF small howel ohstl-l~rtinn intestinal adhesions secondary to prior abdominal srrrgery, intussuscepiion, volvtplus, :tnrl incnrucratccl hernia: 1l1crrlosi cornmoil cnusus of large I~owelobslrur-tinn arc carcinoma, volvulus, and s i p o i d diverhditis. Complications iriclc~dr strangulation ~ I C necrosis I of the hc>~vrl ~v.111 luading lo perT[~ratiotl, peritonitis. sepsis. arlrl h o c k .

18-monthsld male is bro~ig111 to t11e emergency room by his parent? becallsc of acute, intermittent abdominal pain,abrlonli1x11 distcntiorl. and parsaqe oT'*redcurrant jelly" stools.
A11

The chilrl had previously hcrn wrll, m d h i s iniinl~rtizarion scherlule is roi~ipletc-. I-le vomited twice fi~llowing arlmissian.
Child ca-ying ancl scrcarnir~g, with knees dr;lr\n In al~domcn; ;~bclorn~ tenclcr n and distended; oblong (saurage-shaped) mass in atwIornen ( rnr~st often in right uppcr quad[-all[) that hardens wirh palpation: exartlir~ing finger stainctl will1 mucus and blood o n digit;al rectal exan~ination.

Labs

Ko pm-asiitr.~ on stool exam; no pathogrn on ston7 cultrlre.


XR, ;ilidolnen: ga? i r l small inte.itirlr ancl a h ~ e n c e t>Ccccalgas s h x r l o ~TSE: . telescoping of ileum i n t o cecum.
Zh~r.ing operation, thme lavers ar-e seen: entering or inner tube, rc.ti11ning or middle tube, and sl~eatlr or cmter- tube: orltur ~ u h e rallerl it~tussuscipicr~s; innri- and mirldle togetlzur crtllcd int~zssuscrptum.
Ischemic necrosis M ~ I F I alcn~ghing of rnucora, producing "red currant jcll!~" sronls. Hvrlt-nstatic (barium) or pncrl~nalic(air) reduction using an enema; surgical ruduction c)i. refertion ir that fails or is cont~.aindiciltcdowing t o perfbratinn o r ganFcnc. Ninety-five pcrcrn t nf case5 o f i 11 tus~usccptionare irliopa~liir and usuall!. otiginale near the ileocccal ,jurlctio~i. The condition i s as.ioci;ued with adennvirus infcc~ions, ~.hich produce liyperplasia of Peyer's patches in t l ~ c terminal ireurn, which s e ~ ~ as es r7 1 1irll~s for i ~ ~ t ~ ~ s s ~ ~ c 11 ~i~ p also t i u seen r i . with lead points ( e . ~ .Meckcl . 's diver~ic~llum, prjlyp5, parasites. cluplicatinns, I~cmar~gioinas, a n d suture lincs) .

Imaging

Gross PathaZogy

Micro Pathology

Treatment

Discussion

Atlas Link

INTUSSUSCEPTION

ID/CC

1%

3 1 -y=al.-old malt=complains of pruritus and abdominal pain that radiates to his back along with significant weight loss (15 kg) over tTic past 4 months.

HPI

H c also status that his urine is dark a n d tIiat his stools are claycolored (A(;] ~o~.rr;). He admit? to a history of smoking (60 pxk-ycars) and Ilea\? alcohol u\r ti111 multiple prior houb of pancreatitis.

PE

Cachectic malc; scleral icteru.; (indicate? jaundice); hepatomeply palpable gallbladder ( C r ~ r r u v c ~ r sSII:Y) i ~ ' ~ ': ~hard 8-cm mass palpahl~ in midepigastric reginn. Markedly elevated direct bilirubin (20 mg/dL); ahscnce of ~ ~ r i r l ; ~urr~bili~lvgen; r.) markcclly elevated alkaline phospharase; rnildlv c1t.vatt.d tri111sarni1iasr.s; normal P T elevated caccinoembryonic antigen (CEA) and CA 19-9.
CT/US: mass in head of pancreas; dilated intrahepatic bile ducts. ERCP: abrupt cutoff of main pancreatic duct. LTGI: ilarrowerl lumen ol'duorlenllm. Hitr,rl nodular mass lui th ill-rlefiiied Ircjr-rlers invading parlcreatic pwenc hmla a n d obsfructing c o m m o n bile duct around Iirad of' pancreas wit11 Inca! extension and livcr metastasus.
Pancreatic mass biopsy rrcvcals a pclorlv cliffcrcntiated ductal adenncarcinoma it1 cl ilslers, secrw ing r n ~ ~ c ar~rl i n rlen~e collagenmlq r3esmopl;lsiir strnma.

Labs

Imaging

Gross Pathology

Mino Pathology

Treatment

Surgical pancreat icnr[r~ode~zertomv ('M'lit wtr's I'ROCEDL'RE) ; cIlr=lno~hcrapy; supportive and palliative care (hiliary decnmprewinn to relieve-jaundice; celiac plexus block for pain).
Chronic galIhlaclder disease, diahetc? mcllitus, 'tiureditar? panrr-e;iriI is, chronic pancreatitis, cigarette smoking, diets high in meat and rat. xnrl o c c ~ ~ p a t i n nexposure al tn c a r c i n o ~ n arc s prrdisposing factors. Pancreatic carril-loma carries a poor prognosis (8.5% ;lare already locally inv-nsivt. or-nle~astatic i i lthe time of rlingnosis) nild is associated with a mutation in the I<-ras oncogrnu ancI thc p53 tumor suppressor genc. I:ornplica~ions i t~clode I~ypt-1-c~qiitahilitv (resulting in migratory thromhophlebitis, also known as thc Troussear~ 5ign).
PG-P2-010

Discussion

Atlas Link

PANCREATIC CARCINOMA

I DJCC

A newhorn fir1 is brought into the genetic.; tlrpal-tment for a karyotypu study.
She was Imrn or a 45-year-oldmother w h o f c d s t1lat her child i s developmentally retarded ~vith characteristic "mongoloid" facial features; her pregnancy was rinrvcntful. Generali7erl hypotonia; flattcncd facc and low-set ears: macroglossia;flattcnrd r~asal bridge a n d e p i c a d d folds; silverwhitt- spols (In rhe periphrry of irises (BUL~SI I ~ E ~ .;POTS); . D dngle transverse palmas crease ( s r ~ r . \ u c : ~ ~ ~ ) :swidely i . split fixed S2 {dliu to an alria1 septic deCert).

HPI

PE

Labs

hr-yotype: 47,XX; bisomy 21.

Imaging

KUR: double huhble (rlilatcd stomach and proximal rlonden u m ) clue to duodenal atresia. XR, plain: hypoplastic middlc arid terminal pl3alang~s or firth digits (.AGKOMICRI,\).

Gross Pathology

Rrachycephalir tlrad; small brain with shallow sulci: hvpoplasia of frontal sinuscs; endstarclial cashic~rr derect.

Treatment

Snrgery for c u ~ ~ g c t a n lihean tlefects and ctuodet~al atresia: tr-ai n ing i n specialized groups.

Discussion

The most common chromosomal disorder, Dorm's syndrome is m o ~ I'requentlv t car~secl IJT trixomy 21 ( r l ~ ~ to c .no~idisjancrinn); it is lcss co~nnlonly caused by rnosdcirm or a Rohertsonian 1 randocatinn. 1 I i 5 associatrd wirh a higher incidence with advanced maternal age (i~ldic;~tiun for prenatal screening); a highcr incirlt.ilce of cardiac defects. eqpeciallv cndocardial cushion dcfccts: ancl a higher incidencc of acute Iymphocytic lenkemia ancl presenile dementia of Nfieirner's type.

pkq

DOWN'S SYNDROME

ID/CZ

A 7-yc.i1rulclbou i s brough to the optnnietrist fnr diminished visual acuity and requests a prescription Tor cycg4;rsscs.
T h e boy has an ~ ~ n u s r l I~ody al haI~it~is ~ i l long h arms and leg?:a Family hir;toryreveals similar body proportio~ls in o r her familv rncrnhcrs. I-Ic i s r+cFerrrdto his family doctor, who on carcfill c l ~ t e e f r i ing o discloses thi11 at1 uncle died nf a ruptured aortic

HPI

aneurysm,
PE

'Sa11: long extremities; a r m span greater than hciglrt (oor.rr 1 IUSTENCIS~ELL~X) : long, slender Fingers ( m c . 1 r v o n ~ c m ,;~ ) dislocation of lenses ( ~ r r m ~LEN r.~ TI^) ; sr\fert- rnyc~pia; inguinal 1ie1.ni;t; high-archrd palate; Aar feel (1,~:s IJL%NUS);aortic diastolic murmur (anriic insuficiency): funtlel chest due to pectus rxc;w;ltrim: scoliosis of ~horacic spine.

Labs

E klcreased urinarv h ~ m x y p m l i n e .
C?IR/CT/MR: marked dilatation or a5cenrlin.gaorta. XR. plain: ihoraciu and lurn bar kyphoscoliosis. Echo: r n i m valve prolapse.
C y d c medial necrosis of anrta may lead to dissection, rupttrre, aneurysm. or aortic insttfficiency; elastic lung fibers lortllous and thickened; crnph~sema formation.

Imaging

Micro Pathology

Treatment

Spinr bracing; uphthalmologic correction; endocartlitis prophyli~xis: P-adrenergic blockers;aortic va11-e replaretn~nt.
A svsternic conneclive tisnie disease characteri7~d hv an autosomal-dominant pattern aC inheritance, Mnrfan's !iyndi-nrnu i? dne to a defective chromosome 15 fibrillin gene,a glvcoprotrin secrcted by fibroblasts that acts as a scaffolding for the

Discussion

deposition
Atlas Link
F 1'
1 I
I

of rlastiri.

IM2-011

fi

MARFANfS SYNDROME

A 5-year-olcl white frnlale is h~-ought to !lor pediatrician hecause of h e r , marked weakness, pallor, bone pain. and hleetling from

her
HPI

nn5e ( T : P I S T . ~ I S ) .

She ha5 a history of progrcssir.ely irlc1,easing farigahiliq :and rearrent infections over the past few ~ ~ l o n t h s .
VS: fwer. PE: marker1 pallor; epistnxis; ecch ymrrtic patches over skin ; sternal tenderness; slight hrpa~osplet~umegaly with nontender lymphadenopathy no bigns of meningi lis; IIOI-ma1 Funduscopic exam.

PE

Labs

CRC/PHS: nnrmtrcytic, llormochrornic anemia; absolute lymphocytosis with excess blaqts {> 30%) and neutropenia; thrombocytopenia. Common acute Iymphoblastic lcukemia antigen (CALLA) (CD 10) positive;tcmlinal dec)?;y~ransferase (TDT) positive (marker of irnrnatlire T' anrl B lymphocvtes) on c n m c marker st~trlics; negai ~ V PInnnmpnt test fix Epstein-R~rr
virus.

Imaging

CYR: no lymphaden opatliy.


Neoplastic infilrr-atinnof lp~nph nodus. spleen, liver, and bone marrow with l o s s or normal mchitect~lre.

Gross Pathology

Micro Pathology

Myelaphthisic bone rnwrow (distorted archirect~tresecondary to spare-occupying Irsions) with l~mphobla~tir infilmation; Iynmphoblast~ with inconspicuouq nurlenli. condensed tllromatin, and scan 1 c v t o p l a ~ ~ .
Treat infection with a11 tihinlics, anemia ~ 5 t h hlood irat~sfi~sionr;, thrombocytopenia wit I1 pla~elet roncentrations. Remission

Treatment

induc~ion and ronsnlidation chemotherapy. Coiidder hone marrow transplant. Discussion

the most common pediatric neoplasm;i t accounw for 80% of all childhood Icnkcmias. It c;irries a good prognosis.
Acute Iyrnphocytic loukcrr~ia(ALL) i s

Atlas Links

Lq

ACUTE LYMPHOC,,,

L E U K E M I A (ALL,

IO/CC

A 25year+ld woman prcsunts with high-grade fever, menorrhagia, and marked weakr~esq.
Over he paqt seve~xl wccks, she has also hacl recurrent infections.

HPI

PE

Marked pallor; ~ ~ u l t i p I purpuric e patches over skin; hepatosplenomegalv; gingival hyperpIasia; sturnal renrlerness; nt3rmal fundr~scopic and t~clurologiccsarn. CBC/ I'M: normocytic, nomochl-omic anemia; thrombocytopenia; leukoc~~osis composcd riiairlly of myeloblasts and prornyelocytes (rrnnmnn~ring, early blast cells): neutropenia. Prolongecl F T ' and

Labs

m.
Gross %thology

Bone ei.nsion clue to marrow expansion;chloroma formation, ~nainlv in skull; spIenurnegaly.


bTy~lohlastr with myc~urnonocyl.ic rlirrerentiation replace normal mar row ( m r . ~ o ~ t i r HONF ~l~r M c AKKOW) ; basophilic cytoplasmic bodies (ALTER RODS) in mvelorytes; peroxidase-positivestains o r 1 bone marrow a n d gingival biopv,

Micro Pathology

Treatment

Chcmothe~apy: dl-trans retinoic acid in acute promylocytic Irukemia; bone marrow transplant during firfr remission if HM-maiched donor availahlr.
Acu tc mvelogeno~~s leukemia (AWL) ic no1 as common in children as i s ALL. A 1 increased risk is aswciatcd wit11 ioni7ing radiatinn, henzcnc exposure, Tlow~l's ~yllclromu.3 r d cytotnxic cliemotl~urap~lilic agents.

Discussion

Atlas Links

!TZEZ H-F2-014A, H-P2-014B, R-P2-014C, H-P2-014D

ACUTE MYELOGENOVS LEUKEMIA (AML)

ID/CC

A 12-year-old male presents with high fever; markcd pallor, and epistaxis; he has a history of recurrent UlUs ancl high-grade fever that have heen rreatrd with parentcral arlribiotics.
He has also sho~yn marked weakness over the past 3 months. He lives in thc vicinitv o f an industrial itnil that handles petroleum rIisti!lates such as benzene. VS: fever. PE: marked pallor of skin and con,juncriva:nral and nasal mucoaal petechiae; purpuric patches visible on skin: no significant Ivmphadenopathy; no hepatosplenamegaly.

HPI

PE

Labs

CRC/PBS: anemia, neutropenia, and thrombocytopenia ( P A K C ' ~ T ~ ~ P F . N anemia ~,~); with low rrticulocyte cotlnt; nnr~nal RRC morphoIogy, Normal seri trn bilirubin; negative Conmbs' test: normal chromosonial sturlies.

Gross Pathotogy Increased ycIlow rnarrow and decreased rr-ct marrow.


Mirro Pathology

Hypncellular hone marrow with empty spaces populated by fat cells, fibrous stroma, and scattered lymphocv~es; markcd decrease in all cell lines.
Retnoval of myclotoxin (in this case, benzene); hnne rnarroiv transplan ~rtion: immunosupprt~ssive acatmen t with antithyrnoqte globillin : niyeloid growth Fdc tors (e.g.,GM-CSF) fhr-

Treatment

neulropenia.

Discussion

Sixtv-five pel-cent of cascs are idiopathic. -4plastic ancmia following $rug or toxin exposure may be close dependent (e-g., hcnzcnt., cytotoxic drugs, radiation) or idiosyncratic (e.g..chloramphenicol) . Other causrs include viral infection anrl Fanconi's anemia, an autosomal-rrceqsivedisorder in DNA repair.

Atlas Link

r " l H-P2-015

*
+

ANEMIA-APLASTIC

-4 ti6-year-olcl white m a n recently diagnosed with chronic lymphocytic leukemia cornes inlo he errlergency room complaining or fatigue and ~achyca~dia.

HP I

He also states that his urine I-tas been progrt.ssivelv turning dark and red ovcr the course of the day.
VS: mchycarclia, PE: dyspnea; pallor ef skin and mucous membi.ar1~4; slight jaundice; splenomegdy.

PE

Labs

CBC/PBS: severe anemia; positive Coomhs' test; seticulocytosis; sphemcv~osis; "'hite cells." LA: lternnglobinuria. Tnci-eased serum indi I-ecrbilirubin. Congestive splmomegidly (due to extravascular hemolysis in Ihc spleen).
Prednisone; transfi~sions: sple~~ectoms; immuno~upl>r-es~ive Efrt~gs. Discontinue any offending drug. A4utoin~mune hemolvcic anemia is idiopathic in a b o ~ 1 ~ 5 0 of % cases; it is characteri7erl Ily autoanti bodjcs against RBC mcn~branes (Rh} , compIemen~ activation, and phagoqtosis of RBCs bv splenic macraphages. Three main types ex is^: warm antibody (80% tto I)O%: assnciarerl with lelikemia, lytnphorna. SLE, and \-iyjr;ll infrctions); cold reacting antibody (1 0%; associated with EBV/rnycoplasma infections and lymphoma) ; and dn~g-ind~aced (methyldopa, qllinidirle, penicillin).

Gross Pathology

Treatment

Discussion

Atlas Link

9 ANEMIA-AUTOIMMUNE

HEMOLYTIC

ID/CC

A 35-year-nld w m a n is admitted to the hospital with left4ded


weakness upon awakening.

HPI

Shc has no history of prior hcadachus. scizurrs, hvpcrterision. or cliabrtus and nuiehcr smrkcs nor lakes drugs. Her first three pregnancies wcrc spontaneously aborted; the foun-th reculted in unexpected fetal death.
VS: normal. PE-: patienr r o n ~ c i o r l mikt ~ : pallor; left herniplegia with exaggera~erl d e ~ tetl~lor~ p rellexes i u ~ d extensor plantar rcspunsc ( P O S I T ~ TBmm.sw;l'ssrm) : no nrck rigidity fundtls normal; n o carotid h r ~ ~nn i~ cardiac : tn~innnrs; reddish-blue rnottling o T skin in fishnet pattern ( z . m n o RI:~CL~L-\RIS)on cxtrc~ni ties; positive I Tomans' sign in left leg.

PE

Labs

CRC: rnilcl thrombocrytopenia. Prolonged nor-ma1 bleeding and clotting times; false-positive VDRL (titel- < I:18): FTA-ABS for syphilis negative: ELTSA ~horus presence of anticardiolipin antibody (ACA).

Imaging

CT, t~exrl( 2 4 hours later): hypodcrlrii~ ( d i ~ to e infarrt) internal capsuIc.

i11

right

Treatment

Anticoagulant therapy 74th heparin: uqe of lowdose aqpirin and heparin, either alone o r in cornhination wit11 prednisone, is atlvocated during pregnancy in case5 ~i th a complicated obstetric history (e.g., SpnntanPnlIC ahortioris 01- intrauterine clurnisc).
The prewnce nf lupus anticoagulant and ACA defines characterized by antiphospholipid syndrome; it is li~rther recurrent deep venous thrombosis in thu lower cxlreniities, ~hr.c~rnbosiq in t l I-endl ~ and hcpatic veins. pulmonary hypertension. cerebral artery ocdusion associatccl wit11 strokc and tranqient ischemic attacks (TI&), and neurologic tinding5 that resrmblr inulti-infarct dementia or epilepy-

Discussion

ANTIPHOSPHOLIPID ANTIBODY SYNDROME

A 9-year-old girl, thc daughtcr of African immigrants, presen w with a largc swelling of the left side of her face and jaw of 3 weeks' duration.
Trvn weeks i l p , she c o m p h i n e d of loosening of the upper ~ccr~n left d molar. Duspitc the sizc uf the tumor. there is no pain a~soriated with it.
Rillor; large, firm, ill-clrfinetl maw el~cnmpawing e n tire upper mandible, prodttcing mild ipsilateral exophthalmos wit11

deformation
Labs

(311

left side of lace.

CBC/PBS: nurmocytic. normochromic anemia; mild leukopenia; posi~iw direct Cnombs' t a t . Karyotype: chromasomal ~ l ? u ~ s l o c a ~ t(8;14) ion involving c-myc gene.

Imaging

( X U :nn evidence of mediastinal widrminl: (vs. Hr>dgkin's


Ivmphorna).

Gross Pathology

Firm, ill-cletined rumor involving upper mandible ancl defotmi ng neighhori~ig 5trl1chires,b u t no ulceration or uccl-osis; no satellite adenopathy.
Giemsa-~tained FNA show cells of uniform size with iloilgtanulalhasnpl~ilic nucIei and some vacuoles, 2 to 5 nucleoli, and evenly distributed chromatin surrounded by small, ~ h i n rccentric , cstoplasm that is pyroninnphilic; high mitotic index and typical "starry sky" imagc pattern ( d u e to cliff~~re disrrib~~lion of macrophages arnrmg tumor cells).

Micro Pathology

Treatment

14igt1ilose, short-term chemetherapy: rtlkalini7e ~~rirle, Cnl-ce rliuresi%; Ilnrle inarrow rransplantation; inrrathecaI metholrexare Tor meningeal prophy1;~~is.
Burkitt's lymphoma is a small n o n c l ~ a ~ e lymphonla d (non-Hodgkin's lymphoma). It is a pclorly dirferentiatcd &ell IynphohIastic l y n ~ p h o m aThe . endemic (hfiican) fonn is cha~~ctcrjze b~rjaxv d tumors and is assoriared wilh EBV infection; t h c noncndernic (M1estern) Term i5 characterized I>v atldominal anrl pelvic involvement. Thc condition was first drscribed by Denis & u ~ - k ii ~ n t1958 in Uganda.

Discussion

Atlas Link

*
A

BURKITT'S LYMPHOMA

A 65year-old male visits his ranlily doctor Cnr a rol~tine a11nual check~tp.
On rlirecterl hisiory, he admits to a weight loss ofahout 12 pounds over the past 4 mnn I hs. togrther with upisucles of epistaxis and extreme Fatiguc.

PE

Generalized ~ ~ o n t c n dIymphadenopathy: cr pallnr; enlargement of spleen and liver.

Labs

CBCJPBS markedy elevated WBC cormt ( 1 24,000); 90% lymphocytes; no lymphohlasts; mild rhrornh(~cytopenia; Coomb+positive hemolytic anemia; smudge cells (fra~;ite lymphocyt~s).

Imaging

Gross Pathology

Lymph node enlat-~ement i~lrno~t always pr-csmt; I~cpa~osplcnornepl,?~v with tumor r-lndule fol-mariotl.
Bone marrow biopqy reveals es~ensivc. infiltration, mainly by normnl-lnoking lymphr)cytes and a frw Fymphohlastq M-i th small, dark, round nuclei and scant rytoplaqnl; liver, s p l r e ~lymph ~, nnde involvetnenr common: I3 Iymphocvtcs fail to mature

Micro Pathology

~'"Pc'IIc
Treatment

Cf~errlotherapy; prcclnisone or spler~ecrotr~v for complic;-ltions such as autniininur~e l~crnoly~ic ancmia or immune thrombocytopenia.
Chronic lymphocytic le~~keinia (CLL) is a malignant neoplastir disease of R lymphocytes that exprers r h e s~lrfkce marker CD5 (usually in T lymphocytes); it iq cl~aracterizcd by slow progression of anc-mia. I~e~noIytic anrmia. recurrent infections, Iyrrlph node enlargeinen t. and bleeding episodrs.

Discussion

Atlas Link

C H R O N I C LYMPHOCYTIC L E U K E M I A (CLL)

A 40-vcar-old white male visits a doctor fur a life ixhsti!~ance pllvsical examina iioti.

The patient has no majnr complaint5 except for occasional fatigue (due 10 hyperm~taholic stale) and increasing abdominal girth (clue to enlarged spleen).
Pallor of skin and inucolis mernl~r;it~es: markedly enlarged spleen; pain on palpation over sternum (due to tna rt-ow nverexpansion); nr) Ivmpha~Eenopathy: 1 1 0 othcr abnormalities found. Labs CRC/FBS: markedly elevated WBC count ( I 30,000): itnmatore gr;inuioq~cs mixer1 u i ~ h normal-appearing ones: basophilia; eosinnphilia; early ~ I i r o m h o ~ ~ o late ~is t: hrnml~oc~~opcnia. Low lenkoqte alkaline phosphatase; elcva ted serum vi t anii TI B , , Icvcl. Lxvoqyt: chromosornnl tratlslocatinn t(9:22)/bcr-abl gene (PH~L\I~F,I,PFI~ 4 I:HKOVOSI)MI:). US. zlhdorne~i: splet~oinegaly.

Imaging
Gross Pathology

Skull chlorotnas ( ~ n a l i g n a t, ~greencolored l tnnlor arising from ~nvelnid t iqsue) ; enlarged and congested spleen with arcas of
throinhosi~ and microinfarcts; hrpatornegaly ( d u e t r> pro1iFeralion and infiltrai ion bv grannlove precursors and maturc granulocytes).

Micro Pathology

Hep;~~kc 4tlusoirIaZ Iruk~mic inlilrrateq: congestive splenomegalv 1vitl1 rnyeloicl metapbasia; PFliladelphia chromosome in all myelnid pl-ngeny.
Hydroxvurea; rw-interferon; leukapherr5is; bone marrow tran5plantatiun {the crnly potentiafIy curative trealtnent ) Tteanwn~ ineffective afrpr development of hlast crisis.
In chronic rnveloge~locis leukemia (CM?,), clearh us~~allv res~~lts frnrn ac.ceIet;l~ed tiansformation into acute leukemia (RUW cnrsrs) within 2 co 5 yean.

Treatment

Discussion

Atlas Links

C H R O N I C MYELOCENOUS LEUKEMIA (CML)

ID/CC

A 55-year-old male presents with swelling, pain, and redness of the right leg.

HPI:

He is retired and leads a sedentary lifestyTe. Hc admits to a 70-pack-yeas s m o k i ~ ~ history g and occasioi~al alcoIiol in take.
1 ' s : fever ( 3 8 . 4 " C : ) ; tachycardia (HR 106); mild hyperterhsic~n (BP 142/92); notrnal Rn. PE: right lower extremity swollen; pain eIicited on calf palpation and on dorsiflerbon of right foot (HO~L S' Y SICY) .

PE

Labs

Blood D-dimer elevated. US. Doppler: thrombi occluding right cornmon Femoral and popliteal veins. Venography: <goldstandard for diaqnosis, but rarely indicated.

Imaging

Treatment

Anticoagulation u.i LII TV heparin, Cnlln~ved l ~ long-term y an~icoagulation wit11 oral warfarin or subcutancuiis lowmol~cutar-weight heparin.
S~ircliow>triad (venouq staslis, vessel wall injury, and hypercoagulabIe state) contributes to the furmation of vrnous thrombi. C:omplications of DVT inclttdc ptllrnonarv embolism and venous ulceration. and insufficiency. Appl-nximately 200,000 deaths per year in [he Unitecl Statcs arc attrill~itahle to pul-

Discussion

*<

DEEP VENOUS THROMBOSIS

,I 25vea1--oId white fernale continues to bleed sreadilv after a nartnal, spontaneous vaginal delivery.
Mx~iual exploration ofthe uterus iox~ealssetairred placental iis.;lie that requires rlilatation anrl curettage; 50 minz~tcs after thr procedure, the patient hegin5 tn bleed prafwely from her gums and conritlues to b l e d vaginally.
Diffuse hlceding in gums and oral rnucosa; bleeding diathesis of skin (huth petcchiae and purpura) with oozing from

venipuncture sites.
Labs

Low fibrinogen. CIIC: low platelet count. Prolonged PT and activated PTT; elevated fibrin split prodtlcts, csprcially Ddimcrs.

Cross Pathology

May see ctxnplicaltions such as renal cortical necroqis, limb ~hrornhosis with pang-cne, and ischemic adrrnal necrosis.

Micro Pathology

Microthrombi in astl-tioles and capillaries, l e a d i t l ~ to microinfarcts in practically anv organ : also hemorrhages and pctcchiae in ini~olverl organs.
Treat underl$ng disorder; fresh fyo~en plasma: fihritlogen cl-~opretipitare: platel~rs; arninocaprnir acid with heparin.
r)issernina~ed ii~travasc~ila~coagulation (DIC) is a Illeeding disorrler that is clue to cons~~mptinn of plateletc, fibrin, and cr>aplatio~l, factors secondary to excessive clotting in niit:rocir~c~~ia I t~is i (precipitated ~~~. b y cancer, parn-negative septicemia, burns, mu ltiplr trauma, and abstetric complications.

Treatment

Discussion

DISSEMINATED INTRAVASCULAR COAGULATION (DIC)

ID/CC

: I 35-year-old man corn plains of pain in his calf rn~lscles while walking that is rclieved by rest (INI+.I<UII'I.I?N.I C I A I~UICATILIN) tc~gett~rr with exertional rIlest pain.

HPI

Ilcl~asafan~iEyhistoryofpremah~reatheroscleroticcoronary artery disease (CAD).

PE

15: mild hypertension. PE: obese; palrnar xanthomas and tendon xar~ tl~omas: orange-yellow discoloration of palmar
(pathognomonic for dysbetatipoproteinemia); tuhoeruptive xanthomas on pressure sites (elbows. buttocks, ;~nclknees) ; wuak peripheral pulses.
creases

Labs

LFTs normal ; lipid profile reveal5 elevated total cholesterol, triglycerides, and VLDL and reduced LDL and I-FX)L, chylomicrori remrlaills present i11 f'ds~i11g plasma; clec~t-ophor-esis reveals beta migrating VLDL:isoelecrric t'ocuqing shows ETI/ETI genotype (nearly pa~hognoitini~ic) . ?Ingio, coronary: arherr~sclcrotic coronary artery diwa.;e confi~-med.
Yellowish in tralutnin a1 atherosclerotic plaques sccn in tlze aorm and rx her h r g e V P S S ~ ~ F .
CIiaractcristic atherosclerotic plaques.
Weight reduction t o ideal body weight, regular exercise, avoidance of nlcohol and other trig1yccricle-raising drugs; low-fat, Irw-cl~oleskroll diet; in resis~nn 1 cases, gernfibrozil, high-dose nicotinic acid (niacin), and HMG-CoA redudaqe it1 hi hi tom (statin drugs) may he nsed.

Imaging

Gross Pathology

< , 0
z
r)

0 GI

Micro Pathollogy
Treatment

0 r

o G) <

Discussion

@sbetalipoprotein~'mia (TYPE rIr I ~ E R L I P O P R O T E ~ E ~ M is) clef ned as the prcsunce of VLDL particles that migrate to the hela position on electrophoresis (normal \%DL particles typiral ty migrate to rhe pre-hpta location}. Rela-WADI,particle5 are chylomicrons ancl VLDL remilants caused in part by a mutant apo E rhal impairs the Iiepatic llptake of apoproteitl-Ecnntaining lipoprnleins (Vl,f)l, a n d c hy/lotnicroilf).

7 DXSBETALIPOPROTEINEMIA

ID/CC

A 61-vrar-old white male presents with marker1 weakness, gingival bleeding, and an abdominal mass.

HPI

He has a history of'recurrent bacterial infections and has no1 u-aveleil nuuide thp United States.
Pallor; marked splenomegdy: mild hcplziurnegaly: no I p phadenopathy, icterus, or iiscites.
CRI: JPBS:anemia; decreased WBCs and platelets (~.~NCYTOPENI,\); lymphocytes with characteristic long, thin cytoplasmic prqjections ("I 3.41RY CELLS").

PE

Labs

Imaging

CXR:normal. CT/US. ahdompn: massive splenomegaly; mild hepatomegalv; n o Iymphadunopathv; no evidence of portal
hvpcrtension.

Gross Pathology

Liver; spleen. and hone tnarrnw infiltt-aterl Ily leukemic cells; ~pler~onlegaly nrav he significarlt. &one marrow largely repIaced by leukemic cells ( M +TIOPMTI-IISIC ROVE MARROW); large proportinn are hairv cells and contain tartrate-resistant acid phosphatasc (TRAP): splenic biopsy rrvtxls Ieukerrlic in fill ration of red pulp by hairy cells.
Dcoxvcoforrnycin and =-inturforon are highlv effective;

Micro Pathology

Treatment

splenectomy.
Discussion

Hairy cell lc-ukcmia is a chronic B-cell malignancv; atitoitnmune svnclrr)mes are frequently seen, inclirrling vasc~lliiis and arthritis. I t i f alqo characrerized hv atypical rnycobacterial infections.
DTTZ H-P2-024

Atlas Link

7.

H A I R Y CELL LEUKEMIA

ID/CC

An 8-yearald white male presents with an rr-ytheinatous skin rash over the buttocks and legs coupled with joint pains, abdominal pain. ancl hematuria.

HPI

Threc days before h e had complained of cough, coryta, low-grde fever, and sore ihroat. He has a history of allergy to dilsl and pollen.

PE

VS: hyprrtension. PE: palpable pwpuric skin lesions o.~.cr


buttocks and legs; painful restriction of knee and ankle join1 movement with swelling.

Labs

CBC: normal platelet count; normal coaplatioll tests. Increased ESR increased BUN and serum creatininc. UA. RECs and RBC c a s t s ;on urinarv serliment. Positive stool guaiac test (due to orcaI t blnnrl).
Nec~mtizing vascuIitis of k i d n e y and lungs.
Renal biopsy s l ~ o ~ Ibcal vs and seg~netlta! glomerulonephritis ~ vth i crescents (11lesangioproliCerative) ; mesangfd IgA deposits on immunofluoresclmcc.

Cross Pathology
Micro Pathology

x rn

OkAA

3
r o 0
G)

Treatment

St~ppotlive; s~eroicls; high-dose irn1n\rnnglcrhu9i 11 I herap7


U

Discussion

HennchSchrinleitl pul-prira ir: a generally selr-lin~ired. irliopathic disotcler that is also known as anaphylactoid or vascuIar ~ j ~ t r p u rit a ;i s a common vasculitis (s~nall vessel) in children.

G I

Atlas Links

FEPP PED-019A.PED-019B

HENOCH-SCHONLEIN PURPURA

ID/CC

. . 1Gyear-old male i s brought to a specialist bv his parents clue to persis~rnr pain and tenderness on the right side of his chest of a few months' duration.

HPI

Therr

i q no history o f tmurna to the affected area. The child is otherrvise well and is growing normallv.

PE

Exquisitely teilrlur si~ef o l ~ n d overlying ff~utth rib on right irk anterir~rly: remainder of exam ~mrumarkable. Ro1rrin.c la11 przrameters normal.
CkT: punched+ut lesion in foul-111rib otl Tight sicle.

Labs

Imaging
Gmss Pathology

Intrarncdullary expanding, eroding lesion. Brownish granulation tissue containing abundant foamy histiocytes ant1 eosinophils with leukoryte5 and giant cells.
Lcsions rcsolvc span taneouslv; surgical curetvage mav accelerate

Micro Pathology

Treatment

healing.
Discussion

F.ocinnphilic granuloma i q x rvpr of l a n g ~ r h a n ' cell ; histioqtosis; it is an inclolcnt disorcler that affccts cliilclren and young adults, espe~iiillvmales. Solitary Imne leqicltl.: may be asymptomatic or rnnv c a m p pain and tenderness and, in w m e instances, pathnlogic fixture. h ~ t twithout ailv systemic rnanfisla~ions. Diagnosis i s IxtserI on radiopaphic dernonsfrariot-i ol'a localized destr~~ctive lesinil arising Crom inside the marrow cavity. The skull, mandible, and spine are cornrnon locariotls. I n some cases tt~el-e may he spot-lmnenushealing nl- lihrosir, within a period of 1 to 2 veal-s. The disease mav xlqo he multifocal. involving the

HISTIOCYTOSIS X - E O S I N O P H I L I C

GRANULOMA

ID/CC

A 2-yea~rild boy is hrought in for a pediatric cons~~ltatirin because h i 5 parents are concerned ahout the child's protruding
eyes (EXCJPAI'HA I .VOS)

and excewive urine volume

I~OL~YURIA).

HPI

The parents also state that the child has hcrn febrile and has harl multiple rat- infections.
Low weight for age; bilateral exnphthnlmns: painful swellings
over head (due to cystic bony Iesions) ; no icterus; no

PE

lymphadenopathy; mild hepatosplenomcgaly.


Labs

CBC: normal blood counts. lncreaqed serum osmolality;


decreased urine osmolality.

Imaging
Micro Pathology

XR, skl~ll: multiple rounded lytic lesions.


Bone Iliopsy Ft-orn skull lesions sllow gr~nulnrnatour lesions and characteristic Langerhans cells with coffc-c-bcan-shapednucIcj and pale, ahundant cytoplasrti: tennis-racket-shaped tubular structures ( ~ R R I < C a K w r ILKS) nn elec trnn micl-o~copy; positive $1 OO protrin and CD 1 antigen.

Treatment

Cornhiua~iotl chemotherapy, ci ire1 lase of bony le\ic>nq.


A type of Langerhan's cell histiocytosis, I-Tancl-5chiiller-C:hristian syndrome is nlultifocal, producing diabetes insipid~~s d u e to thc involvement of [ h e hyporhala~nus and exophtl~;+lmc~s trorn orbital infiltration by histiocyies.

Discussion

h 2-yearald wl~ile male child is seen with complaints of fever fc~llrrwed kv a &ffuse skin m h .

The chiEd wxs apparently/ we tl a month ago, Imrn aftrr an uncumplicated pregnancy and delivery
17% t;lchl;c;~rctia;fever. PE: mild pallor; otc>scopyof left ear reveals cl~ill, poarlv n ~ o l > itympanic l~ membrane with pus behind i t ([I rrrrs ~ I I : I ) ~ A; ) generalized Ivmphaclenopathy: hepa tosplenomegalv; diffuse maculopapular c8czematoussash.

Labs

CRC: ane~nia: thrombocytopenia ~ vth i Ielrkopen ia (PIN(:YI.OPKNI.Z):t-rlative eosinophilia.

lmaging

CT. abdomen: he pato splen omeraly. X R cystic, rarefied lesions on skliZI and pelvis.
Skin shows prese~lcc~Textensive eczematoid rash; large destrtlctivr hone Icsiuns found on sku11 and pelvis.

Gross Pathology

Micro Pathology

Eosinophilic grantdomatous lesions in aIl involved organs;EM 5h o w cal Langerhans cells with characteristic Birbeck granules; t hcse cr llr: were further fbund to I>eH LA-DK-pa9itir.e and expreqsing CDI antigen.
Corticosteroids; chemotherapy; surgery or radiotI~erapy for lnralirerl h ( n n disease. ~

Treatment

Discussion

Lrttcrer-Siwc disrasc is an acutc o r subacute clinical syndrome of unknown ctiolop affucting children less that1 3 years old. It is marked by revet- drw to Incalized infection followed by a rliff~lse m;~r+ulopap~~lar ec/ernatous purplric skin rash and s~~hsequent l~epatosplenomegal anrl generalized lymphadenopa~hy, It shows sinlilari tics to acutc leukemia and r~thrr infectious processes, n i a h e t ~ s insipidlts, exophth;lIrnos, and hone lesions are ii~uallv wen in conrhinatiot~.

1"1

HISTIOCYTOSIS X-LETTERER-SIW

E DISEASE

ID/CC

,A 24year-old white male complaiiis of rapid enlargement of his abdomen, producing a dragging crnmuon, along wit11 a painless lrmp in his neck fnr lhe p ~ q 2 t months.

HPI

The palien1 also coinplains of iiltcrmittei~ t fever, drrnching night sweats. pruritus. ancl significant weight loss.
Paltor: unilateral nonterrder, rubbery, enlarged cervical lymph nodes; splenomegaly; no enlargement of tonsils.

PE

Labs

CRC/PRS: neritrop hilic leukocytosix ~ i t 1'1t1npliopenia; h normucyric anemia. Elevatccl ESR: elevn~ed serllm coppcr and ferri tin ; ncgativu Man toux e\t.

Imaging
GTOSS Pathology

CXR: bilateral hilar lymphadenopathy


Itlvolvud Ivlnph nodes arc ruhherv and have "crlt-potato9 appearance of cut surf'acr.

1 rn

Micro Pathology

I.!mph norlc biopsy shorrs large h i s ~ i o c y t ceIls ~ with multiInhed nr~cIci ancl eosinophilic 1111rleotus resembling owl's eyes (REF.~-S~T.RNRFH~; I .t. i 1: 5 ) ; nn honc marrow in~olvemen t on bone marron biopsv.

5
\

r 0

Treatment
Discussion

Radiotherapy and cl~etnotherapy.


Four patterns of Ilodgkin's discasc are seen on lymph nodc biopsy: lyrnphocytic predotnir~ar~ce 5% to 10%: nodular sclerosis 65% to 75% (seen 11-eqnentlv in voung {wmexl): mixed cellularity 20% to 305%; and Iympl~ocvte dcpleterl 10%.Prognosis worsens in this ordcr. Ann Arbor staging 1 1 1 ' with ~uhclassificatiunA {no cclrlsti tl~tional synlp tom%)a ~ i r13 l (weight loss, fcver, nigh1 sweatu) !nos1 acrul-ately predicts prognosis. The disease spreads to contigtrouq lymph nodes before heinatogcnous diaserrlir~a~iot~. O 1!TT R-P2-029

< a z r)
o b
I 3

<

Atlas Links

DI7E JMI-049

.a'<

HHOGKKIN'S LYMPHOMA

ID/CC

X 3-year-old white f ~ m a l c i~ hrougllt lo the ~rnel-grncy room with :I skin rash and severe epistaxis.

HPI

T h c patien1 Zlad a LRI c o i l r i ~ t i n g of a srvverc c o u ~ h and a riiilny norc 1 0 davs before the onset of her symptoms. S h e has no prir~t 11 irtory ot'prolonged bleeding follnrving mininlal trzltlma.

PE

Mucosal petechiae; episaaxiq; hemorrhagic budlae in buccal ~ I I U C I I S ~~ : plc~' n1 o1 nl~alpahlc.

Labs

CRC: mild anemia: low platelet count ( I I1.000); RBCs and W C s normal. P~.olnngt=cl I>leedingrime; n o r n ~ a PTT; l nt)rmitl PT.
Purpura (tluc- In r ~ t r a v a ~ ~ l iocbt r lllor>d ~ froin inu-ava~cuIar space in 1r1qkin) : piii-~i7erlhemol-1-h:lges (I~I?:;:~J.:CI I 1 , ~ ) : eccl~vmosis ( l a l - g ~rrhxn pui-p~lrx). N crrrn~~ hoilr l marrow aspir;t!e with increased number of m~gakaryocytes.
Prcclnisnnc; splrucctc-rmy;WIG
I t l i r )1>i11hi< I 11 r.ot~~l)orvtr,pPniT ~ ~ ~ I - P I (II'P) I ~ X is a n a u t o b ~ ~ n e rliw;~sr. 1 1 1 h>r.?n;ltiunof TgG antiplatelet antibodies anrl sul>5equcn1 pIatrIt.1 deqtr uclion in 11iespleen. 1r often folfows a viral inrection all<!i s srlf~limiterl in r.hilclren h ~ r chronic t i r l adults.

Gross Pathology

Micro Pathology

Treatment
Discu5sion

IO/CC

A 64-year-old blztck m;dr nnri casy Fatigability.

1;11ffttr-r frr)iii hone

pain,weig11 t luss,

HPI PE

I I c aIsc>co~nplains 11f reamrent WRrs and frequent nosebleeds.


Pallur: bone tenderness in Ir~wer back and rihr: petechiae on huccal mucora: n o hepatospl~nomegalv.

Labs

CRC JPBS: narrnocytic, normochrnic anemia: ~ ~ e u t r o p ia; en rouleau formation (KBCs adhering together like >tackof poker cliipq). Elevated serum calci~un; normal a1kaline pl~osphatase: mal.kedly increased ESR; gamma spike on serum protein electrophoresis (monoclonal gammopathy) . LTk RenceJones pmteinuria (clue to IgG light chains).
XR. plain: punched-out, Epic bone lesions in vertchrae, long bones, and skull (xuial skrlcton).

Imaging

GTOSS Pathology

Multifocal replaccrnu~it of norrr~al bone tiwue rvii h tumor cells (~dasmacyiorna): prlvi~, skull, and spine most affuctrcl. Infiltration of bonc n~arrcnt~ l~v 11osrnal-lookingplasma cells (al~undan t cytopIasm. eccr-ntric nuclei) in ~tggregates; amyloid deposits in kidney with re1131 ~ ~ ~ l - n cast ~la hrtnation r and intcrstitial fibrosis (can causc renal insufficiency);b o i ~ r erosion and dc.?itruction of curtical hime.

Micro Pathology

Treatment

Cliemoth~r-apeutir I-egimen; hvrlu~rion; treat hypcrcalccmia and Ilvperuriccrnia. Considcr palliative rdrli;-ltioirtherapy.
MultipIr mvrluma is a primary mdignancy of plasma cells ~i~ljth I-rplacemenz of nurmal bvne tnarrow; i t is [lie most cninmon primal-v brine carlrei: The l~rognosis worsens with anemia, renal failurc. and mr~ltiple lptic lrsior~s.

Discussion

Atlas Links

H-P2-0314H-P2-03 1B

"*1
I

'

MULTIPLE MYELOMA

,4 54-year-old white male complairw of easy fatigability, shortness of t>reatl~. headache, and ligh~l~eaderlness over l h e coursc of aftnost onc ycar, with increasing scvcriy.
Eie has also noticed a feeling of heavineqs in his ahdornen and increasing girth a? well as recurrenl deep p i n in the legs and occavic~r~allv in the uppcr abdomen.

Massive splenomeply: en larger1 liver; mudera~e amount of aqcitic fluid; multiple petechjae on thorzcu and extremities; no lymphadenopathy (one cliffrrential f r a tl~r-e ~l~are with d cllronic mvelog~r~rnis leukemia).
Labs

CBCJPBS: ancmia (Hb '7.2) ; low hematocrit; anemia; immaturr. 1.lrBC1;and r~nrmohlitsts src11siinril tat~eously (~.EL~I~~F,~Z~THR~ SMEAR); I~I.~S teardrop-shaped ~I~IC: FSiCs; giant
abnot.ma1 platclcts.

Imaging Gross Pathology

XK, plain: dcrise hones (generalized os~eosclerosis)


Extramedllllsuy hematopoiesis, whiclr is prorninen t in liver and ~pEern, with sig~lifica~i t increase in size and weight together will1 firm consistel~cy.

Mime Pathology

Wry tap" on bone marrow biopsy: l-i>pcellularhone Inarrow (h~-perc~llular early in diseasc) ; significant increase in number
of ~negakar-yocytes; replaccmcn t of mar row cissur will1 fibrosis (positive re~iculin on silver stair^): preservation of normal architt'c(111.e I I spleen. ~

Treatment
Discussion

Also callud ;ignogeiiic ~nveloid rnetitplasia. tnvelofibrosis wilh rnyeloirl metaplasia is an idioparhic canclitiori in which increasrcl S C L ~ L ion ' ~ o f platelet-clerivecl growh factor (PDGF) and TGF-13 catrw% replacement of hone marrow tissue with fibrosis.

Atlas Links

MYELOFIBROSIS W I T H M Y E L O I D METAPLASIA

Plilaternllv i n his IOJCC A 5Jyear-old whitc male notice5 painless I~tmps neck h a t haw slowlv enlarger1 nvcr the past 3 nionrli%.
HRI

AItho11g.h he clunics any pain, he ad~niw to havit~g; episodes of mild fever, night sweats. and some weight loss over this period.

PE
Labs

Bilatuml cervicill firm lymphadenopathy; pallor; ~pIenornttgaly,

CBC: C;oombsrposiuve hemolytic anemia; t l i i . o ! n b n c ~ ~ n p ~ ~ ~ i a . Elevated senm LDH (a uscfril ptopu.;! ic marker) : tlyp.pogammaglohulinrmia. CT/lJS: I\mphacienopathy: splcr~umc.galv. Lymph nocles havc grayish hue o n outside and "cut-potato" appe;lr<lxiceof cut sur-f'ac~.
LvmpIl ilode biopw dcmunstrates notl~xlar. (~t~ell-dirferentiatecl) or diffuse-9pu (poorly clifikr-entjared) ly mphocctic Iymphoma; histiocytiu and strrn cell lvm~>homn. AlkyTaril~g agents in varior~scombination^; r-xrliotl~~rapy if localizrd: bone marrow ~ransplantarion.
I
rn

Imaging

Gross Pathology

Micro Pathology

3
b

2
Treatment
G)

\ 0

Discussion

Primary maIignan t nroplasms of l~rnphocy~er: arise in IyrnpEloirl tiss11eanywhere in the hodv; thev occur mainly in lymph r~cldrs h m may involve inu-a-ahclominrrl organs and bone marrow. The prognosis is more dcpundcnt un g a d c tIlar~o n stage. Follicular (B-ccll) I ~ m p l ~ o m a are , s the 111r1stcoltlrnon form and are associated wi 1111 (14; 18) of 13cF-2 (an anti-apoptosis proruin). H T C T patients h a w a highcr incidence of 11011-Hodqkin'5 lylnphoina.

2 r) o
r

w I 3 <

r"C1

NON-HODGKIN'S LYMPHOMA

A 62-year-old~lrwisli male visits his fxrnilv doclor because o f


epistaxis, Eieadaclie, and dizziness.

The p a t i ~ n c hnd black. tarry stools

{VT:LI,.V.Z)

"months

ago and

M a s pi-rviuu.;lv admitted tu thr Ilr)spi la1 L'ni deep venous

thrombosis. Hc alcn drscri hes episndes or severe generalized itching ( I V ~ urr U crs) , primarily after showering.
PE

VS:hypertenqion ( I%P1 70/ 100). PE: ahese and plethoric; mild


cy~nc~sis: cngoraud, tortuous retinal w i n s with dark rerl hue on

( i i i ~ d ~ ~ ~palpable c ~ p y : spleen.
Labs

CBC: markedly increased RBC count, hemoglobin ievel, and hematocrit: C1.TRC.;;md plat elel.; ;114<1 increawd. Normal Pn?, Pc.o,. i n ~ d P-T: i~lc,r~aved viramin F5, IeveIs: increased leukocyte alkali~~e p hnsp hatafe; i ncrpawcl Feruln n n d urine uric aoicl Ir.r.t=lq:demaqed erythmpoietin level ( c l i s t i r ~ ~ i i ~ l l c s pulvcvthcmia vcrit from sccunciary p o l ~ ( ~ y h e ~ n i i ~ ) .
Increased blood volume and viscosity (RBC s l u d e g and t h l - o m l > i rol-mation ~~ n~ainly in licart alld h r a i ~ i )sttbr~orrr~al ; plrltplpt !'unction (bleeding tcnrlcncy) : increast,d frequrn? af peptic 111c~f.;1tir~ii.

Gross Pathology

Micro Pathology

Bone Iniwrow I)inpsv SIIOM'S increase in erythroid series precursors i11lr1, to a Irssei. exreti t, in ~rlegakaryocytes and WBC precursors: t h r o ~ ~ ~ h forrnacion us with microinfi~rcts ill brain and hrart; myelr~lihr~osi.; lnav t-nsue with cllaracteristic Iindings.
Phlebotomy: hylrnxvllrea;
treat

Treatment
Discussion

I~!-per~~riccrnia
ittl

increase i 11 FSC: mass \zrith incrrasecl bloocl vul~~rnc and r . i s c o s i ~i:t nlay he primill-y (pnlvcyiliemia vera) or q~cnnrS;u-y(due r a COPD, smoking. 011esi 0; eic.) . PC\' may pl-ng1-erqr o chi-onic n l v e l o g c i ~ o ~ ~ s Icu kcmia. myclofihrosis. or acutc myulogrnou~ luu kenria,

Polyc! 111~1ni;l i q rharilcirrijled hv

Atlas Link

POLYCYTHEMIA V E R A (PCV)

ID/CC

A 4vear-nld fernale i s t>i-o(~ghl t>y her mother tn the pediamic cl i nit afrer qlip (iti~ls blood and a "lump" i n the child's vagina.
The child's fathcr died of brain cancur. arid her rnother i s rccriw ng treamwnl for b r ~ a scancer. i Hrt- granrlfarhrr d i d of rneiastaiic col(lr.ectal cancer,
Pelvic w a r n reveals ulcerated, polypoid, grape-like mass arising from wall of vagina.

HPI

PE

Labs

Routinc Iah u-ark on urinc. I>luocl,ancl stool yields tlo pathologic findings. Bulkv himor mass with mu1tilohecl papillary prqjections resembling mass of q a p e s .

Gross Pathology

Micro Pathology

Rinpw of trltnor tnasl; s l ~ c > desrnin~ ~ s and myoglobin-positive (muscle ttlmor) , elnngated rhabdomyohlasts with largr. eosinophilic cytoplasm ancl cross-striations.
Sur.gical rrscc ti011 ~ i t :jc!jlivar~t h c hernother-ap): radin~hel-apy.
Sarcoma hotrynidcs is a polvpoidal: subtype uf embryonal rhabdomyosarcoma that characteristically protrudes like a mass of' graperi fi-om the cagina or kladrier.:i t i s [he mosl coin m o n sarcoma in children. 12habrlan~vosarcorl~as m e aften found in "cancer families'' (e.g.. Li-Fra~~rriei~i 5yndr.orne).

Treatment

2 3
r o

Discussion

< ---. 0
z
( 7

0 G 1

o m <

",

SARCOMA BOTRYOIIIES

A 10-year-old black child prewntq with a c l ~ r n n i c nonhel c e r otl hi5 lower leg.
He hac had recurrent episodes of abdominal and chest pain (due LC) micl-ovascttlar ocrl~ision) along with diminution of vision. His tnar ernal cousin s~lffct-s Ti-om a hloort disorder.
VS: fcvcr. PS: pallor: mild ictena: f~mtluscopy revealq hypoxic spots with neovascularization ( " S F ; \ F:ws'') ; non hcaling c hrorlic r~lccr o n left luwcr*leg.

Labs

CBC/PES: rlecreasecl hematocrit; megalohlastic anemia: sickleshaped N C s ; HoweM-Jolly bodies and Cabot ring; sickling o f R&Cs on sodimn metakis~ilfitt. peripheral film (Sickledex prep). Serwrn bilirubin rnoderalply eletraied:q1iantirath.e hemoglobin eleccrnpl~oresi~ shows 85% HbS. LIA: micsos~opic htmaturia. CT/ US, abdon~un: srndl, calcified spleen.
I.ocal ~ h e r a p v for leg ulcer: laser tlierapv for proliferative rctinopathy; anti biotic prophylaxis against capsulated bacresia; hydroqwrea may heIp irlcrcase lktal hemoglobin levelq.
Sickle celI anemia i s rai~serl by a point mutation on the gene totIing for the P chain uf l ~ c m r ~ ~ l u li~ ~show?; i n : autosomalrecessive inheritance. Gl~~mtnic acid is a r h s ~tuted i bv valine nt positinn ti. leading to chronic henlolytic ancmia. In the reduced form. FIhS Corills poljmers that damagc the RBC membrane. Fatturs that hastun sic kling i n ~ l ~ t d acirlosi.i r and hyoxemia. Prcni~taI diagnosis is ax.;rilal~le for at-risk fctl~scs.

Imaging
Treatment

Discussion

Atlas Links

fl

SICKLE CELL A N E M I A

IDJCC

An I I-month+ld malc presents with ntarked pallor, failure to thrive, and delayed developmental motor milestones.
TIIC child'^ parents are Indian immigrant<.

HPI

PE

Marked pallor; mild ictcrus; frontill bossing and maxillary hypertrophy ( ' ' ( : ~ I I P ~ I L ~F N.K ~ C I E S: ~splenomegdy. )
CBC: severc microcvtic, hypnchrcrmic anemia with anisopoikilocytosis; dccrpaqed 1-erirulocvtosis.HbA absent; HbF 95%; mildly increaserl unmnjupated bilirubin. XR, skull (lalural): maxillary cwrrgrowth and wider~ing of diploic spaces with "hair on end" appearance o f ff-nntal hone, causecl hv vertical trabeculac.
Expansion of hcmatvpoietic hone nlarrow, causing thillrling of cortic-dl bborle or new bone formation.

Labs

Imaging

Gross Pathology

Micro Pathology

R ~ r marrow l incrcascd; yeIlow 1nar1-owd e c r e a ~ ~ marked d: erytl~roid hyerplasia in marrow (ineffective erythropoiosis) Rlood kanqfusion, folic acid supplement, iron chelation therapy with desferriosamine to reverse hemosirlernsis, and bone marrow transplantation using HLi-matched sibling dnnc~rr. Beta-thalasscmia resulh from cl~cr-easecl svntl~esis of fl-globin chaitiu rltw to errors in the ixanscription,splicii~g or translation or mRNA. hlphx-thalasscmia results frorn decreased synthesis of a-glohin cIlaii~s clue to delrtior~ oi'onp or rnore or the four rw genes that are notmally present.
0 0

Treatment

Discus5ion

<

THALASSEMIA-BETA

A "Lyearold white female cliagnosed 2 years ago as HIV positive i s 1>1.1>11ght to the Plnrrgency mom hy her husband becausc of tachycardia, shortness of breath. hcadachr, intermittent disorientation,at-sd aphasia.

Shc had started propht;lacdc TMP-SMX 3 weeks ago. On thc previous day. she had finished her rnenqtrual period, which was ab~tndan anrl ~ had lasted [or 7 rtavs. Her hnrband also points our a generalized red rash all over her bodv.

PE

I T S :ti~chvcardia: fever. PF,: pale skin and mlrrous membranes; confusion ant1 apathy with lucid periods; petechiae on chest and exrr~lni t ips; pnri tive Bahinski's sign.

Labs

CBC/PBS: rnicroangiopathic hemolytic anemia {Hh 7.2) wit11 s~r king i retinhocytosk ai~cl fragmented RBCs (sr.~~sromms) ; low platelet munt (50,000): negative Coninhs' test. Elevated indirrct I~iliruhin(3.5). LJA: hrmatutia. Absent haptoglobin (due lo in~ravascc~lar. hemolysis); normal coagulation tests; elevated LDH.
Tllrnmh~ts formation i11 scveral o r p n s with platelet rlepletitln and rr~ici-oangiopathic hetnnlytic anemia: kiclney, brain, and heart most aflectrd by thrombosis. Multiplc h y l i n r thrombi iri brain, rnyr>cardil~m, renal cortex, arll.enals, and pancreas. Plasrnapheresis and fresh fro7en plasma e ~ r h a n g eprednisone; ; splenectomy.
AZsu known as Musc hcowil/'s yi~drorne. r hromhotic tllroml~ocytopellit purpum ( n P ) is an idiopathic disease Found in pregnant and W-positive pal ien ts anrl Ihllowing c~posurr to drug5 s i ~ as h antibiotics and estrogens.

Gross Pathology

Micro Pathology

Treatment

Discussion

THROMBOTIC THROMBOCYTOPENIC PURPURA

ID/tC

18-year-old hospitalized inale cornplains of fever, nausea, vomiting, and chest pain fuIlowir~g i\ I~lotld transfi~sion .
:\I]

WPI

He was invoIved in a rnc.>torrycle accident and 1 . ~ rushccl ~ 5 to the emcrgcncy room, where he received five units of blood before heir18 taken tn the O R for repair of a ruphircrl spleen and liver.
no hepatosplenorne~alv or lymphadenopathy; sut.gicxl lapai-otomy worirld unr-emarkable.

PE VS: fever. PS:


Labs

Positive Coombs' tcst (indicating a ~toan r til~odies to RBCs) ; decreased serum haptoglobin: elevated inrlircct bilircil~in; cola-cnIored urine (due to hen~oglr)hi t~uria).
Hyclratiorr: tbr-ce diuresis with martrlito! or r~~roscmide; hydrccorlisnne: alkalinizt. urine wit11 HCO,.

Treatment

Discussion

~ZCLIIPl l e r n ~ l ~tir ci ~ n $ h i s i ~ t.eaction )~~ may he the requlc of cemplete complemt=ntactivation: most c o ~ ~ ~ m o it n lis ya result of mismatched blood, producing intravasclar hemolysis. I t severe, rorlal shuidnwn or disseminated intrawscular coagiilarion (DICE

TRANSFU5ION REACTION-ACUTE

HEMOLYTIC

IO/CC

During the administration of a blood transfusion. a '$5-ymr-uld male I.)rcsorits zvi t l l fever, headache, and facial flushing.
An hnun- larer h p cI~vt=lnps frank rigors. HP lias resrived several transfusions in the p m , all of wh ich were unevcntf~~l. Thc last oiit W,IS a few weeks ago.
G'S: !'ever; RP nol-tnal: t a c l ~ y a r d i i i . PF,: ~nai-krrl ~>alloi.: facial flii.;hiii~; 1 1 0 c~anosis. icier-(1s. or. rrspiraturv dis~r-rc.; r~irlerzt.

HPI

PE

Labs

sel-11m hiliruhir~: no

CRC/PBS: negative direct and indirect Coonlbs' test. Normal incon~patibititr' fi~i~n oli d reprat czuss~nnlcliing ol'donor ~ r u m and patienl's hlood.
Suppc~rtirc; ;In tipvrrtics: Ie~~kocyte-depIete ruture transfusions hy Liltration.

Treatment

Discussion

Frhrile nor1hemolviic t~-;lndi~qion rmction ic ratrqerl hy preformed l e n k o a g u (qwttosirantil~orlicq)dt-i)elop~rl af~elP~PV~~PIS lri~tlufl~\ir)ils it : i s prinririlv : i type J J hypersensitivity reaction. Skin rash anrl pl-11r-itns or anapliylaui5 occur it1 rrllel.gic r~acrirrns media'ted by IgE (due to n type I hypersensitivity reaction).

TRANSFUSION REACTION-FEBRILE

NONHEMOLYTIC

ID/CC
HPI

A 12-vear-uTd while female is 11rc)l~gI11 trl thp emergency room k~ecausr of uncontrollable bleeding following a tooth extraction.
She has a history of prolonged bIeeding fc~llowirlg rr~iriii~~al Li+auma.He]- father also ha5 a bleeding disorder.
Mucosal pctcchinc; cpisraxis.

PE

Labs

Pmlongd bleeding time; nlorlerately prolonged PTT; quantitative xsay for factor VlII reduced; platclctc do not axkTcratc. with ri~toceti~l trst; low von Willcl~mi~d'r L;~ci(n(vMT) arltiget~ levels; lo~v \MIF acl hlty
Dcsmoprcssin. virallv ;~ttrn~~atccI v11T conccntrale (Tfl~maie-P); avoid aspirin. R c-ommon congellital disorder of hcmostasis, von IVillrhrai~d'~ discasc is also callcd ~ a s c u l a r hcmopl~ilia. Tympes I and J I are autosomd dominant; vWF factor is necessary fol- platclet

Treatment

Discussion

52
0 r 0

<
0

m
r-l

P a

C1

<

VON WILLEBRANDFS DISEASE

ID/CC

A 68-year-old ~vl~ite male visits his doctor complaining of weight loss, increa~ing fati*~e weakness. , headache, and visual disturbances over the past sevcl-al:monrhs.
I,lr also complain^ of eay bruiskg and bleeding gums while

HPI

I~rushing his teeth.


PE

Genrralized lpphadenopath~ engorgement of retinal veins with hemorrhages: r~loclerate tiepatosplenomegalp


CBC/PBS: anemia (Hb 7.3); RBC rouleau Formation. IgM paraprotein (n~or~oclonal spike on serum protcin
pier t mpliol-esis); increased

Labs

sel-tim visco<i~y. U A : normal.

Imaging
Micro Pathology

XR, plain: absence of lytic lesions (vq. multiple myeloma).


L!mph node 1,iopw may be labeled pleornoz-phicIvmphorna; Imne marrow and spleen typically infiltrated with plasma cell precnrsar5 (plasmacytic lvmphoq.te\); mav show cytnplxxmic cosinophilic. PM-posi tive inclusion bodies D L:T<:I rER RODIES) .

Treatment
Discussion

Fla.;mapheresis: chlol-a~nbucil; cyclophrlrphamid~.


Waldenstrbm's macroglohulinrmia is a m x l i p a n t Blympl-tt~cytc diqorder chal-acterizrd by excessive XgM (rrracroglohulin) production ancl hyperviscosity syndrome.

IDJCC

A 2-yea]-+Id male i% brought to liir pediatrician hecause of rccurrent epistaxis artrl chronic eczematous dermatitis.

HPI

H e has a history of recurring pneumonia and hilateral chronic s~ppi~rnrive otitis media. A male cousin sul'ress Crnm a similar illness.

PE Epistaxis; ccxmatous dermatitis ovcr 1~0th legs; several


purpuric patches over skin; mild splenomegaly and cervical Ivrnphadenoparhy,
Labs

CBC/PBS: thrombocytopenia:lvmphopenia. Decrexqed isohemagglutinins; decreased IgM: increased IgE, ilorrnal IgG. and increased IgA; inability ta form XgM antibody to carbohydrate antigens (i.e., capnilar- polysaccharides of bacteria).

Treatment
Discussion

Largely supportive; bone marrow transplant; splenectom):


L

Wiskoi t-Aldrich synrlrorn~ is a rare X-linked recessive diseaqe with B- and T-cell deficiency chal-actcrized Ily a triad of tharnboqtopenia, eczema, and recurrent pyogenic infections; it i q due to a delelion of ihe WASP gene in the p l l region of the X chrnrnosotne. The conditiot~ is associated wirh an increased incidence of lymphornaq.

3
0

<
m

' 3

0 Z

WISKOTT-ALDRICH SYNDROME

IDJCC

1 1 &y~ar-oldwhite female is brought to the etnergency mom hy her mother becalrse n i severe itching, joint pain, and a generalized skin eruption.
She had received an injection of penicillin 6 dayr3 before for stre~~ococcal tonsiEIiCi5, Hrr- mother denies any reIemnt past meclical history, including allergies. ORCC in the hospital, the child rlevelnpcd fever, edema or t h c ankles and knees, hematuria, and Ict1iat.p.
VS: li-vex-. PE: gcn erali zerl urticaria1 skin rash;axillar y arid inp~inal l~mpl~adcnnpatlly; splcnomegaly; rednrss a11cI swelling or knees ancl ankles.

HPI

RE

Labs

Jncrea~ed ESR: decreased GS, C4 levelq. UR: proteinuria; hcmaturia.

Gross Pathology
Micro Pathology

Gcnrralized wheals throughoul hnrly.

Vascular lesions show fibrinoid necrosiq and a ner~tropl~ilic infiltrate: immune complex deposition i n kidney andjoints.
Ai~tihistamines:cor~ico~tt~-r~ids: aspirin; cpinephrinc if srvel-e.

Treatment
Discussion

Scrurn s i c k n e ~ is s a type III hypersensitivity reaction (irnmurle cnrnpl~x rt isease) with a Iat~ncy period I~etween exposure lo the d e n d i n g agcnt { d r ~ l gserum) ~. and lhe appearance o f s i p s and s y p l o m s : it is usrlally sellllimiting.

SER'UM SICKNESS

A 50-year-old male presrnts with complaints of palpitations and chest pain.

Ute pair) increases with physical activity and is relicved by rest. He has multiple s e x d partners.
VS: high-volume, collapsing puke ( W ~ T E R - H A M M F RPULSE) ; wide pulse pressure. PE: p15toI shots Ileal-rl over braclliaJ artery: to-and-frv murmur hearcl over fernoral artery (Drr~nzrb'/'s hi^-RML~R) ; cardio~negalv: loud aortic component of S2; gradc IT1 early diastolic murmur heard rddiatir~g do\vn right sternal edge {rn~~rrnur of aortic inco~nprrence); mid-dias~olic mtlrrnul heard at apes ( , 4 u s n ~ Fr ,INI- M I~ R M U R ) .

Labs

ECG: left veniric~dar hypertrophy with strain pattern. VDRL and FTA-ABS positive.

Imaging

CXR: "tree bark" calcification of asce~iding aorta ail$ arch of aorta;mediasltinal widening and cardiomega?iy.Echo: aortic incompetence; left vcntsiculal- hvpertrophy and dilatation.
Gt-nss cardiac hypcrtrrlphy (cm- bovinum) ; aoriic aneurysm

Gross Pathology

in\,olving tlxe arch and thc ascending aorta and rxtenciing into the sol-I ic valve, rendering it incompetunt.
Micro Pathology

Obliterative endarteritis of viira vasortrrn; d~generation and til~rosisof' outer two-thirds of aortic media; comperlsatory irregular fihroits thickening or aortic i~ltima.
PenicilIin; surgical excisinn and rcpai~,
hortitis occurs in the tertiary stage o f syphilis, orten arising many rlrrarles d t e r thc primary inrection. U'eakening of the aortic wall cansrs clilalat i o n or the aortic root ac weIl as aortic incompctrr~ac arid aneurysms. Inti~rlalfiln-04s causcs nan-owins oi' the openings of the coronary arteries {ostial stenosi5), resulting in niyocardial ischemia.

Treatment

Discussion

Atlas Link

"

SYPHILIS-TERTIARY

(AORTITIS)

IDjCC

A terni funale n e ~ h o r n is notccl to have edema, d w n e a ,


cyanosis, and marked jaundice.

HPI

Her mother is blood type AB rCh-negative. I-lcr previous childbirth w a s an uneventf~ll full-term vagirlal delivery cnnrll~ct~rl nutsidp the United States 4 years ago. T h e mother did not receive any subsequent immunizations,
Pallor: marked jaundice; hypotonia; SS and S4; h epatosplenom ~g~ly; generalized edema. BEaocI 6-pc of mother Rh negative: bfootl tvpe of-faliler A Rh posirivu; blood type of first child A TUI positive. Mot her'q serum: positive indirect Cwombs' test, anti-D antihndv titer > P:64. Nenna te's serum: poqirive direct Cooinha' test. increaser1 indirecc bilirubin. Brain spccir~lun from au tops): reveals yellow ~taining of hasal ganglia by ~inconj~rgated bilirubin (IU:IINI(XEH~~S). Phototherapy (promotrs elimination of hilil-ubin); exchange transfusion. antibodies ow in^ to her exposure rn D antigen during hcr dcLirery of an Rh-positive i n b n t . In her sl~hseqzlen t pregnancy, t Iiese an tihodies crossrcF the plac~nta and reacted with the fehlq's RBCs (Rh positive), prodl~cing hemolysis and fetal heart failwe with g e n d i z e d edema ( H ~ K O P I+:TA~.Ts). S T c 1 pwxre;erlrR1k isoitnmun isatiotl, all MI-negative i~lothers with an MI-positive fetus shur~ld receive RhO (D) immune globulin following deliveries, abortions, ectopic pregnancies. o r even amr~iocentesis.

PE

Labs

Gross Pathology

Treatment

Discussion

The mother prodt~ced anti-ll (I$)

ERYTH ROB LASTOSIS FETALIS

sc 2-month-old male, IDJCC Paramedics arc c;~lleda1 7:00 a.m. h e c a ~ ~ a

thc child of Cuban in~mfgrants, canilot he awakened by his rnoil~er: upon arrival, it is clear that tlie child has heen dear1 ror at least 4 hours.
HPI

The child was siight3y prernatllre, but aside from this, his history was unremarkable. These was nothing that c o ~ ~directly ld
explain t h e episode. On directed history, the mother admits to being a smoker and remembers that the child had a URI 4 days ago.

PE

No pathologic causc rr-vealed that could explain death.


Autopy reveals pctechiae on pleural and poricardial surfacrs, pulmonary congen ion, and scattcrcd foci of Iymphoqtic tissue in interstitiurn nClungs.

Gross Pathology

Discussion

Sudden infant death syndrome (SlnS) refers to death of an infant under I year of age, usually during sleep. in which death remains unexplained wen after co1np1eteautnpsv; most have a history of mirlor UFUs.

SUDDEN INFANT DEATH SYNDROME (SIDS)

IDJCC A 1 7-vrar-olrl whire male undergoing c hernotherapy foldissprninated 1-lodgkin'sIvmphoma complains of serrrc hcarlacher, nausea, and weight loss.
HPI

TIIPpatien 1 had bren o n aminoglycosides, I l l i e n questioned, he is ~ i n c ~ r t a i of n place and time, h11t despite his c o n f ~ ~ s i o hc n
clescrihe5 llis l~ritie as appearing rerlclish+range over the past CPW weeks.

PE
Labs

Corlfi~wd but alert: underweight; nc) aci~te distress.

I,yt PS: it~creased p o t a s ~ i l l tUh: ~ ~ . oliguria: I~ematllria; mild pmteinul-ia: g1-annlar casts in tlrine; renal k~hular epithelial ccIls in scdirncnt; isotonic urine osmolality; elevated urinary sodium (> 4 0 tnErl/L). Increased serum inorga~licphosphor-us; azoternia with BL'N/creatinir~eratio of 3 (within t~nrrnallimitr).
Kidi~cvs enlarged, flahky. ancl pale
cclc~nii.

Gross Pathology
Micru Pathology

Necrmis of t111~1lar rpizhelial cells (hat 5lough into lutnen, forming casls aizrl causing hlockarle: hydropic ~ I e g ~ n ~ t a t i of on epirl~elium. Discontinue offending agent; fluid and electrolytr management.
Acll te t 11bular necr-osis i s defined as acutc tu bulacr darnage resulting in acute renal failure: it is causer1 bv prnlongerl iqcllemia or toxi~ls(ncphrotoxic i l r ~ ~ g iind s ) is usiially se~bersible.
a-'El PM-P2-048, PG-P2-048

f reatment
Discussion

Atlas Links

I k
-

ACUTE TUBULAR NECROSIS (ATN)


-

ID/CC

A 4 7-yar-old whitc male enters the emcrgcrlcy r oorn cnt~zplaining of :I sudden-nn~e~, severe headache that is the "worst headache of his life." He also descl-ihw ~low-onset dull p i n in his 1eTr flank and bloorl in hic urinc. F-IU was recer~tlvtr-eatect for recurrent UTIs. which were attril~utccftu an enlarqerl prosrate glar~rl. His father died of chronic renal failure, a n d his p;lternal grandfather rlicd of cerebral hemorrhap.
VS: h~pertension(BP 170, ' 110 ) . PF,: palpable. non tender abclomind mass n r I t ~ cth flanks; nuc ha1 rigidity
CA: alhuminuria; rr1icrc)rcnpir hemahu-ta ( n o MTRCs or casts). Sfigf~~lv increaser1 BUN, crcatinir~c..

SIP1

PE

Labs

Imaging

hngio. nruru: ruptu1,e.d herry aneurysm.CT/LlS, al~dntnen: multiple kidney and liver cyst%. Kidney tnarkecllv c ~ ~ l a r g e,c~ lnd hein.>: wi tli hunclrcds of cysts hat almovt replace nor.mal parenrhvma: cvsts thick-~t.allrd, ranging froin it f e tnillilnefers ~ to srveral cer~tirneterq in diameter. C:v?tic dilatation of tubt~les; epithelial cell hyperplasia: c ~ ~ b o i d a l epi~ helium lining cyst<.

Gross Pathology

Micro Pathology

Treatment

Dialy~is ancl rcurtl n-a~lsplaiitation. Ad it1 t polycvrr ir kidnes disease (,4PF;D) is ;In autosomal-dominant cliscasc causcd by :I dcfecl in chromosome 16 in which the renal pa~-c~~cf~ is y cor~~er.tetl m;t In hllnrlt-erlsof fl~~icl-filled cvsts, resulting ill p n s ~ e s i v u renal f~ill~re in adulthoocl. Cmtq ITl,lV ;duo involve the panclras. livcr. l ~ n ~ ganrl s . sp1ee1.1.It is associated with 1w-t-y anrurwms of the circle of M'iIlis, hypertension. and niit~aI yaIw

Discussion

; DpC"

ADULT POLYCYSTIC KIDNEY D I S E A S E (APKD)


---

female is brought ID/CC A 5 - v r a ~ n l d

to the pediatrician because her !nother tlnticecl blood i n her urine and diminished vision acuity.

HPI

Her family is Mormon. Her mother surfers frnm chronic renal fbilur-e.
1:s: BP normal. PE: appears well nourished: bilntcral sensorineural hearing loss; bilateral cataracts.

PE

Labs

CIIC;/PBS: nnrmoc hromic, normoqtic anemia. High-ton? ren~orineuralloss detected on audiomctry: eletpated serum cre;uiiiinc and RUN. UA: proteinuria; hematuria; RI3C casts.
Srnal't,s1tloot31kidney.

Gross Pathology

Micro Pathology

Lorigjtudinal thinning ancl splitting r ~glomer~~tar f hascrncnt mern brane, procli~cing cha~acteristic laminated appearance with glornvl-r~lar wlero~is; interstitial infiltrate containing fat-filled rnarrnphages ( 1 ~ ~ rF. ~ e A VC:EI.LS).

Treatment
Discussion

ACE i n l ~ i b i l o rrenal ~; transplantation


Alpart's ?mrlrnme can he autosomal-domi narlt car x-lit~kerl: and i s caused hy a defcct in tho or chain of tvpe n ? ' col2ayen. It is also

A 4,Feyear-olrlwhite female complains of palpilacinns and


shortilcss of breath, morning s\velling of the eyeq. a1 Ins, arlcl legs, and nt~mbiless of thc lower legs together with weight loss and Catigue.

Her past medical history is unremarkable.


Mi lcl cardiomegal:?: rnacroglnssia; pitting edema in lowcr extremities; ascites; cardiac arrhythmia on ausrulration.

Labs

LJ-k proteinl~ria. ECG: ~ ~ e n t r i c ~ t hvpertrophy lar ar~d Icw vnltagc (resttictive cadiomyopathy). Hymproteinemia; hyperlipidemia.

Imaging
Gross Pathology

CXR: biventricular cardiac enlargerr~ent,


Pathologic deposition of amyloicl glycnprotrin irl several organs, primarily heart, kidney, and rertal ancl gingit;ll tiqsue; kidneys palc. waxy, pa): and firm: spleen and liver ]nay be enlarged; clecp-brown discolnration characteristic of amyloid-infiltrated organs exposer1 to iodine.

Micro Pathology

Applegreen birefringence in polarized light when stained with Congo red; amyloirl depositinn in mesaugizim a5 well as in er~dotheliun~ surrounding hepat ir si t~i~soids and in spleen; Elvaline thickening of arteriolx watlq, leading LO narrowing o f lulnetl a n d ischemia.
Supportive.

Treatment Discussion

Primary amyloidosis cnmlnnnlv prescn ts ~vivjlhnep hroric synclrornc. hmyloidosiq mnv hc primary (in which the proteins arc monoclonal immunoglobulin light chain) or secondary LO ch t-rmic inflammatory stales (esperiallv rheumatoid ariht-itis a11d tubel-culc~sir). The primary typc is ufien associated with &cell clyqc~aqias, especially multiple myeloma, and in lhcse ca5es Bence Jones proteins are altnost alwavs present in the serum ancl mine.

Atlas Link

. A 5fiyear.-old male corn plains nl' urinary frequency anrl in~~rruptio of n thc urinary sb-rarrl owr tllch pa51 fi tt~nrllh~: lie wake up mt~ltipFe rime5 cluring the ~rlso compl;~ir~s of haling night 10 urinate ( ~ n ~ m : ~ i , \ ) .

Tlic patirnt's histnrv iilcluclcr onc episodc of acutc UI-iriary I - r t r n tion rmc n a o r l ~ hago that was i*rlie\,rtlrvit11 cathe1~1-iyation. Hr denim any historr nFhc.lr~at~~ri;t (w. ur~rcinorrsa of the hli~dder) or I~nck pain ( r s . rnctast;~si~rd pr.r~staiicr;rrcir>oma).He also atlmits to liavitlg a reduced caliber of urine stream ancl terminal dribhhg a? well as urinary hesitancy.

PE

Dipj;ltal rrctal csam rcr~rals smooth enlargement of the prostate pl-otr-m-lingi r ~ t o the wcri~til: ovetlving rectal muccjsa mol~ilc; bladder percussibfe trp to ~unbiIicus.

Labs

UA: 2+ bacteria; positivtz nitrite and leukocyte cqtcrasc. P~,nst;iie-hprcific anligen (PSA) lrvrlg nnrinal: ~ ~ r r ~ r l y n a n i i r sntrlie5 rlert-tonqtnt~ bladder neck obsimction with incrcasrd reqirlual urine volume; milrlly cIc~;ltcdSCI-urn crcatiilinc :tr~clRP:h'. P-S: hrnign-appraring enlargernrii t of' nirdian I o h ~ .

Imaging
Gross Pathology

En lar,ge-rtlprnszvle 1t.i1 1 wcll-de~n;~r.calerl ~ i~ctd\iles 1111 1 0 1 crii in diaiii~telin median lobe pl-mtate.
Both strrlma ancl glandu shc31c' hyperplasia on 11ir)psv: lih1-c)t1tvc~atlei10ttia1n~1~ perpla'iia 5ee11 i n which ptnIiCet-ating gla11ds arc s ~ ~ r r n n n d e by d prolifrrating ~ m o o t l i rn~lsclc cells and fihrt>l~laut~.

Micro Pathology

Treatment

Discussion

A~t:c.-rlcpr.nrlent changes o l ' cststjgr~lu and ;*ndr.ng~nr arc. Iwlie~lerl tr, catlsc hen igtl pl-nutatic tlvpcrtl-ophy ( BPI-I): :In increasing incidrnct. is notcd starting at 40 YtzillASof age. Tt ~~ffrcctq up t o 75% o f ~ n r n Ilv lllc a g r o f 8 0 vrars.

Atlas Link

BENIGN PROSTATIC HYPERTROPHY (BPH)

A 65-year-old white male cnmplnins o f painless hemahria OF several &T-s' rl~~t-alicl~~.

He i s a heavy smoker.
I.11ng~ clcar; abdurnen nnntencler; nt, palpable masses; g c n i ~ ~ l i a wi~llin rlortnal limits: n o Iy~~~pl~arlenopatl~y. Labs

CBC: slight nor,rr~ocytic, nnrrnucl~rolnicanemia. LTA: hemat~lria and a l x u ~ ~ l a ppit11eIi;d nt cclls.


IX'l-'/Cysrojiram: irregular filling defects above trignne.

Imaging
Gross Pathology

Pc'odular. cauliflower-like lcsior~ ~ vlh i cell tral necrosis ar~tl ~r~iiiirnal inva~inno f hladdcr wall. C;ytnIn~,rv of ltri1-1~ sl~c~ws rnal ignailt cclls. Biopg' nl' hladrler shc~ws p p d e I. siagt. R transitional cell c ~ o m ('1'C;C) a arising fi-orn arr)c.piihelil~rn and projecting inlo blatlder.

Micro Pathology

Treatment

Discussion

Thrrr i\a t hl-eeCr)lrlincl-casc iri r-i\li i l l men, nnd thc average age ai cliagnosis i< (ij. Risk fz~ctors f o papillary ~ rxrcirloma of the I>l;~clrler inclrtde incll~stsi-a1 expnsllw L n arylamines (especially 2-nap11t hr lalninr) , cigarette smoke, Sclr istosoma haerna tobitinl infPcti011 (although innst Srlrbturomn infections are associated wirh Fquamtnls ~iroptasis), analgesic abrse (especially phenaceti~t ) . ar~cllonglerrn cyclophosphamide I l~erapy. Complirat ions i ~ c l u c l e invasiori ot per.ivesicrtlar tisst~c. ureteral h~osis. invasion w-i t l i 111-inaryol,s~ruction(luxling Ijycll-nnej~ pvelnnephl-iti~, ancl rcn;~l failt~i-e) . anrl m r castases tr, tlle lung. hone. and li~~er. TGC apprars 10 IIP asrnciaterl with rnuta~ions ill rile p53 h ~ m o suppr-~fsor r gene anrl clclrtion\ in chromosomes

9 p . i ~ i ( F 9q. Atlas Link

-T

BLADDER CANCER

IO/CC
HPL

A 65-vual--ol(t rrlale prrsen ts with acute urinary retention.


For lhe pxqt few yal-s, lae has notril m increased frequency of mictut-itinn along w t h inct-ea~ecl hesitancy, urgency, dccseased forcc a n d stream uf ilrinc, >l.ulcl a furling of hcompTete evacuation of thc I>ladcIrr.FOFthe past fcw months h e has h ~ g t n t o rxprricncr increasing fatigability and laqstude.

PE

Pallnl-: hladder fill1 T ~ ahrlo~ninal I exan~ination; recta! exam reveals grade K D prostate enlargement.

Labs

CRC: rlr>tmocv~ic ;~r~cmia. L p s : hyncalcemia; h y l l e r p h n ~ ~ > h i ~ ~ u Elevated ~ i i a . BUN and creatinine. t!A: pmteirl~~ria; no KBCs or seen.
US, kidncys-s: biIateral hydroureter and hydmnephrosis.
In aclrli tion tn h~dl-otwphrosis and livrll-o~~rctcr. interstitial

Imaging
Micro Pathology

kidney di~eaqe is w e n on m icrnqcopic exxmination.


Treatment

Transrlrethral resection of' I he pr'ostiilr (TURP) r o 1.~1ieve the r>hslrl~ctioti is the hxsic atld most 11sefi11 SIPP.

'Discus~ion Ol~stt-wtivr nephrnpattlv rcsz~lts from t h r impaired orttfiow o f urine hut rrlay 31x0 produce chronic interstitial damage. Cll-~strurtive ncphropatltv is common in chilcI31ood (from congrni tal ahnorrn;~litir.q)and in irtdividuals older hat1 ti0 vearx, w h e n benign prosta~ic hypertrophy and prmtatic and gynrcoEogic cancers become Inore common.

7 BLADDER OUTLET OBSTRUCTION, NEPHROPATHY

ID/CC

-4 4%vean,ld white fefurnalc il; acl~nitrrcl to the hospitill l ~ e c a t ~ s e of worsening generalized edema a11cF rveakxlesl; along wir h hypertension.

HPZ

She has a l o n ~ h isr01.y or tvpe 1 diabetes mcllitus hut no history


01' Ilet~~atul-ia. rere11I sol-r tllmaf or skin i t-tiertinns.

PE

\IS: hypertenqion (BP 160/ 110). PE: gep;ene&ed pitting edema; n o cvirlcncc of pleul-al c f f ~ ~ s i o or n awites; I t u ~ g I,ase% clr;~ran auscultation; ,PT normal: rici ther kidnt.): palp;ihle: F I I I I C ~ ~ I F C C ) ~ ~ C exair) reveals prcsence ol' proliferative diahctic rctinopathy.
Elevated fasting blood st~gar (23.1 mg/dL); ele\b;i~cd glvcosvtared I~cmuglobin( I 0 % ): elevated BUN and serum creatinine; decreased serum albumin; elevated blood chfialesterol. UA: P I - ~ W ' T I of L-~ stigar and S+ protein: bvoa~1 C ~ S I ant1 S fatty casts; ~levnttd ql~xnti iative pl-otei11 (3.5 g m / P 4 hr).

Labs

Micro Pathology

lncrezed rnesangial matrix cln srn;tl biopw: thickening of capillary basement m e m h m e cnr~~hinerl with acellula~e( )ritiophilir n o r l l l l ~ in ~ i n e s a ~ i g i ~ (KMMI. ~m ISI 11. I . - W I L DIS~CLTE): ~~N hv,llinc arrcriosclcrosis of both affcrrnt and effrrunt arterioles: 110 i i i i ~ n u CCIIII~IPX ~~e depopi~q qew.
Rloorl sugar control; control of svstcmic hypci-tension. pl-efcriiblg tvi th ACE irlh il>itor; dielary protein ;~tirl phnsphare yes! tictioll; avoiclancc or ncphrotoxic drrigs; dialysis ur rcnal o-anspIantation.

Treatment

Wscussion

Dinhrtic glon~crulosclernuis is a t-cnitl n1a11ilesiatinn o f diahe~ ic ~nicrnanginl~atl~v atlrl presenf? at l e a ~ t 10 vears aftet-diahere~ appears (mol-ccommonlv in IDDM); it is ustiallv thc prcludt. to end-slage dial~rt ic renal rliseaue.

Atlas Link

mP M - ~ 2 - 0 5 5

D I A B E T I C NEPHROPATHY

ID/CC

11

S&year+ld ~ v h i t male r cornplains of a chrwnic c o t ~ g h of'slrl~eral mot1thc' dt~ration. ncc~~mpanicd by liyl~rl~carlotl~it.rs, f311 i g IP. and ninlnise: yrstpl-day lir co-righedup blood.

HPI

He ;11.;n <lcsc~-ilws in~errnittei~t I p w r AIICI tit'acIaches in arlcIitivn r o small vc3l11ni~s 01' dark orange mine. Ile rlcnic~ nlcohol t l s r hu~ arliiii~s 10I>eiilg a lieaw smoker.
L)il'fusr p111monn1-vcrncktrs hi1:ltcrallv.
.hot em ia. I I)$: r l l i q i z r i : ~ :hmaturia: pro teinuriz. Iron deficiency anemia: hloorl d ~ t c c t e c in i cputurn. tI3Gs: hvpoxcrriiit.

PE Labs

Imaging

CXR: l ~ i l i l ~ talvt-oi;~r - ~ ~ ~ ! i~ifilt~:~~es,


Incl.~asc ill iwigh~ ol' I~tng-5 wit11 areas of r~ecrosis: kidneys enlarged and pxlc with clecrcased crsnrirtcncy
Kiclt i ~ hioprv v st~ows pl-r>liI'el-;r tiire,
nWI'f1 t i ~ i n gcrescentic , gTgfomcruloncphritiswith a r r ~ u n ~ ~ l a tor i n ~ieutrophils u and rn:~crnphaqe~ irl Blxvrnan''; C ~ I ~ S Ichal-3ctcrisric I~C: linear T g G deposits in glomerdar hacement membrane and alveolar septa on i1nm1111oll~1nl-e~cc~1re; anti-glomerular 'basement membrane antibodies in ~ n - u r nnccrc~rizi~ig : hcrnorrhagic alveolitis on lt111g

Gross Pathotogy

Micro Pathology

i>i<I]>5yn

Treatment

I ' l a ~ n i aexchange: corticoctet-oids; immunos~~pprrssivt. ~Eier.spy.

Discussion

C:ontllx~st I Ire's ~vnrlrotr~e i~ heinorr3i:rgic al~erll i ti5 with ncphri ti\ and iron rleficienn, ar~emix c : ~ r ~ ~17v ccl anti-;~lon~crr~lar bi1st.111enl mcmhr;mc a11t i l ~ r its ~d ( R ~ C I1 ~ V P L ' I - I C ' I tivi I S ~ I? rt'ac1io11) .

GOODPASTURE'S SYNDROME

EDJCC A 45-vual-uldblack male prescnts with ~ ~ r ~ c o n t r n lhypertension led


and complains ol' severu occipi~al hearlache ancl ringing ill h i 4
ears.

HPI

He a l ~ o reports markedly diminished urine output over the past 24 hours. On direc trcl que~t ioi~ing, hc ;ilso reports some vis~ra'l blurring.
VS: severe hypertension. FE: flrndurcopv re~~eals presence of papiIIederna with hypertensive retinopathy.

PE

Labs

ViZ: protcinuria: microscopic hematllria: red cell cmtq. Elevated BUN and creatinine. CBC:: micronngiopathic l~enlnlptic ailcmii~. ECG: lefl-axis rlrviation with Ieft h m e rtticl~lall hyertrophy
Echo: ronccntric left vrn~ricl~lalhypertrophv tvi~11i.eciureri ejectinn fraction. LTS, abdomen: presence of parenchymal renal disease in norrnal+ized kidneys (unlike that of henign I I P ~ E I I-OSCI~I-osis, ~vlvheretllcr-c are I>ilatrt-a1 contracted kirlnevc).

Imaging

Micro %thology

PatIloIogic changrs includc fibrinoid necrosis of arterioles ( N I:< ROTl71VC 4RTERlr31 m s ) hyperplastic arteriolosclemsis ( " 0 ~ 1 0 SU;INNIN~;*), ~ and n e c r o l i z i ~ glotner~llitis ~~ associa~ed with a thrombotic rnici-crangiopa~hy.

Treatment

Reduction o f diastolic blood presstlre to at least 100 rnmHg; maintain urine output > 20 rnL/horw.
Sodirun nitmprusside i.i the cafest ancl most efcctive drug for usc in hypvrtcnsive errirrgencies: I3ecarl~c it docs not irl~pair mvuca~-dial bloclrl flnw, it iq especiallr: u s c f ~ ~ inl imrlrl-lying i~chenlic hexrr discasc. Howrvet; i t ir; inerrtl~olizrclto c y r i i c i t ' xnd thio~!a~lale; thrt-ef'ore. proloilgcd us(: rnnily lead LO r!anide toxicity or to thiocyanate tnxicitv. Blood tlli4 ~ cana y re Ievcls should be cteterminert frequentlv.

Discussion

ID/CC

,I 2-vear-old white male romplaills of rec~zrser~l epi~nrles of "bloody h e ' ' that laslril Tor several clays in conjunction with a
URI.

HPI PE
Labs

Hc was well lrt~tilthe onset of'wtnplnmq.


Pallor; slight palpebrnt cdci-na.

Zr;ZI: ~rrnteinllria; red cell casts in urine; gross henlaillria. Increased serum IgA.
Focal glotneri~lnnel~hritis involvirlg otjlv sclected glc3meruli wit11 mesandal proliferation and segn~eiltalnecrosis with crescents: immunofluorcscheucerypically reveals mcsarigial TgA deposits with w m c IKIW. TgG, and C3.

Micro Pathology

Treatment

S~ppol-t ive; IgA clep~sits r o r n r ~ l ~ l treappear ll~ fvllowi tlg kidney I ransplantation.
IgI:4 ncpkxopa~hy is idiopathic hut associated with rlppcr respil-atrlr y cji- G T infections lacking a laserlcy period (VF. postrtreprococcal glomcrulonrphritis). Lt.4orls are variable and may he mesangicprolifurdtive, focal ptnliSe~;itivu, or possil>lyct-ewendc glomerularlephritis. The glomerular pathology seer1 in B~rger's disease is similar lo that wen in Henoch-Schiinlein purpura. ~vliich is seen in children. It is scrn with illcreased f;cqlre~lc!,in patients with t r l i a c disease and liver disuase (due rIefectivc Ig14uImr.aiice). Chronic rend failure may ultimately develop.

Discussion

ID/CC

Pc 30-year-old black woman preqcnts with pain in both her knee


joints and in t l ~ e srn;1ll-jnint5oCthr h a n d logether wit11 milt1 fever; atwl-&a, weight loss, and 1099 or hair.

HPI

She alqn has a history of recurrent oral ~~lcemtions a13d a phoiosemihe skin rash.No,joint deformities art: reported.

PE VS: h y p e r ~ e ~ ~ s i o PE: i i . oral aphthous ulcers uoted; cry lhernatnus


photosensitive skin rash: "butterfly raqh" nver 1 1 1alar area of hce; pallor; n o ahdominaI o r renal I>ruitq hrarrl.
La bii

C;BC:: normocyt is, 1 1 0 r ~ t l o c h r o ~ nanemia. ic FA: ~nicroscopic hemau~ria zvi t h RaC casts in aclclitit,n to pl-oteiuuria. Elevated RUN and creatinine; antinuclear antibodies positive in high titel-: LE cell phenomenon posi~ive; anti-Sm antibody and anti& DNA anFibody positive; VDRL positive but FTA-JIBS negative.
Rcn;~!I ~ i o l ~ reveal5 w 1eatu1-csof diffuse proliferative glomerulonephricis. Klertmn microscop\ reve;lls immune complex deposit9 that are typicall! subendothelid nnd form "'wire loops.'"
Cni.ticost~r.oirls; cyrotoxic ill-11gs(cyrlopho~pharniclc, iuathiop r i n ~a , n d c11Ior;~nlbucil); Inng-term hcrnodialysis nl- transplant. Tl1ei.e are five pattcrns of I i~pu.; nephritis. Class I i s ~iorrnal Ily light. EM, and i~nmt~nofluoresccncc luicroscopy. Clam I1 pl-ewnt c; as rnesangial lupus gZomeruloncphritis and is foun rl in ahout 25% ofpatients; i t is ;isw~ciaterl with minimal Ile~natitria or prnteinuria. Class I11 is chamtesizccl hv focal proliferative glomc*rl~lo~~el~h an f 'd i~ is is assnciatccl wit11 r-eclu-renr l a c n ~ a h ~ r i a arlcl tnilrl r e n d i~ls~rfficiency. !.Claw I S ' is clcscril>eclin tllis rase and is hv far thu rrlosl common I'orrn. Class V pre5ent5 as membranous glomerulonephritis ancl i'i seen in 15% or cases; i t iliclt~ces severe protrin~rria (11.nephrntic qndrunrc=.

Micro Pathology

Treatment

Discusslion

LUPUS NEPHRITIS

IDJCC

AIT1 I-year-rllcltvlii~e girl is brought to the perliatrician hucausr of hc:tdaclic. cheri palpitalions, and ringing in llcr ears t o ~ e t h e r ~i2h generalized edema,
She 11x7 nn hiqtorv nF dvspnea, Fore throat, skin infec~ innlq, otlcvrr. C a r e f ~ que4tionin~ ~l reveals that shr has a h had hematuria.

HPI

PE

S'S: t~\.per~~ension ( HP 140/ 1 00). PE:: g e n e d i e d (including prriorl~ital) pitting ederna;JrP nnrn~;~l; lung hases clcar; neither.
liidnpy palpnhle; no P V ~ E ~ E ' I I Cor P pleu tal eIr115iorlor ascitcs.

Labs

Elrva~etl BUK and serum crei~~init~c: clecreased s~t-uni alh~~min: clcv;t tcd wl-iim ~righcerides; serum hypocomplementemia; anrirlt~clear atltihorly (ANA) negative; normal A S 0 titcrs. UA: fatty casts and oval bodies i n addition to proteins, Dii'fusc glornc.r~rl;lr involve~rlrnt with ~hickened capillary walls and FohuIar ~ n u . w ~ ~ gprcdiferatic~n iaI or) lighr rnictnscopv. SpIitting of has em en^ membrane causing railroad-back appearance with ! ' A S reagent or silver ~ t ; ~ i prominent n: granular immanofluorescence;mrs;tngi;-11 arid sohenrI(>t Eirlial tlepositr of inin~iine cornpleseh,
Cort icosteruids; renal transplan~a tiori .

Micro Pathology

Treatment
Discussion

Ven~hnnopr.oliferxtiv~ glr~tner~alonephritis (MPGK) is idiopathic hut mav he aqsnciated ~vitliiuflcrited dcficicncic.; of conlple~nent compo~lrn ts arld partial lipudvs~r.ophy, It is subdivided into two I Y ~ P S : npe I MPGN (hot11 classic a n d alternatir~c complerncr~t pathways activated) ancl type I1 MPGK (clcnsr deposit diseasr; a c t i ;I ~tion of nltrrn;~tecomplcmrn t p;lthwav). Apprn~iniatelv 50% of psticnrs w i ~ h 3ZPC;K will go on In rlev~lop chronic renal failure. Thrr-c is a high rrpcurrence rate Lollrwing rer~;~l ~rat~rplantation.

MEMBRANOPROLIFERATIVE GLOMERULONEPHRITIS (MPGN)

h 47-war-old black diahetic Ternale complains ofwcight luss, pl-ng~-es~ive shortnew o f 111-path, anrl swelling of the lower leg and arrn5.
Hrr past meclical Ilistor), is tiii~-en~ar-kal~Ie.
Pallor; pitting erlen~a in exmerniaics; clecrmsorl lung sounds with cracklcs hilatcriill~ in Ic~wcr lr~ng Iirlcls: periorbital edma; mites.
Labs

UA: proteinuria (> 3 . 5 g/24 hr); lipiduna with o'i~:~l fat hodie5 a n d Iatw and was? casts in lit-inary sediment. Hypoallbumhernia (< 3 g /dl,) ; hyperlipidemia (serum cholesterol 250 r n ~ J d L ) .
Iiidilevs r tllargecl, p a k , and be p ~ e s e r ~ t .
1-11bherv;

Gross Pathology

renal t~cin thronlbosiq

may

Micro Pathology

Thickened basement membrane; .;lthc.pitheli;~l depclri ts of IgG i ~ ~ CS l d along h a ~ c m c n t nlcrnhranr srcn i1-r "spike and dome" pattern un m c t h c ~ l a m i n e silver qrain; i m m u n e cleposits in a
"lumpv-bumpy" (disconrinurrr~s) pattern o n ~~nrnunofl~!crr~ercrtice.

Treatment

Corticr~stcrnlrls; yclophosphamiclc; renal transp1;;lntat inr1; ACE inhibirors redricu urinary prmteiri Ir,ss. h'cphrotic m~cIrumc may hc idic~p~thic or cauqetl Ijy m e n i b ~ a nous gl:lonrcr~ilr~i~rphriti.; (the m(w1 cnmmnn ca~tsc. in adults). rriinimal change cfiqeasr ( I . I I Y ) I I V ) I.E~I-IKO~ (the I \ ) most common in cl~ildrrr~), IbcA glomer~l toscler(3~is. or mcmhranoproliferalive glorner~~loiiephri~i~. Patiet~~.: wi~h nephtncic wndt-rrme have, hpercoaplabiity sccondnrv to locs of antithrombin I11 in t h e trine ( p . ~ .inrreaspd , incidence pcrip11e~-;ll vein thl.nmhnsir).

Discussion

"

MEMBRANOUS GLOMERULONEPHRITIS (MPCN)

ID/CC

i2 5year-olcl white rnale presents \v<tl? generaked edema and ahdnminal clistcntion , producing respiratorv embarrassment.

HPI

T h c child had a URZ 1 u ~ r k agn.

PE

VS: 13P normal. PE: generalized pitting edema; frce asdtic fluid in prrironeal ca~ity; shifiing dullncs~ and fluid thrill present; nortnal rund~~scopic exam.
L J k .I t. proteinuria ( > 3 g/24 1). Hypoalbuminemia; hypercholesterol~mia: hvpet-triglvccriclenlia: rlrcreasccl s e i - I I ~ I ic111ic calcium: normal CS levcls; normal serum crcatii~ine and BVN.
liid~lt>\~s slight2y enlai-zed, soft, ancl vcllowi5h.
I .igl~r micrascopv and immunofluorescen t shldies normal on renal biopsy ( n a ~\<rLcnce of immune complex deposition). EM rcrcals uniform and diffuse l n s s or the poclocytic foot processes.

Labs

Gross Pathology
Micro Pathology

Treatment

Corticosteroids; salr-restricted dict: dilirei icq: eIccnoIyte therapy and rnonitclr-ing.

Discussion

Xlsn caller1 Lipoid nephrosis, nlinimal change disease is tlze most common r m s c of idi oIjarf~ic nephrotic r;yndrome in children ancl is associate(1witla ini'cctiuns or vaccinations. It carries a good prognosis.

Atlas Links

fIFZI PM-P2-062

I IM2-036

A tj8-vear-old black male complains nf dysuriia, pr~~gressively increased u h a r y Frequency, and hack pain 11ia t has lasted several months.

IJe report5 high animal-fat intake.


Nodulal; r o c k - h d , irrep4ar area of indrrratinn irl peripheral lobe of prostate on d i ~ i t arectal l rxilm: midline fwrrow between prosta~ic lobes obscured; extension to serninal vesicles detected.

Labs

Markedly elevated prostatespecific antigen (PSA) and acid phosphatase.


Transrectal VS, prostate: hypoechoic masses in periplleral zone with extensioi~tn seminal vesicles. NIIC, brine scan: hot lesions of spine, sacrum, and pelvic bones (axial skeleton). C T / M R: prostare mass with capqular pener ration ancl ~tilarged seminal vesicIes.

Imaging

Gross Pathology Irregularly enlarged, firm, nodular prosla ie.


Micro Pathology Gore needle biopsy of prostate reveals single layer of malig-nant neoplastic cells arranged haphxardly in ;trlt.rloplastic struma.

Treatment

Proqtatectorny with radiation; or-chicctomy: leupralide: antiandrogens such a s flutamirle. A primary malignant neoplasn-i o f the prastare commonly arising from the peripheral zonc (70%).prostatc carcinoma is thc most common male cancer. Its prognoqis ancl rrr;3ttncnt depend heavilv on stage. Most case5 are diagnosed in asymptomatic mcn un digital rectal exam. Prostale cancer exh i hi ts hernatogenous dissemination. Inosr row mon lv to bone, forming osteohlastic lesions. Thr tumor can a l ~ o invade sacral nerve rOOF& causing significant pain.

Discussion

Atlas Links

ID/CC

X tifl-year-old white inale complains of r i g h ~ flank pain and hcmaturia.

MPI

H e Elas heen n heavy smoker for the past 24 years: he lost 5 pounds over the past month and i s net on a diet.

PE

VS: To.~v:r-ade fever; moderaw hypertrnsion. PE: pallor: pdpable mass in right flank.
Elcvatcd ESR. CRC/PBS: norrnoqtic. normochromic anemia. t'A: gross hemamria. n T J C T J U S :mass in upper pole of right kidney. MR: no invasion uf renal vcin o r infcrior vena cava (RTC).
Yellowi~h areas of necrolic [issue with fncal areas of hemorrhage within rcnal parenchyma.

Labs

Imaging

Gross Pathology

Micro Pathobogy

PoIy~onalclear cells ( c r l t ~ l atling i glycogen) with evidence of rytologic sepia invading renal parenchyma.
Right nephrectomy; cunsidcr renal-sparing partial ncphrcctomy.
The most common rend tumor, renal cell carcinoma is Crcqticntl~ speradic hut is, seen in association with von Hippelk d a u syndrome and djalysisrelated acquired p o l p t i c kidney disease. I 1 rreqr~enOy invades the renal vein and TVC arid mctastasi7es to lungs and hone ~ i Iletnatoget~ou~ a dissemination. Ic can also cause paraneoplastic syndromes (secondary to the pr-c3rI11rt ion r )f erythrnpoietin para thyroid-like hormone, ACTH, and renin).

Treatment

Discwssion

Atlas Link

PM-P2-064

h RENAL CELL CARCINOMA

ID/CC

A 63-!car-old

white male compIains of sudden-onset pain in the right flank lugether tvirll gross hernaturia, nausea. and vomiting.

HBf

He is overweight, has !>ernrlial~~tic T o r 15 years. is a hcar?? smoker and drinker, atid 1-tas Iwrn surgicallv treated for aortofernoral occluskve disease grid^).
VS: nn fever; mild hvpcrtension (BP 150/10O). PE: acute fislress: pilllor; swc-ating; st-vtre right flank p;~in; xanthelasma in hot11

PE

r-vclirls.

Labs

an zrrior

Normal RUN and creatininr. UA: hernaturia. FCC;: old silent ~'ilI~ l ~~yo~il~ i~ifarctiotl. -dial Elevated LDH.

Imaging

CT, a1,domr.n: wedgeshaped, nonenhancing lesion in right kidney, US. rei~al: edeinatous kidney with Focal region nf dpcreaserl color flow.

Gross Pathology

Pale. yellouish-white, wedge-sl~apecl area wit11 hernorrllagic


necrosis in renal cortex.

Micro Pathology

Cr,;~guI;ttion necrosis irlvolving renal cortical nephrons cxtcncling into cortitomed~~lla~junction.

Treatment

Remove arterial obstr~~crion IIV thromholvsis; Iwparin an tico~gulation ro prevent recurrence.

Discussion

Risk Fitcror~ Inr emholic events inclr~dc atherosclerosis and r n ~ ~ rthrombi al in the Iieart and aorta. infuctious enrtocarditiq ~egetatinnq. and atheromatous plaqiics in the atma. Crwnplic:\tion- fr-urn I-rrlalartery rmhnliqtn incll~rle renal failure, hvpel-rensiotr, acute pycIonephritis, and rcnal abscess.

RENAL INFARCTION

ID/CI:

, 4 30-year-old &it? physical vxam.

female

iq

fmind tn be hypertensive on rolltine

HPI

S h r claims lo have no history of hypertension 3 r d denies any changes in lifestyIe or excessive strcss.
VS: hypertension (13P 1 75/ 105). PE: loud S2; hlnrlt~rcopicexam normal; abdamind bruit present.
Elevated plasma renin: hvpokale~nia.

PE

Labs

Imaging

A n ~ i orcnal: , cnniirmator~; unilateral lcft renal artery stenusis i n a "string of pearls" pattern.

Gross Pathology

In fihromu5cular dysplasia. the rei-ial artery lumen is decreased dur. ~uhvpcrpfastic fibrotic wall thicken in^.
Mi~scular hygerplasi;~with fibrosis and segrnetltal s!enosis.

Micru Pathology

Treatment

ACE inhib'itors (contraindicated in hilateral renal artury stenosis). Ik~lloon a t~ginpla.itv; s z e n ~ n g~unjcal ; corrrction.

Discussi~n Renovascul;u- I~ypertcnsionis secondarv wsteinic hyp-pertension rai 1xe~1 11)' I~y~el-secret ion of r.enin ti-om Fiypnperfuwtl kidiwy(s). I t is mast ohen cauwd by fibromuscular dysplasia (young Caucasian women) or atherosclemsis (older men) ancl accountq for < .5%, cof all caucc-s of hpc-rrrnsic~n.
Atlas Link

T 1 PG-PZ-066

RENOVASCULAR HYPERTENSION

S&yeardd wliitc male prcscnts wit11 progressive painless enlargement of the left testicle of' 2 mnr-frhs' rltlration.

He also cumplains of a w i s e or hrsviness in his scr-oturn. He t l r i i i r s i i r l y histnl.~ nf p a i n or ir-auma a t the ~ i t e .


Walni~r-~ized. nontender. s111uot11. firm mass at upper end of left testicIe: mass does not bansilluminate: epididymis and vas clererens normal on palpation: prostate a n d srminal vesicle5 t~ol-inal oil digital rectal w a r n : xhdnlninal lvlnph nocles not pnlpahlu: no hepa~ornegza2v.

tabs

h'cjrrnal lcvels or hC:G; normal levels of serum a-fetoprotcin and LDH; histologic ( i i i t q n m i s hased o n postopurative specimen
stuclv.

Imaging

CSR: n o met;lst;~ris.US, ahdonlurl and pelvis/scrotum: 5olirl intratc5ricular m a w CT: no 1neti1st;tsi.s.

Grass PathoIogy

Micro Pathology

Shepts of germ ceIls ctin~aintng clear cyoplaxm wi rh lymphocytes in f i b r o l ~ s strclma.

Treatment

OrcIiicctomv ui th rctrc~prrilonralItmph nocle diswcrion; chcm orhcrapv ni th cispl:~tir~; rilciiot het-apv.


Seminnma i q tlre most cornmon Cpc tjf germ cell rumor. Dvsgpl-lninnmaq in nvarie~ are histnlngicnlly similai: T11rnni-s iivp extrcmc-IT radiosensitive. It is assr,ci;~ted rvi th ;I good pl-ngnosis. Cryptorchidism prcdi5poscs 'to the decelupmcn~ of lev ic~llar tltrnol-~.

Discussion

Atlas Links

ID/CC A 30-yrar-old

man cornplaiils of a small painless nodular swelling ever his right testicle that he noticrd a few mo11~hq ago, coupled wit11 increaqing growth of his breast tissue.

HPI

He also compIains of mild rhortncss of breath on exerrion (nmpu~x) cough, , and hlnorl-streakccl sptl~um.

PE

\rS: normal. PE: bilateral gyneco~nastia(breast tissue paIpahlc) ; srnaI1, pea-shaped swelling involr.ir~g the right testicle; testicular
sensation lost: no Iransillurninatiot~; left s~~praclavicular lymphadenopathy; h epa!nmeply.

Labs

CRC;: mild anemia. Serum 0-hCG elevated.


CXR: I I ~ O''cannonball" parenchynlal masses (due to metastases). T=T, aabclornrn: erilarged rernvptritonral lvrnph riodes and multiple hepatic nletastasw. US, scrotum: complex. solid r i ~ h resticular t mass.

Imaging

Gross Pathology

Small. p~a-shaped henrorthqic mass seeti in right testicle.


Polvgunal, co~nparatively r~nifclrln cytotrophoblastic cells with clear cytoptasm growing in s h ~ e and ~ s corrls. mixed with multin~~cleate syncytiotrophoblasticcells that have eoslnopMc vacuolated cytoplasm with readily d e r n ~ ~ bhCG; l e no well-

Micro Pathology

dcreIoped villi seen.

Treatment

Chemotherapy with cisplatin, etoposide, and blromycin in qome cornhin;~tion.Fnllowecl lrry radical inguinal orchiectomy ancl retroperitoneal lymph node dissection; gynecomastia rrgres3es once rhc source nf hCC; ( t h t~~ i ~ ~ ii~ o removecl. r) Choriocarcinoma is thr most malignant of all ~ ~ s t i c u l a turnurq: r i t metaqtasizcs relarively early via bolh the fymphatics and she bloodstream even w h e n i t remains very small locally. Follow up with P-h CG levels.

Discussion

TTESTICULAR C H O R I O C A R C I N O M A

I'O/Ct

A nelvhol-n hahy is uv;duatt.cl for ambiguous external pnitalia.


The baby was delivered v;iginally a1 fill1 term without any pw-. inrra-, or. posrnar~! complications; the mother did not take hormones or. any o~held r u g during p r e p n c y . Incompletely virilized external genitalia: Ii!~ospadia5;bilateral inpina1 swelling.

HPI

PE

Labs

Ki~i-vurttw: 46,XY. Miillerian structures absent: inguinal swelling4 prmed lo he maldescended dysgenetic testes.
US: absence of miillesian
trstes.
si rticrl tres

Imaging

and presence of dysgenic

Micro Pathotogy

Testes rh:1rilr1~l-i7erl hv seminiferous tubule degeneration and i~~vasion hy connuctivc tissr~carranged in wberlx.

Treatment

Conadectomy to protect againsr increaqecl risk n T testicular tumor; hormone replacement ~lier;~py given at puhertv.

Discussion

T ~ intitlcnce C u F gonadal tumors in d~genetic gonads rnav reach 1123 tn SOW, i n a k i n ~ ot-chirttr>myand s ~ ~ h w q u e n t hormn~le r s p l a r e r n ~ttthe ~ hest tl~erapeutic option.

Ir9 TESTICULAR PYSGENESIS

ID/CC

A ?hear-nld white male is seen I)? his Faillilr;physician because or dmnea. bilateral enlargement of the breasts (I:~~KC:OMASTL~), arid a painless lump in the right testis of approximalely 2 monchs'
di~ration.

HPI

H P denies anv history uf STDs, genital ulcers, drug use, or


Il'illltllit.

PE

Bilatcral nonterlder gyiiecornastia (duc iucreaserl hCG); left supraclalicular I y n p h a c l ~ n o p ~ t lk ~ ym ; hard mass palpablc on righi testis, distorted shape; no1.1nal rectal exam.

Labs

Markedly elevated blood hCG and a-fetoprotein ( A m ) . US/MR, 1e.ites: solid inmtesticular mass with some Coci of hemorrhage (iiltratessicr~lar masses usually ~r~aligrlan t).
Cvtotl-ophehlastic and syncytintrophohlastic ceIls with h C G dcmonstralhe within cytoplasm.

Imaging

Micro Pathology

Treatment

High I-adicalingttinal orchicctomyt foIlowerl by cisplatin-based con1billation chemotherapy.


Testirulitl-cancer rnav he pure or niixerl (mixed germ celI neoplasm) and is higl~ly ~~lalig~nant with early xnrt widespread metastaris. It i s thr most coinmon neoplasm in men aged 20 to 35.Wk sac ~~~~~~s ptnduce 011ly AFP, wlier~aq rhoriocarcirlotnas procluce oi1Iy hCG.

Discussion

"

TESTICVLAR TERATOMA (MIXED)

ID/CC

A 9-yeat~)lcl ldack male is hrougt~t in to the errlrrgrncy room hecause of sudden-onset severe pain that he expericncud in the lower at>domen and scrotum while p l a ~ i n g soccer. He has no relevant medit;ll history. r p o n admission, he llecame n;l~iseniecl ar~rl vomited three times.
Tsl-itakility;righl testicle tender, swollen, and e1cv;ttcd; palpable

HPI

PE

r~orrnal epididvmis antcriorlv; increased pain with elevation of mass (PRI. I IV'S SI(:N); 1 1 0 her-nia palpalde: 1 1 0 transfIlurnination nl tnass.

Labs

Uh: mild le~tkocytn~is.


in tel;zicle. Nirc-Tc9S: doughnut s i p (due to ccntr;ll lec~ic~tliirischemia ancl circl~mferential co1l;ltur;~lflow).
U S , scrcllrtiii: aqvinrri~trirrlecreaserl color flow

Imaging

Gross Pathotoqy

Tcsticlr rnarkcdlv cnl;t~-gcd will1 hrmtlrrf~agic necrosis; scrotlrm may bc p ~ i ~ p l i scol-rl h ; twisted.

Micro Pathology

Severe venous congestion : interstitial hemorrhage: hernorrl~agic necrosis.

Treatment

Immediate surgery (clttclrsion ;md fixation of tesris to scmtum) dnc to risk of iusiicle loss (less than 4 hours); contr-alatcral orc11iopc.x):propliylaci ically (high incirlenre or bilatcrality) ; atrophic testicle should hc rcmu~ccI due tr) puwihlr autnimmune rlestrucl ion of' con il-alaterxl testis.

Discussion

Tcrticular torsion is a s l ~ r ~ i c ernel-genet; al 1ha1 needs to hr clirk~-enti;~zed TI-oin orcl~iti~, epidirlymitis, and s t r a n p ~ l a t ~ d I-lertlia. I r i s seen more Frequentlv in an ondescended testide
(( ~wroirt:i-rinrs~).

TESTICULAR TORSION

ID/CC

A 45j-ear-old man with a high-grade nan-Hodgkin's lpmphoma develops olipiria, severe malaise, and fatigue :$ti hours foIlowing chemotherapy eeament.

PE Carpoperlal spasm prcsunt; n e i ~ h e r kidnev is palpable: urinary


hladdel- is empty.
Labs

Lytes: h?.purkalemia,hyperuricemia. ,ind hyperphosphatemia with scco~idxy hypocalcemia. BUN and matinine eIevated. IrA: acidic urine with numerous rhomboid crystals: no casts or cells
wen.

Treatment

Main tcnance of good bydriltivn. b~isk alkaline diuscsis. and pretreatment with allop~~rinol are keys to preventi13nof this syndrome; once acute renal failure has developed. fIuid and clecuoly~e balance must bc maintained m i d clialvsis may be necessal-): Tumui- lysis syndronle is tnost often seen in patierits wit t i lymphoma or leukemia hut is also scen in patients with a. mriety of solid tumors. The presencc uf ; i large turnor burden, a high growth fraction, an increased pretreatment LDH level and uric acid level, or preexisting renal insufficienq~ increases the likelihood that a. patient will develop tumor Iysis uyt~drotne. Incrrased Ic-vels o f uric acid, xanthinc, and p hosphatc may restdl in precipitation c ~ these f subsratlres in the kidney Renal sludging and acutc renal insufficienry or failrire Flirt1 1 ~ 1 aggravate . tlle

Discussion

F?a

URATE NEPHROPATHY

A 3yearald male is brought to his pediatrician for cvalualjon of an abdominal maw that his parents noticed.
HPI PE

The chiId has heen well ;ill his life.


Slight pallor: weight and hcigh; within normal range; nontcnder, large, fii-m, ; ~ n d smooth intl-a-ahdoininal mass to right of midline: sight cryptorchidism and aniridia. UA: ~nicroscopic hemaiwia; normal llrinarv vanillylmandelic acid (VM.4); BUN increased: serum crythwpoiedn clcvated.
hT:displacement and distortion af r i g h ~ pelvicaliceal sysrem.

Labs

Imaging

CT,abdomen: mmnr at-isirrg from right kidney with areas of


low densirv (due to necrosis) ; persistent ellipsoid arm ofenhancemen t (dile t o compressed renal par~nrhynla) ; no

widence of vasculal- i r ~ v a q i o ~ ~ .
Gross Pathotogy Whitish, solid rumor wit11 awaq nF hemorrhagic ~lucrosis distorting normal renaI parenchyna comprew~d inlo narrow rim; may. he involvement o f perirenal fat; metastasis 11sualIyto lunp.

Micro Pathology

Glomcruloid and t~thulnr structure5 enclosed within spindlr cell stroma; areas of carrifage. bone, or striated muscle tissue. of kidnev con raining tnmnr: chemotl~erapv Surgical rcmov~l with actinomvcin D and vincristine; radiorhelapv. Ncphroblastoma (WILMS' TL~STOR) is a malignat~c tumor of embryonal origin. 11 is associated with deletions on chromosome I lp involving the WT-1 gene and should he diffcrcntiated from ~ ~ e u r o h l a ~ t oa rn d a malignant Ivmphomn, which are other small cell turrlnrs of childhood. WAGR syndrome consists of Wilms' tumor, aniridia, genitor~rinary al~normali tics, and mental retardatinn.

Treatment

Discussion

WILMS' TUMOR

IQ/CC

A 4.5-vc.ar-old whitc remale is r r ~ s l ~ u to d the OR because of shock rlne 50 postoperative hlccding; during in tl~hatiorl, she vomit.; and aspirates thar day7shreilkfast,

H P I She had 11nde1-gone a cholcn~stectorny 2 d;~vsbefore atid had


presrn tcd with postoperative blecding requi~ing surgical explorr~tio~~.

PE

1 ' s : tachycardia: tachypnea; fever; hypotension. P E central cyannsis; warm, moist skin: intercostal retraction; inspiratory
crepitant rales heard over both lung fields.

Labs

CRC/T)BS: marked leukocytosis with neuh-ophilia; hagtnented TU3Cs: thrornbocytopenia. ABGs: severe hypoxemia with no improvement on 100% oxygen. Inrl-e.~qerl131TN and ct-eatinine; i ncr-eas~rl AST and ALT.

Imaging

CXR: typical diffuse and symmetric parahilar ("hat-wing" pattern) alveolar fitling process suggestive of noncardiogenic pulmonary edema.
Fomatioll of hyaline membranes wirh proteinaceous deposits in i~lveoli: pulmonary edema will] red, heal? lungs which, comhirter? 1vit11 widespread atelectasis, produce stiff lung with
fiht-045.

Gross Pathology

Micro Pathology

1711dr )t I-rrIial and ;~lr.eolocapillary clilmagr rvirh edema, hyaIi t ~ e


memhrane formation, and inflammatory infiltrate: loss of surfactant with fihsohlasr activity i n later 5tages.

Treatment

.Mecha~iicalven tila~ion, a n tihiotics, steroids, close monitoring of

I l e m o d ~ ~ a r nfi~nction. ic
Discussion

Adult respirarorv diqtl-ess synrtrome is a corldition that i q ;issociatecl with high mortality; i t is caused by gram-negative sepsis, mawive trauma;burns, dissemina~ed in travascu1;lr coaguIat inn ( DIC) , acute pancreat i ti?. r~arcot ic overdose, m d n e a r - r l r w n i t ~ g .I t i~ characterized 1 3 ccliCF~rw ~ alveolar capillary in,jurv. ~ I - t i c h Icads t o a n incrcasc in vascular pcx-mcitbilivand p~ltt~ot~a eld-e yt r ~ a .

Atlas Link

ADULT RESPIRATORY DISTRESS SYNDROME (ARDS)

ID/CC

A 65-year-eld male prrsents with pmgreasively increasing c011gh and dyspnea r)n exel-tion.
We i~ a retired construction worker anrl has a nca1.1~ L OO-pack-year smoking historv.
T C

HP I

auscu1t:rtt.d ~ PE VS: normal. PE: pxtle TI dubbing; lirte c ~ a c k l e hilateri~lly over lung bascs. Labs

z
CRC normal. PIT<:niixetl ohrtrllctivc anrl restrictive diseiise patt ~ r t l reT111ced : Dl+tl- M i ~ t - n s ~ ocxam p i ~ t ~ sputtlrr~ f re\+ealsgoldenbrown beaded rods (~szsn~srt,s onnres) composerl of asbcstor G~PI-F coated wit11 an iron-con raining protei~l;+cer>t~q rnaterinl.

Imaging

CXR: ir~reglilar. lirtear, interstitial infiltrates in 1owc.r lo1)es wi 111 cirr l t tnscri hed racliopaqiae cl~nsitirs( ~ ' i . ~ u ~PL~QLIES) ,it. . CT (high resolutia~l): posterior anrl Iatcral p l c ~ m thickened ~ with calcifiecl plaques sren bilaterally.
Difti~se pulrl~or~ary interstitial fibrosis with bilateral pleural calcification and ~ l ~ i c k ieng n and i~~volvcmcn I c ~ thu f rliaphraatn.

Grass Pathology

Micro Pathology

Calci~iinconraining dmsc plruraI opaciries ancl plaqltes of cnll;lge~i ; askesrc~q l~nrlies.

Treatment

Suppnl-tive anrl syrnptnt~~at ic treanncn t (oxycn. I~~-onchodilators, antihiocics):prevention of h t h e r exposlue: smoking cessation: counseling re~ardir~g high risk orbranchogenic carcinoma and malignant mesotheliorna.

Discussion

Prolonged rxposr1r.c to asbestos in ~igt~ificar~ tlv curnulatir~ close.; results in pulmonary parenchymal scarring. This process is seify pc-rpetuating, 1x11cessaiior) 01 eupuullr may slow rliseastl prcgre~sion . Cnmplica tion5 itlcl~idebranchogenic carcinoma, malignant rnesothelioma, cor pulmonale, arxl death.

ASBESTOSIS

ID/CC

A 10-year-nlcl girl is brought illto the ER in acute respiratory distress.


The patient is known to 11r allergic to cats and pollen: her rnotller stales that she had a recent URT. She also c-omplai~~s of a hil;tot-vof moderate intermittent dpspnea that is exacerbated by
exercise.

HPI

PE

VS: n o fever: hchypnea (KR 322);RP: nor~nal. PE: inspiratory and


expiratory wheezes (due to hronchoconstric t i m , small airway i nflammatini~) ;h n ~ g y a n d pale nawl mucosa; accessory m d e use during hreathi ng; enlarged ch esz AP diameter; hyperresonant to percu~sion.

Labs

.kRC;s: primarv respiratory al kalosiq (h per-uentilarion). CBC: eosinophilia (1 3 % ) .PFTs: low FEV, /FLrC.

Imaging

a hypesinflation with flattened d i a p h r a p s (incrcasecl residual vrdurne rlnr to air trapping);peribrontliial ct~ffit~g. C

GT~SS Pathology

Hyperinflation with air trapping in alveoli; plugs of inspissated mucus: erIenla of m ~ ~ c o slin a l itlg.
Inilammatory i ~ i i l t r z t e or bronchial epithelium. m;tir~ly cosinophilic: plugging of airways with thickened mum5 (CL~RSC:HM.~NN'S SPIRALS) ; I~ypertraphv of rnllcolls ~ ~ a n d s : ~longatert rhomboid crysmls derived from ~ o s i n n p l ~rytopIasin il (CI~PX(:D.I-LFM)FN I : R w ~ . ~ L: . hyperplasia s) oC smooth muscle of

Micro Pathalogy

bronchi.
Treatment

Tnl~aled, 01-al, and parenteral hronchndila~or~; steroirls: r l - o m o l y : ralirlltkast.


Bronchial asthma is characterized hy hyperreactivity of the airwa~ and ohstrr~ction due to hronchospas~n, edema, a n d mucus. I t is also knowm as reactive airway disease.

Discussion

ASTHMA

ID/CI:
HPI

A 5l).yea.-old white male de~~elops a fever 24 holm aFter surgery.


He underwent an emcrgenq lapamtomy for n perforated peptic ulcer ~ i t i l o any ~~t intraoperativc or immcdiatc postoperative complicationq.
VS: fever; BP normal; tachypnea; tachycardia. PE: no cyanr,sis; scattered rales and decreased breath sounds;n o calf tendcrncss; no liematoma or discharge Crnm woiuld: n o inflamm;~tionof l V linr veins: n o urinary svmptoms. ABGs: mild hypoxemia. GBC: sl iff11 t nea trophilic leukocvtosis. R h o d and spliturn culture sterile. ECG: sinus ~achycardia.
-0

PE

C
0
Z

5 70 X-

<

Labs

Imaging

CXR: dense opacity in right lower lobe (collapsrd lobe) with elevillion o f right hemidiaphragm (due to volume 10s.;).
Chest phyqiotherap); (incentive spiromrtry): dcep in~pirations; m~tcalvtic agents.

f reatment

Discussion

Poxtnperat i 1 . e atelectasis is i h e mnqr roinlnoti cause nf postaperative river- in the first 48 hours; alveolar collapse is produced hy occl~~sio due n to viscid secretions hvorcd by rccumhcr~q, hypovon tilalion, and oversedatior~. Other-causes of pnstopet-ative fever. l ~ s ~ ~ a seen l l v later in the postoperative period, include UTI, IV catheter infection. deep venous thrombosis, wotlnd infection, and clrug reac~ious.

ATELECTASIS-POSTOPERATIVE

A 1.1-ycal+-oldmale pi-esents with complait~w of exertional dyspnea, chronic productive cough, and occasional hemoppis. I-lc was d i a p o s c c l wit11 cystic fibrosis at agc 4 and !ins Iiad recurrent pulmonary infections r c q ~ ~ i r i n f rg eqr~cnt hospitalizatinns.
1 ' s : Ir>w%ratle fever. (38C); ~ a c h y c x r l i a(H R 1: 113): t a c h y n e a (RR 28). PE1: pallor and grade IJ clubbing nored: coarse crackles a ~ ~ s r ~ ~ l t river aterl 110th I11119 iielrl~.
Labs

CRC: norrnocytic, norrnochromic anemia: low hernatoctil. S p ~ ~ t trultrlre nl reveals S ~ ~ J I ) J ? / T I ~n Irr Ir TnIr,~ I .Pms: clecrtlascd F W ,JFC'C; ~uggestive of oh~truc tive pathnlov.

Imaging

XR. chest: incrcasecl l~ronchc)vascular m a r k i n ~ s honeycoml) : a p ~ m ~ r a ~(dur l c e I(>PIICI-(111 ~hadow'i of dilated hrr~rlchinles); loss of lung volrmle (atelectasis). CT (high resolutioi-i). chcst: dilated bronchi0Itz-s with "'signetring" appearance ( duc ti3 adjacent hranrh of puI~nonal-7 artery).
Lot~g, tubelike, irre\~ersihlyrlilated bronchides extending to the plcrlra with loss of l n n g parenchyma.

Gross Pathology

Treatment

Supportive measures; antibiotics; bmnchadilators, cxpeccorsnts, a11d physical therapy l o pro~nnt Iworlr ~ hial dl-ainage. Surgery may hr indicatccl lor localizccl or segmcn till hronchicctasis or when medical flel-apy hifs.
Dilatation of thc I~ronchialtrer Icads ro infections and to fnrillel- il-reversil>ledilamt ion. Under-Fving callses inclt~rle obstruction clue m tumor, foreign hodics, anrl r n ~ ~ c impaction; z~s congenital disorders such as k t a g e n e r ' s syndrome. 1~'iIliam~(hr~~pl>pll S Y I ~ ~ ~ O ~ ancl I I C ,cystic fibrosis; i ~ n d infections due to Ilm?~dr*~r~/lri / ) m / ? ti+, ~ tnpvit.1 IS.RW. n1ea4 es. and R'lymt)rzr/priwm 111I~~~trlnr Complications i.~. iticludc lung abscesses, metastatic brain abscesses, arnyloidosis, and cor pulmonale.

Discussion

BRONCHIECTASIS

ID/CC

X 60-year-old male is rukrrud to arl i~llrrgist for late-onset asthma rhai has been unresponsive to bronchodilators and antibiotics.

HPI

tlc 1x1s also been hming chcst pain ( n ~ c i ~ n f i)l i , g ~ wanorexia. . and pain in both calves ( c : r , \ r ~ n r r m c ~ (111 ~ )exer-lion that are of r c c c n t onsrt.
VS: ~ a c l i y i ~ emild a ; fever; mild hypertension (I3P 15n/ 100) (secourlal-y to renal vascular involrcment). PE: mai-kcd respiratory diqrrew; rvidespread wheezes hilatcrallv; numt.sc>lis purpuric lesions on feet (rI~rr to clrtancotis small vessel x~wulitis).

PE

Labs

CRC:: mild at~enlia: leltkocyto5is (> 10,00O/pL); Hct < 35%: r hrnml~ocvtnsi~ ( > ?P00.000/pL); eosinophilia (> 1000JpL). Elevated BUN and creatinine: P-kVCA positive. LTA: proteinwia:
prewnct of RRCs, SITRCs. and granular casts. PWs: FEV,/WC. T. hll1os ra~ict reduced (obstructivepulmonary disease). EGC. tachycardia.
-'

Ernaging

CXR: bilateral upper and fewer lobe infiltrates i ~ t ~ noncavimtd ina nnclirles.
t,unq ~ h n w s he~nol-rhapyic infarcts secondary to thrombi in afecwrl artel-ieq.
Tran~bmnchial lung biopsy shows ganulomatol~s; lesions in vascular and extravsrsclrlar sites accompanied by intense eosinophilia: skiit biopsy nf ~ > l ~ rt-ic p ~lesions 't shnws vasculitic lesion-fil~rinoid necrt,sis of mcdia ~ i mixttiru ~ h of irlflamtnalory cells extending along arlvetltitia; octxcional :Inenr~smsand seranrlary thmnihnses wen; tltc artcrinl in tcrnal clastic lamina i~ clestrovrrd and intima and mcclia arc thickened.

Gross Pathology

Micro Pathology

Treatment

Pcednisolone effective in i n r l ~ l c 1 1 i g remiwion: cyrlophospharnide u ~ e rin l those refractory to srcroids; monitor disuaso c o t w e l~sirlg ESR leveLs.
C;h~~rg-Strxuss svndrolne is an idiopathic systemic small- and medium-vessel p u l o m a t o u s vasculitis (grouped ~ vth i pnlval-teritis nnrlaca [PAN].ivflic1l cloc?i not i~~vr)lrre Iizngsl thal is cliararterizrrl hp a triad of Fate-onset asthma. a fluctiii~ting eosinophiIia, and an extrapulmonary vaqc~ilitis.

Discussion

CHURG-STRAUSS SYNDROME

ID/CC

A 5(T-year-oldwhite male smoker presents with productive cough, copious sputum, shortness of breath, and fever.

HPI

The parient has a 40-pack-yearsmoking hiqtory. Hc has also .experienced chrnnic dyspnea on exertion; chronic productive cough. usually in the mornings. for several years; and niultiple colds each winter.
VS: fevcr. PE: stocky build with plethora; whee7es CBC: elevated IVBC cout~l(14,000); neutrophits predominant; secondary polycythemia. Str+tnrocrix~ p n m moniw o r Hnmophib~a i?lflurmwo n Gram stain of sputum sample. Al3Gs: decreased Po?; ~ l e m f e cPcn,. l PlTs: decreased vital capacity; decreased mv,.

PE
Labs

Imaging

CXR: increased bmnc11ovascuIar markings in lower lung fields.

Gross Pathology
Micro Pathology

Thick

mucous secretion:

cdema of hmnchial mrlcosa.

Incceawd size and number of mucous gFands (Reid's index > 50); inflammation: fibrosis; sqlramous metaplasia.

7T ' *

COPD-CHRONIC

BRONCHITIS
.

A 55-)~eawld ninle complains or progl-essivt=l~, incl-c-asing shortness o f breath on exertion for t h e past few ~nonths.
Hr x l w ~ romplains o f a nonpl-oductivc mild cough ;111d11as a 40-pack-year smoking history hut 113s no hiqtory or hemoprysis or uccupa tiunal esposm-e lo inorganic or rrl-ganic dusts.

1 5 : mocIcrate rachvpnea. PE: moclel-ate respiratorv distress; using accessory muscles of respiration; full ncss of neck veins during expiration; chest barrel-shaped pemzssinn nole hyperresonant; cardiac and Iiver duIlness are obliterated; scattcrccl rhonchi 1ljlater;llly;heart sound* heard distant hut normal.
Labs XBGs: mild llr~oxia with respiratorv ;~lkalosiu. PFTr;: ir-tcreaserl r c ~ i d u a~ l ~ o l u ndecreased ~e; FEV, /FVCratio (oncr 1r1 11.1.:IYISFA~E PATTI:RN) ; decreased DL.,,.

Imaging

CXR (PA view) : hyperlucent lung fields with a few b~~llae: flattening of diaphragm ancl elongaied tlthular I~eat-t shadn~v.
Air space5 dilared; tapper lobes most affected.

Gross Pathology

Micro Pathology

Pattern of centrilohular emphysema: alveolar sepra arc visiblv clilrlinishrd in number along with increased air spaccs.
Ccssatiun of smoking, brc)t3chntlilatois. s~ptmids it1 resistant caws, ar~tibinticq cturing acute esacrrhations, and h a m r u x y g c ~
t herapv.

Treatment

Discussion

Ernpkyselna is defined as ahi~ormal pel-n~anent enlargement of t h r air spaces distal to the turmii~al hmnchiolr arcnmpanierl tllr desfruction o f the alveolar wallf; ~mphvsema n~ay inyolye r he acinus arid the lohu te unifnrnilv in a pattern cal1c.d parmcinar, or i t m w primariIv iilvolve t h e respiratorv hronchir~les, termed cell triacin;~r, Pi~naci tlar enlphvqerna iq C O I I I I ~ O Iin ~ patie 11 ts with a,-antitrypsin deficiency. Centsiacinar cmphyiiema is con111ionlv fout~rl in rigzarette smokers and is rare in nonsrr~nkers:i~ is ~15ualEy mnrc rxtcnsivr and qcvcre in tlie upper lohes.

Atlas Links

COPD-EMPHYSEMA

I DJCC

A 37-year-old male in t h c ICU devrlops petechiae, altered sensorium, and marked dyspnea that pl-nve rprracta1.y to oxygen thcrnpy.
Twenty-Fourhours a5o311u %$as aclnlittrd ro the hoqpital with fractures of the shafts of both femurs, the pelvis, and the ri@t humerus, rl~stained following a fall from a 20-fool-high stepladder.

HPI

PE

VS: rt=~~t.r; marked <dv.ipnea. PF,: delirium; tend cyanasis; u?ing accessor-v ~ ~ ~ u s r 01' l erespiration; s wheezing heard over both Iirtlg
Cielrls.

Labs

ABGs: profound arterial hypoxemia with hypercapnia. CRC:/PRS: i1ii.ntn bocyrop~nia. Fat demonstrated in urine and sputum: normal PT a n d PTT.

Imaging

CXR: earlv, normal; later, bilateral perihilar ("R.%T-M'IKc;'*) appra~'ance nf pulmonary infiltrates withnut carcliomugaly (due tn r~oncardinge~iic pulmonary edema). r ; R .plain: Ion%l3one rr:ici~~res.
Obstruction of pulrnonar y vessels by far glnh~r!eq; chemical p~~rurnon iq. il
Intrrmittrnt positive pressure vcr~tilation wit11 190%nxxygen, supportivr managurncnt.

Micro Pathology

Treatment

Discussion

Fat cmbolizatinn usually occurs 24 to 72 hours after fractures of the shafts of the long bones.
PM-P2-082

Atlas Link

FAT EMBOLISM

IDJCC

2 1 .in-year-nlcl farmer p r t ~ ~ e nwith t s sei7vrVshortness o f breath


( I ) W P K F , ~a11d ) fatigue.

HPI

Hr also complnins of'n dry cough anrl mild fever. His synptoms a1.r ex;icei-bated when h e wnl-kl; in tile fields. c~pr~ciallv T ~ . ~he I~II cortles inlo canracr rviih moldy hay. He does not smokc and drinks alcnhol occasionallv.

tabs

CEC: Icukncytosis ~ i t shift h to IrSt. Elcvi~trdESR; serum antibodies against thermophilic A c t i n o rnp ups organisms: hronchonlrcr~lar l a n g r sllr~~cu ni:lr kctl I~rnpliocvt nqir, primarily rirppl-rc.;oi--cytrhr )xic T r.ell5. P l T q : restriccivc lung disease pattern. G S R : bilateral retimlanodular inFi1trates with fihrnsis. CT: al-eas of grour~d-glass abr~orirlalities with c r n ~ r i l o b ~ ~ perihrnnchial lar nodule.;.
Fil~rosis wit11 honeycombing. B~-oncl~osct~pic I~iufi I>ir>psv r c ~ e i ~ interstilia1 ll; piier~tnr~iiia with I \ - m p h o ~ i canrl s plnslnii c ~ l l s in nl\,eolal-walls as well as sca ctei-cd g r a n ~ ~ l o l nwith a < Coreign body ~ i a tnccllq. Srricr ay(>idanccof cour;~ct with a ~ p e r g i l l spores; ~~s stesoiris. Ftypei-sensitivity pneuri~urtitis (;rllei.pjc.;~lverstit is) rrfrrq In in tcrstiti~~l Iung dircnsc thnt results 1rt)m inhali~tiouc ~ organic f a~~tigrn Hyersmrilivizy s. p i ~ e ~ ~ r n ic q~ i ci !>clievrcf ~ir tr) h a w a n irnrri~~r~olugic hasil; (t..g., qlutc~xic. irnmtule cvinplr?r, aild cellt~ie~liarerl reartinns) ; the most common Corm of hypersensitivity pneumonitis, called farmer's lung, is caused by inhalation o f a thermophilic A ctisnmjlces organism present in moldy hay and grain. C)t her cornrlro 11 cauqes nl t ~ p p ~ r s e n t ivi s i ry pnrumonitis i n c l ~ ~ rpigeon le Im-errIer's rlirease anrl Iil-cl Cancier'~ clisrasc, in which in halcd qcr-run protcir~s from pigeons or ~,a~-akt-eb ind!~re thc svi~rlmme. Huinirlilicr luilg rtiwasc ~ - c s u lii-om t ~ cxposurc to cont;~rninaterl Forced-air svrtrliil;.

Imaging

GTOSS Pathology Micro Pathology

Treatment

Discussion

HYPERSENSITIVITY PNEUMONITIS

A Ei5-yrar-c1ld ir~ale colnplai~ir nT pl-ojircssivc. shortness r l t ' I~reath on exer-tion and a chronic dry cough.

"The prjtient hns never smoked cigaretrr~ and has n o hirtory of exposure tn occupational dusts or fitnles; he has tint had a procluctivt. cough or-hrmoprysis.
VS: rvann but cyanosed: tachycarrlia (HR 108); tac!l)ipiiea: RP nor-ni;\l.PE: clubbing present; Jl'P not elevated: heart sounds ilormal with no a d c l i ~ i o ~sotincIs ~al or 1nltr.rnul.s;respiratory examination reveals presence nf hilatcral basal fine inspiratory crepitations.
Labs

hl3Gs: hypoxernia. P!LT5: decreased DL,,: clesaturatinn with exurcisc: p~-oporlional ely redt~cerl FEV, ancl FVC so that ratio rc=rriairlecl~lnchangecl(due t o rertrictivc cli~ease). l~rnnrhoalrenlar la~nge predominiinnrly neutrophilic; serum c;~lci~inl ancl /ICE levels Eo~v.

Imaging

CSR: reticiilo~lodnlaishadow in both lower lung fields with ncc:isir~nalarea5 of "honeycombing." CT (high I-esolt~tion) : lihrosis in Inwer lung 1ol)es suggestive of lislial interstitial pne~unonitis pattern of FPF.
Rronchoscopically obtained lurig biopsy rwealc; presence of 6hro3is, inflan~matory round cell infilrrare, ant1 thickening of chc alvcolai- sepba.

Micro Pathology

Treatment

Discussion

T h e rnain differential diagnoses to consicler are lung fibrosis aswciatetl with a cnilnective tissue disorder (rule nu1by history ;~nd clinical c-xam), extrinsic alveolitis due to organic dusts, leftsided Elcart failure, sarroidosis {rule nil1 otr the haris of ahscnce of anv othrr svsrern invofvei~~enl, nnrmal calcium and ACE lcwls, ncgative Kvei~n's test, a n d lack nC hilar 1yrnph;idcnopatliy ohrerved on CXR), lymphangitis carcinomatosa (rtite o u ~ on biopsy 2nd CT). a r ~ pneurnoccrnic>sis. l The onset of idiopathic pulinonat-v Fihmsis i s typically in the fifth or sixth rlecacle.

Atlas Link

I D I O P A T H I C PULMONARY F I B R O S I S (IPF)

lD/CC

,4 .?Rye-ar-oEd trlalc prcsentr with shortness of breath hoarseness, cough ancl hemoptysis.
:h,

(rnl;tlut,~)

H PI

H p has an 80-pack-year .smoking history. O v r r - thc past 2 mor ~ t s, h hc has a l ~ o harl a significant loss of appetite and weight.
Marker1 pallor: cachexia; clubbing: mild whet-zing at r r s t : c h r s ~ barrel shaped (ernphysrrnato~t~) ;mrl nlovemrnlq rliniiniuhed o n right; dullness to percussion rtyel- ~.iqht 111 iilclle Inl>c;no breath sounds Ilea1d oyrr r-igh t middle Ir~he: vncal freiiiitlis rerlucrd in \ : 1 1 1 1 e ;il'eil.
T C

PE

5
o z B

tabs

CBC: normocytic, normochrornic anemia. Clixrn a n d ZN r t x i n ~ rlf' spltturn fbr acid-f*ilstI~ilctll i neg:u ive: ylzlturn c l ~ c d rcveals o ~ presence of malignant squamow cells.

Imaging

CXR/C:T: i r r c ~ i 1 a 1 hililr nlass on tigfll side. pi~odiiring :III ohsunttiun iitrlec~asiu rlf r-ieht micldlp Inhe. Rroi1rhosct)pr: righl-sided liilar mass nhauucting right middle h r o n c l l ~ ~ s .
Pnnnu-gical sperimeil 1-eveaI4an irl-cpl;lr inv;tsivr m;ics o f g~~"yi$li-tan tumor spreiidillg 0111 Fr-oril rig111 tniddle hronchns ai~d ul~~tructin il. g

Gross Pathology
I

Micro Pathology

Diop~y revcaIs prcccncc o C n~alignanc squamaus rc511$,ccllit!:*rqtmtificatior~, intercell~llar bridges, anrl "keratin pearls."
Surgici~l rcwc tion can hc p o ~ c n ~ i a Icurative lv in par ien ts wit 11 nun-smalIccll 111112 C C I I C ~ T .

Treatment

Discussion

Lung cancrr is l h r most preventable cancer. Owins In rhe i~icrea~e incidellce d r)i srr~r>l;irip. Ii~ng cancel- has r x c ~ e r l e d brrilst carlrer a.i the Iradi~rg c a u s r c~f cancer rteath irl women. h Pancoast's trlmor i~a lilng nlnlor Ioci~tcd at tllr lu11,q ttpex in ~ h superior c p~rlmr~nar .;t~lc~tr y tli;~ I caltses corn presqintl nC the crr.vic.al syrtipa~hc~ic plex~ls, resl~lting in Horner's syndrome (ptosis. niimis. anI~idrosis) :IT well as sci~pt~lar paill a ~ ulnar d
ncrvc radiculc~pa~l-ty.

Atlas Links

r ' l PM-P2-085, PG-P2-085

LUNG CARCINOMA

A 67-yrar-nlrt tnak is referrecl to a clinic for evaluatint-i or pleuritic pain, weight 1 1 ~ 5pacl~ri~Fly , progre~sivedyspnea, and a nonproductive cough of a few rnon t h ~ rlz~ration. '
H e worker1 in ;I shipyard ror 20 years bcforc retiring. an orcupation that involved asbestos exposure.
VS: normal. PS: clubbing of fmprs: mild c!lannsis; reduced chest expansion; end-i~~spiratory rnles auscultated over bath lung fields; c l d percussion, reduced breath sotmdq, and egophony in right side { ~ t lto ~ p1~1lml r errtlsion). C:RC/PRS: pnlycyt hernia; marked eosinophilia, PFTs: restrictive pattern observcd (dccrrased vital capacity and decreased total l t r r g capacity i\.ilIinr)rmal FEV,/F'VC; razio). Rcduced diffusion capacity; pleural efftl~ion bloody a11dshorvs acidic pH (< 7.3).
Imaging

CXK: rigllc-sided pleural effrisiou; diffuse bilaleml interstitial fibrosis; parietal pleural calcifications. T=T: highly i r i - u p l a r p lel~r-al-hafed rnasseq; 11 e n ~ o s s h a g i c effusion.

Cross Pathology

Thick, fibrous pleural plaques with calc=cation; diffi~se interstitial fibrosis: asbestos cc~rnpol~nd\ Form nesr for further
c?epnuiiir,n nf il-or1 saltq and glvcoproteins ( m ~ l i r " r . r ~ c >ASRPSTOS us
l3( ~IIIT:~).

Micro Pathology

Epi tlrelioicl pattrrn of pletu-al malignant sarcomatous t l - a n s f i l r m a t i o n with cellular atypia and high mitotic itldex.
S l ~ s ~ ~poorr y : prognosis.

Treatment

Discussion

Occlipationd exposure to asbestos is found in 80% :,C cases of malignant mesotheliorna; it produces lung fibrosis wit11 a : restrictive pattern. Ashcstos and tobdcco expnwr 1-e syn~rgisticallv i n c i - c a s c thc risk of lung ~ ~ L ' I I O C ; I ~ C ~ T ~ O I I ~ ~ .

Atlas Links

MALIGNANT MESOTHELIOMA

ID/CC

complaining OF pIeuritic pain o n the r i g h ~ side nf llcr clievt i111t1 dyspnea logether- will1 fever- ; n ~ d ; i prc~dllcti~ co~lgh. e
, ; \ 57-yeal--old remale comes ro the ernergcnv room

HPI

TIi~re is n o hemnptvsis. T h e pail? i q nqicallv sharp and stabbing, a n d it nriws when 511e takes a deep Iwearh I K I ~ ~ ) .
(r31.r:t

PE

Decreased chest movement dtuing inhalation oil right side; dullness 011 percus5ion orright Iung base: reduced or absent breath sounds ores right lnng haw: I~ronchial I>rcath sounds in~sc~ tatctl t l 011 rig11t side; fr-iction ri~l>: Irrar ir )II r lf dullness moves with respiration; decreased tactile fremitus ovei- right

f
0

g <

lung.
Labs

CBC: clevatrcl U'EC count with prerlorriinancr o i 'neutl-nphils. Gram-poriiive tliplr~coccinn qplltllrn smear and clzlt~lre: elevated protein, decreased glucose, and many neakophils in pleural
exudate.

Imaging

CSR: c o n ~ l i d a t i o n of ri$t lower lobe: p1eur;il rff'i~sintl on right side. XI<. lateral ~ I e r ~ l ~ i layering i l ~ s : of fluid (tllereforc not loculated).
Antibiotics ailcF ncecllc drainagu nf cffitsic~rl( T H O I L I ~ : I ~ ~ K ~ ~ - F . ~ I ~ } : tc~inrrtimer ol>litrl.a~ iot~ nf plrllral space. Plctrri~l cfft~sions rrlav be d u e lo iril'cc~ion(viral. har rerial, nlyrolmctc-rial, h11iq1):o t h r r ciillsrs are ttlal ig11:111cies, congestive heart fililw-e, cirrhosis, nrphrotic synclrvnlr. tmirna, p a ~ i c ~ - ~is, a tcollagen il tfi5e;irt.r. atlrl tf!-~lq reactioils. E1T11sions1 1 1 m Ile transudative (< 5 ~ / r l L aT PI-otein) or- exudative (> 3 g JdL of prolei 11). F,I~vatedpleural flilirl LDIH Irvcls mar. he s~~ggustive of n-lalignarrcv.Tms~adative pleural efrusions arc cornmonty citusrd hy conges~i\.e hyart failure, cil.rI~n\is, ant-1 nel~hrotic svndmmc, wherea~ exudative pleural effi~sions are c;titsed by TB, infections. malignancr: panel-ca~itis. pulmonary emlmll~s, a n d chvlothnrax (milky pleural fluid).

Treatment

Discussion

PLEURAL EFFUSION

u-hitc male complains or sudden pIe~tritic chest ID/CC h 2-i-yrar-r>Id


pain and shortness of breath that awakens him at night.
HPI

I-Ie smokes onc pack of cigaretteq a clay and states that his patcrnaI uncle once had a similar episode.
Tall, thin p a r i e n ~diapl~oretic ; and fuels wcali; lrft chest expands poorly o n inspiration; trachca and apex heat dirrplaced to right; left qide hyperresonant to percussiorl: decreawd breath sounds; demeaqed tactile fremitrrs.
r\RC;s: rlecl-eased Po,; elevated Pc:o,.

PE

Labs
Imaging

CXR: partial collapse of left 111119 with no lung markings excupt thin line parallel to chest wan; costaphrenic sulcus a b n o m d y radiolucent ( ' ' I ~ E P P SI~L.CL~ SICS) S" in supine film.
Types: rraumatic, sponmrieous, lension. open: common causes: s~irgical puncture, rLiplilrr of eniphvsematous bullae, positive pressure mechanical vet~tilatinn.hro~~choploural fistula. Pnrumothorax wacuatiori via p l e ~ ~ r a catherer l (r,~-rr:s~ TUBE).

Gross Pathology

Treatment

Discussion

T I he (rsual cause of spnn taneous pneumothoras is rupture of a


subplerd ble b .

fD/tC

X -10-vrar-old III;IIP is Ijrn11g111 to [ h e ER nit11 cornplaintu c l f sudden-onset. severe right-sided chest pain followed by sevcre dirfic~llty breathing.

HPI

Hc i\ ;I c h ~ n t l i r smoker a n d has prerlolninal~tly emphysematous COPD.


-D

PE

I'S: .;cvurt=t;~chrcal-dia: t;~cliypnea;l~yporrnsinn: n o few-r. PE: cyanosis;trachea shifted to left; cheqi exam r ~ v c a l s l~yprrresonant percussion note on right, d t n i s h e d breath sounds. ;inti decreased tactile fernih~s.
1\113Gs: h!puxcmia: rc.;piriltor

5 0
z
7~ > <

Labs

alkatosis. F.C:C;: nnrmal.

Imaging

C X R (afttr- paticrzl 41 ahil i;.es) : right pneurnothorax compressing Ir~ng parenchyma and shifting of mediastinurn toward left. Flattrnecl lcft h c m i d i a p t ~ ~ + a ~ m .
Plcnral 5piI.c.c i u lilletl with air anrl 1n1tgis aaelecta~ic( t o r l r r n o ~ ~ ~ t~ r~ i ~~ ~e eu l n c i t h o atl aurupqv, a~ thc c11c.ut r.a\rilv is npentd illnrlcr watcr, lettiilg ail huhl~les rrcape). S~ctir,noI I i ~ n a slinws collap~ecl a1vrc)lar spnccs. Immerlintc- Tife-saving trcxlment conqihrs o f 'illsrr~inga wiclv-horc o i l I l ~ ai7kriecl e sirlr I(> rlecor~~prcss thc pleural ci~tqtv if ;I ~ I I P C Idrain is n o r irnmcdiatelv availaMe; rhr wiclr-hnr~ neetlle call then hc rep1:lcetl h~ a che\r drain cot~ilected to an nncln'watrr ~ r ; l l .

GKOSS Pathology

Micro Pathology
Treatment

I f ' c;~n r\ltl;i

Discussion

111 ~eilsionp~wrimntllorax, air entPrs thc pleural spacc d ~ ~ r i i i g inupiratinn anrl i s pl-eve11 ~ r from d cqr-aping cl~rring coxpir-alio~l { l > e r s ~at1 ~.; airwily ~ or ~~SFII flap C acts ;IS ;-I onc--wa~~ valve): rhel-ei s a pnq~,?-ossivt. incrc-nw it1 [ ~ l e u ~air: x l whir11 is unrlcr pressul-c (i.c., t e i i ~ i o ~T l )e . t ~ ~ pne11inot1lcrr-a~ i~n occllrs in onlv 1 tto 2% o r common manifestntiun of ilic barnts;inma. [ha[ m:m orcllr rluring prnitivr prcshure mechanicitl ~ e n ~ i l a t i n K ts ~k . L:lr;rt)l-s for ~poiltxr~eotl? p r ~ c ~ ~ r n o t h o rincludc+ax COPD, c y t i c Cil~rc lril;.

PNEUMOTHORAX-TENSION

ID/CC

A 34year-old white obese female complains of shomess of breath. dizzirloss, and near-hinting spells.
Shc has bee11 lltkirlg prescription medication for approximateIv 6 won ths in order to lose weight.

HPI

PE

Obeqitv; ~ ~ l iqanosis; ld large "a" wave in jug-uIar venous pressure; parastcrnal heave; loud S2: narrow splitting of 52; mles on both bases; hepatornega lv.
CBC: polycythemia. ECC:: right-axis deviation; right ventricle and right atrial hypertrophy, rLBGs: hvpoxemia.

Cabs

Imaging

CXR: enlarged light vcntriclc.: rnlarged tnain pulmonary artery with peripheral pruning.
Enlargccl right ventricle with mvocardial fiber hypcrtropky; atherosclerosis or pulmonary artery; narrc3wing of al-terioIes.
Atheromas in main elastic arteries. Thickening of the media and intima in rnedilinl size m u s c ~ ~ larteries. ar causing near-oblit~ration uf the lumen.

Gross Pathology

Micro Pathology

Treatment

Calcil~m channel blockers: prostacyclin; inhaled raitric oxide; phlrhntomy: heart-lung transplaillation can he cnnsidered.
Primary plxlrnonary hypertension is a p a t h o l a ~ c incr-ease in pulmonary artery pressme; if long-stanrling, i t raines latit1 right heart L~ilurc. It may he- primary (irliopathic) or secondary to intrinsic pulmonary disease.

Discussion

Atlas Link

U C D I PG-P2-090

P R I M A R Y PULMONARY HYPERTENSION

A ti(Lycar+lrl female who had ~ ~ n d c s g o n right e total hip replacement presents on t h e sixth pn5topcrativc day with ccnlral chest pain and acute-onset dyspnea.

She has been immobile since the wr-gei-y.


13:l o ~ ~ fever: ~ d .tachvcxr-dia: e tachypnea; hypotension.PE: r e n tral cyan osis: elevated JVP; right ventricular gallop rhythm with widely split 52.

Labs

.=C;'i:

hypoxia and hypescapnia {type 2 respiratory rnilure). FCC;: SIQST3 pat1ei.n and sirills tachycardia.
LJS, Doppler: clot in right common femoral vein. CXR: right lo~+,cllohc atrlcctasis. Y J Q :thrue it1.t.its of ventilation-prrft~sioi~ mismatch in right lung. Anpjv. p l ~ l ~ n o r ~ a cor~fir.malol-y: ry: not reqnired il'Y/Q can is high probability.
Large thramhus scrn in pulmonary artrry. Large ocrlz~sive I hl-omh ~ seen n in pi~lmonar-y artcrv wit11 mriabIc clegrcc of rrcani12ization.
Sltpy>or-r ive; ~lir-~!nboly tic 1hemp?: cot~sirler. ein hrhectoiny; heparin, Chun~adin, and low-molecular-~vcigl~t heparin ( cnclsapnrin ) insti tu terl for prop11ylaxi.i (monizc>rINR) .

Imaging

Gross Pathology
Micro Pathology

Treatment

Discussion

Pltlmnnal-y erlllboli mosr rommoiilv originate from pruxinlal deep venot 15 ~ h t - c m hosis. Pulmonary angiography is t i i t gold stanrl;~rdin tIlc rliawosis of pz~lrnntrary etnholism, h ~ obtain ~ t a V/Q cran initially if clinically suspcctrd. Vircbow's triad ouilines 111c risk Lictors S i r thrombus forn~atior~ and inc-lucieq hlnocl Ftasis f r .g., iriirnol1ili7alior1), enrlor helid rlanlage ( e . ~ . surgery) , , anrl hyperrnag-t~lahle states ( c . ~ malignancy, ., prcgnarxcynsew-re I3~1rnsl. Large emholi may c a u s c car~liovasculi~r cullapse I; lid uudderr r l ~ ; ~ t h .

Atlas Links

PULMONARY EMBOLISM

I D/CC

A 28year4d black female cotnplai n F of fever, dmnea, arthralgia, and erythernatouq, tender nodules on hnth legs.
She has no tlistory of Careign traiwl or curltact with a, ~lzherc~~lar patient.

HPI

PE 15: Ccver. PF: tcn clcr, erythematous nodules over extenwr aspects
of both legs (FR\TW.RI,~ vonos~mf): artIir.algia.r,of hot11 knees: rpleno1nega1y
Labs CBC: Iymphopenia; eosinophilia. L,ytes: elevated serum calcium; hypercafciuria. ACE levels elevated: 11Errnrl cultlrres negative; Mantoux test negative: 111ngal serolo~ negativc. PFTs: evidence of restrictive changes. T ~ - a n s l ~ r c ~ ~1~ Lc I~ l~ IX bit ia~ l psv ordered.

Imaging

CXR: hilateraT hilar lymphadenopathy aucl rig11r paratracheal aclenopathv; interstitial infiltrates: n o pleural eff~lsirsn.
Firm nt>clulcsonly a frw millimctel-sin size in affecrcd ol-ganq; can llecnrne con fluent a n d gite lise lo larger nnrlulcl;.

Gross Pathology

Micro Pathology

Lbmp t~ node biopsy reveals noncaseatinig granulomas with fil>r.oticacell~rlar core surr-ounded l,v I?mpI tocyles, r pit lrelioirf cells, ancl Lnt~gerlian's giant cells.
Corticosteroids.
111 tlnc United States, the incidence of sal-coidosis is highest in 13Tack wornen, with oTiset behveen 20 and 40 years of age. The diwast: rrlay ljc asyrnpturnatic: Iiowui-er, syrnptoIrls rnay he cnnsti~~tintial a n d [nay iinvolvp many di Cf'eren t organ svstems. including rhc l u n ~ slymph , nocles, skin, rvc, uppcr respiratory tr.acl, r~rticu1nendc)tlieliaI system, Iiver, kidneys. ilervow qstem, and heart. tZppl-oximatelp 60% to 7 1 3 % or sal-coidosispatients recover with T e ~ v or no residual wrnproms.

Treatment

Discussion

Atlas Link

mPM-PZ-092

SARCOIDOSIS

ID/CC

115Tb1~ear=>TrE t~lalc.presents T ~ ~ progrc.;sively LII increasing dyspnea xrlrl dry cough n o T severaI years' rlr~ralion.
Hc is a ~ ~ o r ~ s r n o kheu rt, his o c c ~ ~ l > a ~ i r11 ,n istot-? ; ~ l inrludes mining and quarrying.

HP1

PE

N{I l - l ~ ~ l > l ~wanorili, ing, or IympI~acterlc~patI~y reduced chest expansion 0x1 inqpirnrinn: d r y inspiratory crackles a~iscullated in upper Infwk of ht>th lrmgs.

0
Z

Labs

PITS: cornl,inetl ohsitrxrctive and restrictive pattern of functional iliip~irin~ I. n Rrrjnch o~c~picallv-~uicIcr1 lung biopsv cst;~ldi?hes rlinfi-nnsi~: negative Mantoux i r s t : sputlun cvtr>lov:;u~d stailling ~ O I acid-f'ls~ bacilli ~ ~ c j i a l i v e . CXR. PA: I-oundetls ~ n ; ~opaci~irc ll in tipper lobes with retraction anrl hilar Iyrnphadenapathy; "eggshell" calcification of lymph
nodes.

Imaging

Gross PathoZogy

Del~ru, small c c ~ l l a ~ r i ~iznrlules c ~ l ~ u in the IIPPPI.l ~ ~ n ill g s~ h early c T ~ ~ I ~FP~SI :P Xantl ~ hero~ne more rliff~ice as clistasi~ pr.o.qrp'cses. Hvalirlizetl whorl5 of c n l l a ~ e u ~ r . i l hlitlle o r n o inflammation: pc~lar-iserl ligh~ r1~t~lot1strate.c silica particles w i l l ~ i i tlo~lliles. ~ S ~ ~ y ~ p o r tavoitlarlce i\r: of I'r~rtll~r expnqllre

Micro PathoIogy

Treatment
Discussion

Thrrc is an increased incidence of tuberculosis it] rilicmis ~ x ~ i i e n tSilicosis s. Irxrls to rrstriclivr l u l ~ g tlisease { h a t varie5 in wvriilv FI.OTII ~iiilcl 10 dis:~l>li~jg.

+ SILICOSIS

Das könnte Ihnen auch gefallen